You are on page 1of 184

Copyright © YSCHOOL

This study module contains copyright subject matter proprietary to YSCHOOL,


the content development wing of YSCHOOL, India. No part of this module may be
reproduced, stored in a retrieval system or transmitted in any form or by any
means, electronic, mechanical, photocopy, recording, or otherwise, by anyone,
without prior written permission from YSCHOOL. Violators are liable to be
legally prosecuted.

2
3
CHEMISTRY SOME BASIC CONCEPTS OF CHEMISTRY

SOME BASIC CONCEPTS OF CHEMISTRY


Chemistry is defined as that branch of science which deals with the study of composition, structure and properties
of matter &changes which the matter undergoes under different conditions &laws which govern these changes.
Chemistry plays an important role in meeting human needs for food, health care products and other materials
aimed at improving the quality of life. This is exemplified by the large scale production of a variety of fertilizers,
improved varieties of pesticides and insecticides. Similarly, many lifesaving drugs such as cisplatin and taxol,
are effective in cancer therapy and AZT (Azidothymidine) used for helping AIDS victims, have been isolated
from plant and animal sources or prepared by synthetic methods.
Ancient Indian and Greek philosopher’s believed that the wide variety of object around us are made from
combination of five basic elements: Earth, Fire, Water, Air and Sky. Indian philosopher Kanad (600 BC) was of
the view that matter was composed of very small, indivisible particle called “parmanus”. Ancient Greek
philosopher also believed that all matter was composed of tiny building blocks which were hard and indivisible.
The Greek philosopher Domocritus named these building blocks as atoms, meaning indivisible.

MATTER
Matter may be defined as anything which has mass and occupies space. e.g. water, air, milk, salt, sand, etc.
Matter may be classified into 3 states namely solid, liquid and gas. At macroscopic level matter is:

FORMULA FOR SIMPLE COMPOUNDS

Sr. No. Name of Compound Symbols with valancy Formula


1. Calcium chloride Ca2 Cl1 CaCl2
Mg2(SO4)2 ~ MgSO4
2. Magnesium sulphate Mg2 SO42
(Simple Ratio)
3. Stannic Sulphide Sn4S2 SnS2
1 1
4. Potassium perchlorate K ClO 4 KClO4
5. Sodium Zincate Na1 ZnO22 Na2ZnO2
6. Magnesium bicarbonate Mg2HCO13 Mg(HCO3)2
7. Sodium carbonate Na1CO32 Na2CO3
8. Ammonium Oxalate NH14C2O24 (NH4)2C2O4
9. Sodium thiosulphate Na1S2O32 Na2S2O3
10. Potassium permanganate K1MnO14 KMnO4
11. Sodium Iodate Na1IO13 NaIO3
12. Sodium periodate Na1IO14 NaIO4

4
SOME BASIC CONCEPTS OF CHEMISTRY CHEMISTRY

SOME IMPORTANT REACTIONS


(i) Decomposition Reaction: A single compound breaks down into two or more elements or new
compounds.
 
CaCO3(s) ⎯
⎯→ CaO(s) + CO2(g) MgCO3(s) ⎯
⎯→ MgO(s) + CO2(g)
  
2NaHCO3 ⎯
⎯→ Na2CO3 + H2O + CO2 2HI ⎯
⎯→ H2 + I2 2NH3 ⎯
⎯→ N2 + 3H2

Note: Carbonates of 1st group elements i.e. Na, K, Rb, Cs do not decompose on heating.

Li 2CO3 ⎯⎯ → Li 2O + CO2

(ii) Single Displacement Reactions:


(i) It is a type of oxidation reduction reaction.
(ii) One element in a compound is replaced by another
(iii) 2 types
(a) cation displacement reaction
(b) Anion displacement reaction
Zn(s) + CuSO4→ZnSO4 + Cu Fe + CuSO4→FeSO4 + Cu
Pb + CuCl2→PbCl2 + Cu Zn + H2SO4→ZnSO4 + H2
Cl2 + 2NaBr → 2NaCl + Br2 Br2 + 2NaI→ 2NaBr + I2
(iii) Double Displacement Reactions: Cations and anions of two different compounds switch places,
forming two entirely different compounds.
NaCl + AgNO3→ AgCl↓ (white ppt.) + NaNO3 NaBr + AgNO3→AgBr↓ (pale yellow) +NaNO3
NaI + AgNO3→AgI↓ (yellow) + NaNO3 Na2SO4 + BaCl2→BaSO4↓ (white) + 2NaCl

Note:
1. Similarly, sulphides as HgS (Black), PbS (Black), Bi2S3 (Black), CuS (Black), CdS
(Yellow), As2S3 (Yellow), Sb2S3 (orange), SnS (Brown), SnS2(Yellow) give precipitate.
2. Carbonates of 2nd group elements also give precipitate.

(iv) Neutralization Reactions: Reaction between acids (contain replaceable H+) and bases (containing
replaceable OH–) is known as neutralisation reaction
Examples: NaOH + HCl→NaCl + H2O 2KOH + H2SO4→K2SO4 + 2H2O

CLASSIFICATION OF MATTER
Physical Classification – Solids, Liquids & Gases
Difference between gas & vapour Vapours represent a gaseous state of a substance which is a liquid at
room temp.
Chemical Classification – Homogeneous & Heterogeneous
• A material is said to be homogeneous if it consists of only 1 phase.
• A material is said to be heterogeneous if it consists of a number of phases.
• A material containing only 1 substance is called a “pure substance”.
• Materials containing more than 1 substance are not pure and are called “mixtures”.
Elements: Pure substance that contains only 1 kind of particles. These particles may be atoms or molecules.
Types: Metals, non-metals & metalloids.
Compounds: A pure substance containing 2 or more than 2 elements combined together in a fixed proportion by
mass and which can be decomposed into its constituent elements by suitable chemical methods.

5
CHEMISTRY SOME BASIC CONCEPTS OF CHEMISTRY

The properties of a compound are completely different from those of its constituent elements.
Mixtures:A material containing 2 or more substances in any proportion its called a mixture. The properties
of a mixture are the properties of its constituents. Further, a mixture can be separated into its constituents by
simple physical methods.
Types:A mixture is said to be homogeneous if its composition is uniform throughout. (Solution)
A mixture is said to be heterogeneous if its composition is not uniform throughout.
Atoms: The smallest particle of an element which may or maynot be capable of independent existence.
Molecules: The smallest particle of an element or a compound which can exist freely.
Molecules of elements: Made up of only 1 kind of atoms, called homo-atomic or homo-nuclear molecules.
Molecules of compounds: Made up of atoms of different elements, hence called hetero-atomic molecules.
Atomicity: No. of atoms present in 1 molecule of an element or a compound is called its atomicity.

Note:
1. The existence of an element in 2 or more chemically similar but physically different
forms is called allotropy. E.g. Diamond and Graphite
2. Existence of a compound in diff. crystalline forms is called polymorphism & diff.
forms are polymorphs. E.g. Zinc Blende (ZnS) and Wurtzite (ZnS)
3. Existence of diff. compounds with similar chemical composition is the same crystalline
form is isomorphism. E.g. ZnSO4.7H2O and FeSO4.7H2O, Na3PO4 and Na3AsO4
4. Substances which absorb moisture from air are called hygroscopic substances, e.g.,
anhydrous CuSO4, CaO, etc.
5. Solid substances which absorb a large amount of moisture from air so that they become
wet or pass into solution are called deliquescent and the phenomenon is called
deliquescence, e.g., NaOH, KOH, MgCl2, CaCl2, etc.
6. Some crystalline solids e.g. CuSO4.5H2O, Na2CO3.10H2O, etc. when exposed to air lose
their water partly or wholly. Such substances are called efflorescent and this property is
called efflorescence.

INTRODUCTION TO UNITS, DIMENSIONAL ANALYSIS, UNIT


CONVERSION METHOD, PRECISION AND ACCURACY
Prefixes:
deci (d) 10–1 Deka (da) 10
centi (c) 10–2 kilo (k) 103
milli (m) 10–3 mega (M) 106
micro () 10–6 giga (G) 109
nano (n) 10–9 tera (T) 1012
pico (p) 10–12 peta (P) 1015
femto (f) 10–15 exa (E) 1018
atto (a) 10–18 zeta (Z) 1021
zepto (z) 10–21 yotta (Y) 1024
yocto (y) 10–24

6
SOME BASIC CONCEPTS OF CHEMISTRY CHEMISTRY

Measurement of Physical Quantities


(Quantities which we come across during our scientific studies)
It consists of 2 parts: 1.The number 2. The unit
e.g.: If an object weighs 4.5 kg, it involves 2 parts: 4.5 i.e., the number & kg i.e., the unit.

Unit
A unit is defined as the standard of reference chosen to measure any physical quantity.

International System of Units (S.I. Units)


Seven Base Units Short-Cut
Metal Ti (M – Mass, E – Electric Current, T – Time, A – Amount, L – Length, T – Temp., i – Intensity)
Relation between oC&oF:
o
C = 5/9(oF – 32) or o
F = 9/5(oC) + 32
Example 1: Many countries including USA express the temperature of atmosphere in degrees Fahrenheit.If
you land in USA and temperature is 59 oF, what do you expect cold or hot. Compare with
corresponding temperature in India.
5 o 5
Solution: o
C= ( F – 32) = ( 59 − 32 ) = 15 oC
9 9
It will be cold.

Important Points about S.I. Units

1. The unit named after a scientist is started with a small letter and not with a capital letter. Eg.newton
not Newton
2. Symbols of the units do not have a plural ending like ‘s’. E.g. 10 cm and not 10 cms
3. Words and symbols shouldn’t be mixed. E.g. Either joules per mole or Jmol-1 and not Joules mol-1
4. Prefixes like kilo, deca, etc. are used with base units. E.g. Kilometer means 1000m
5. Unit written with a prefix and a power is a power for the complete unit. E.g. cm3 means (centimetre)3
6. Unit combinations are represented by leaving a small space between them. E.g. joule second as J s
not Js
7. Temperatures in Kelvin are expressed as 25 K and not 25oK
8. In explaining the text, the word ‘mole’ is used but when used as unit, it is written as ‘mol’.

Dimensional Analysis

Any calculation involving use of dimensions of diff. physical quantities involved is called Dimensional
Analysis.
1. To convert a physical quantity given in one type of units into some other units.
The method used is called factor label method or unit factor method or Unit conversion method.
It consists of following steps: First determine the ‘unit conversion factor/factors’.
E.g. For conversion of pounds into kg or vice versa, we use 1 = 2.205 lb/1 kg
or 1 = 1 kg/2.205 lb
Similarly, 1 inch = 2.54 cm then 1 = 2.54 cm/1 inch
or 1 = 1 inch/2.54 cm (unit conversion factors)

Note: Retain the units & multiply unit conversion factor with the quantity so that the desired units
remain after cancelling out.

7
CHEMISTRY SOME BASIC CONCEPTS OF CHEMISTRY

2. In solving Problems
Example 2: A man weighs 175 lb. Express his weight in kg. Given that 1 kg = 2.205 lb.
Solution: 1 kg = 2.205 lb
2.205lb 1 kg
 1= =
1 kg 2.205lb
1 kg
Hence, 175 lb = 175 lb  = 79.4 kg
2.205 lb
The units ‘lb’ cancel out from the numerator and denominator and the answer is in the required
units, i.e. kg.
Example 3: How many inches are there in 3.00 km? Given that 1 km = 1000 m, 1 m = 1.094 yd,1 yd = 36
inch.
Solution: The unit conversion factors will be:
1000 m 1 km 1.094 yd 1m 36 in 1 yd
1= = , 1= = , 1= =
1 km 1000 m 1m 1.094 yd 1yd 36 in
Here, the conversion involves a number of steps. Hence, the unit conversion factors are applied
in such a way that the units of the preceding factor cancel out. Thus,
1000 m 1.094 yd 36 in
3.00 km = 3.00 km    = 1.18 105 inch
1 km 1m 1 yd

Example 4: What is the mass in g of Al block whose dimensions are 2.0 in  3.0 in  4.0 in and whose
density = 2.7 g/cm3.
2.54 cm 1 in
Solution: Here, unit conversion factors are 1 = =
1 in 2.54cm
Hence, required mass (in g)
2.54 cm 2.54 cm 2.54 cm 2.7 g
= 2.0in  3.0 in  4.0 in     3
= 1.1103 g
1in 1in 1in 1 cm

DIFFERENCE BETWEEN PRECISION AND ACCURACY


The accuracy of any such measurement depends upon (i) accuracy of the measuring device used (ii) Skill of
its operator.
• If avg. value of diff. measurements is close to correct value, it is said to be accurate (Individual one may
not be close to each other.)
• If values of diff. measurements are close to each other and hence close to their avg. value, the
measurement is said to be precise. (Average value of different measurements may not be close to the
correct value.)
The precision depends upon the measuring device as well as the skill of the operator. Eg., suppose the actual length
of the room is 10.5 m. 4 different persons report the result of their 5 measurements as follows:
Measurement (m) 1 2 3 4 5 Average
Person A 10.3 10.4 10.5 10.6 10.7 10.5
Person B 10.0 10.1 10.2 10.3 10.4 10.2
Person C 10.1 10.3 10.5 10.7 10.9 10.5
Person D 10.0 10.7 10.9 11.1 11.3 10.8
Measurement by person A is both accurate and precise.
Measurement by person B has poor accurate but good precision.

8
SOME BASIC CONCEPTS OF CHEMISTRY CHEMISTRY

Measurement by person C has poor precision but good accuracy.


Measurement by person D has poor accuracy and poor precision.

SIGNIFICANT FIGURES:
The uncertainly in the experimental or the calculated value is indicated by mentioning the number of
significant figures.
Significant figures are the total number of digits in a number including the last digit whose value is
uncertain.
The uncertainly is indicated by writing the certain digit and the last uncertain digit. Suppose the volume of a
solution is 20.5 mL, 20 is certain and 5 is uncertain and the uncertainly is  in the last digit. All measured
quantities are reported in such a way that only the last digit is uncertain usually by  1.
Rules for Determining the Number of Significant Figures
a. All non-zero digits and zeros between non-zero digits are significant. For example,
• 876 m has three significant figures.
• 0.56 g has two significant figures.
• 4006 g has four significant figures.
• 4.06 cm has three significant figures.
b. Zeros to the left of the first non-zero digit in the number are not significant. Such zeros indicate the
position of decimal. For example,
• 0.04 m has one significant figure,
• 0.0063 kg has two significant figures,
c. If a number ends in zeros but these zeros are to the right of decimal point, then these zeros are significant,
For example,
• 6.0 m has two significant figures.
• 6.40 m has three significant figures.
• 5.400 g has four significant figures.
• 0.0500 kg has three significant figures.
d. Zeros at the end or right of a number are significant provided they are on the right side of decimal point,
otherwise the zeros are not significant figures.
• 0.400 g has three significant figures.
• 200 g has only one significant figure.
e. Exact numbers have an infinite number of significant figures. For example, in 4 chairs or 40 apples, there
are infinite significant figures as these are exact numbers and can be represented by writing infinite
number of zeros after placing the decimal, i.e. 4 = 4.000000 or 40 = 40.000000.
f. If a number ends in zero but these zeros are not to the right of a decimal point, then these zeros may or
may not be significant, e.g., 20600 g may have three, four or five significant figures. This ambiguity is
removed by expressing the value in the exponential form. The above mass can be written in three
different exponential forms known as scientific notation as follows:
• 2.06  104 g has three significant figures.

9
CHEMISTRY SOME BASIC CONCEPTS OF CHEMISTRY

or
• 2.060  104 g has four significant figures.
or
• 2.0600 104 g has five significant figures.
When numbers are written in scientific notation, the number of digits between 1 and 10 gives the number of
significant figures. Thus, in such cases, the significant figures of only the first factor are counted. Note that
all zeros to the right of a decimal point are significant. Thus, the general notation is N  10n , where N = a
number with a single non-zero digit to the left of the decimal point and n = an integer.
The above method of expressing a number is called exponential or scientific notation.
Addition and Subtraction of Significant Figures
The result should not have more digits to the right of decimal point than either of the original numbers. For
example.
22.22
15.0
2.02
________
39.244
_________
In the above case, 15.0 has only one digit after the decimal point. So, the result should be reported only upto
one digit after decimal point and therefore the result is 39.2.

Multiplication and Division of Significant Figures


For these two operations, the result should be reported with no more significant figures as are there in the
measurement with the few significant figures, e.g.
4.5  2.15 = 9.675
Since 4.5 has two significant figures, the result should not have more than two significant figures. Thus, the
result is 9.6
Similarly, for,
69.7  4.576 = 15.2316
The result is 15.2
Rounding off the Numbers

a. If the rightmost digit to be removed is more than 5, the preceding number is increased by 1, e.g., in
2.376, if 6 has to be removed, then the preceding number 7 has to be round off to 8. So, the result should
be 2.38.
b. If the rightmost digit to be removed is less than 5, the preceding number is not changed, e.g., in 5.464, if
4 is to be removed then the result is 5.46.
c. If the rightmost digit to be removed is 5, the preceding number will not be changed if it is an even
number, but it will increase by 1 if it is an odd number. For example, 7.75 is to be rounded by removing
5, increases 7 to 8 giving the result as 7.8. But if 7.85 is to be rounded off its result will be 7.8.

10
SOME BASIC CONCEPTS OF CHEMISTRY CHEMISTRY

Illustration 1.1: Calculate the following:

a. (6.7 105 )  (4.6 104 )

b. (7.6 107 )  (3.8 10−4 )

c. (6.8 10−3 )  (5.2 10−4 )

d. (6.7 105 )  (4.6 104 )

e. (7.6 107 )  (3.8 10−4 )

f. (6.8 10−3 )  (5.2 10−4 )

g. 7.65  102 + 2.72  103

h. 7.87 10−4 − 2.6110−5

Sol. a. (6.7  4.6)(105+4 ) = 30.82 109

b. (7.6  3.8) (107−4 ) = 28.88 103

c. (6.8  5.2) (10−3−4 ) = 35.36 10−7

6.7 105
d.
4.6 10 4
=
6.7
4.6
( )
 105−4 = 1.456 101 = 14.56.

7.6 107
e.
3.8 10 −4
=
7.6
3.8
( )
 107−( −4) = 2.0 1011

6.8 10−3
f.
5.2 10 −4
=
6.8
5.2
( )
 10−3−( −4) = 1.3 101 = 13.0

g. 7.65 102 + 2.72 103 = (7.65 + 2.72 101 ) 102 = (7.65 + 27.2) 102 = 34.85 102

h. (7.87 10−4 − 2.6110−5 )

= (7.87 10−4 − 2.6110−4 10−1 )

= (7.87 10−4 − 2.6110−4 )

= (7.87 − 2.61) 10−4 = 7.609 10−4 )

Illustration 1.2: How many significant figures are there in each of the following numbers?
a. π
b. The sum of 16.4 + 0.3254
c. The product of 12  7.435
d. 0.0075
e. 5.033  1022
f. 7.007

11
CHEMISTRY SOME BASIC CONCEPTS OF CHEMISTRY

g. 6000
h. The Structure of 19.3 – 0.4567
22
Sol. a. As π = = 3.1428571 …. hence it has infinite number of significant figures.
7
b. The result should have upto one decimal point as in 16.4. The sum is 16.7254. So, the result is 16.7.
So, it has three significant figures.
c. The significant figures. The product is 89.22. The result is reported with least number of significant
figures involved in the calculation (i.e., 12)
d. Two significant figure because the zero on the left of the first non-zero number are not significant.
e. Four significant figures because the first term gives the significant figures and the exponential term is
not considered.
f. Four significant figures because the zeros between the non-zero digits are significant figures.
g. Four significant figures, But in scientific or exponential notation significant figures vary, e.g.,

6.0 103 has two significant figures, 6.00  103 or 6.000  103 has three or four significant numbers,
respectively.
h. Three significant figures because the number with least significant figures involved in the
calculation (i.e., 19.3) has three significant figures.
Illustration 1.3: What is the difference between 2.0m and 2.00m.
Sol. a. There are significant different although they seem to be same. 2.0 m has two significant figures and
hence its precision is 0.1 part in 2, i.e., 50 ppt (parts per thousand). 2.00 m has three significant figures
and its precision is 0.01 parts in 2, i.e. 5 ppt. Hence, 2.00 m is more precise measurement than 2.0 m.

Illustration 1.4: Express the result of the following data to the appropriate number significant figures.
4.84  0.0744
6.016
4.84  0.0744
Sol. = 0.0598885
6.016
As 4.84 has least number of three significant figures, the result should contain three significant figures
only. Hence, the result after rounding off is 0.0599.

Illustration 1.5: The density of copper is 7.8 g cm−3 and its weight is 5.642 g. Report the volume of copper to
correct decimal point.
Mass 5.642
Volume = =
Density 7.8g cm−3
= 0.7233 cm3 = 0.72 cm3

12
SOME BASIC CONCEPTS OF CHEMISTRY CHEMISTRY

The result should have two significant figures because the least precise term (7.8) has two significant
figures.
Illustration 1.6: What is the number of significant figures in Avogadro’s number (6.0  1023) and Planck’s
constant (6.62 10−34 J s).

Sol. Avogadro’s number (6.0 1023 ) has 2 significant figures while Planck’s constant (6.62 10−34 J s) has
three significant figures since only significant figures of 6.0 and 6.62 are considered.
Illustration 1.7: Express the following to four significant figures.
i. 6.58768  105 ii. 8.35783
iii. 98.2350 iv. 0.003586
v. 90000
Sol. i. 6.587  105 and after rounding off 6.588  105 .
ii. 8.357 and after rounding off 8.358.
iii. 98.23 and after rounding off 98.24.
The rightmost digit to be removed is 5, and the preceding number is 3( odd number), so it is increased
by one.
iv. 0.003586(zeros after decimal point and to the left of a number are not significant).
v. 9.000  104
Illustration 1.8: Express the number 68000 in exponential notation which shows
i. Two Significant figures
ii. Three significant figures
Sol. i. 6.8 104 (Two Significant figures)

ii. 6.80 104 (Three Significant figures)


Illustration 1.9: An analytic balance has uncertainly in measurement equal to 1 mg. Then report the result in
terms of percentage if the weight of a compound is
a. 1 g b.10 g c.100 g
Sol. a. 1  0.1% b. 10  0.01% c. 100  0.001%
The uncertainly in measurement is expressed in terms of percentage by putting  sign before it, e.g, 150
1% . Etc. Smaller the quantity to be measured, greater is the percentage uncertainly, and the instrument
should be more precise for the measurement of smaller quantities.
1 1 1
1 mg = g= 100 = = 0.1 = 1  .01% .
100 1000 10

13
CHEMISTRY SOME BASIC CONCEPTS OF CHEMISTRY

LAWS OF CHEMICAL COMBINATION


Law of Conservation of Mass (Antoine Lavoisier, 1789)
In all physical & chemical changes, the total mass of the reactants is equal to that of the products. i.e.
Matter can neither be created nor destroyed. The mass and energy are inter-convertible but the total sum
of the mass and energy during any physical or chemical change remains constant.
E.g.: A reaction between AgNO3 solution and KI solution.
AgNO3 ( aq ) + KI ( aq ) ⎯⎯ → AgI + KNO3 ( aq )
(yellow ppt.)
Mass of AgNO3(aq.)+ Mass of KI(aq.)= Mass of the ppt. of AgI + Mass of KNO3(aq.)
Example 8: If 6.3 g of NaHCO3 are added to 15.0 g of CH3COOH solution, the residue is found to weigh
18.0 g. What is the mass of CO2 released in the reaction?
Solution: → CH3COONa+H2O +CO2 (1)
NaHCO3 + CH3COOH ⎯⎯
6.3 g 15.0
Residue=18.0g

Total mass of reactants = 6.3 + 15.0 = 21 .3 g


Total mass of products = 18 g + x g
According to Law of conservation of mass
21.3 g = 18 g + x
x = 21.3 g – 18
Thus mass of CO2 released in above equation = 3.3g
Example 9: When 20 g of NaHCO3 is heated, 12.62 g of Na2CO3 and 5.24g of CO2 is produced. How many
grams of H2O is produced?
Solution: NaHCO3 → Na 2CO3 + CO2 + H2O
20 g 12.62 g 5.24 g

Total mass of NaHCO3 heated = 20 gms;


Total mass Na2CO3 produced = 12.62 gms
Total mass of CO2 produced = 5.24 gms
 Mass of H2O produced = 20–12.62 –5.24 = 2.14 gms

Law of Constant Composition or Definite Proportion (J.L. Proust, 1799)


The composition of a compound always remains constant (i.e. the ratio of weights of different elements
in a compound) no matter by whatever method, it is prepared or obtained from different sources.
LIMITATIONS
1. It is not applicable if an element exists in diff. isotopes which may be involved in the formation of
the compound.
2. Elements may combine in the same ratio but compounds formed may be diff. E.g. C2H5OH & CH3OCH3
E.g.: CO2 can be formed by either of the following processes:

(i) By heating CaCO3: CaCO3 : CaCO3 ⎯⎯ → CaO + CO2

(ii) By heating NaHCO3: 2NaHCO3 ⎯⎯ → Na 2CO3 + H 2O + CO2
CO2 is collected separately as a product of each reaction and the analysis of each collection reveals
that it has the combination ratio of carbon and oxygen as 12 : 32 by weight.
Example 10: Silver chloride is prepared by
(i) Dissolving 0.5 gm of sliver wire in nitric acid and adding excess of hydrochloric acid to
sliver nitrate formed. The silver chloride precipitated is separated, washed and dried. The
weight of silver chloride is 0.66 g.

14
SOME BASIC CONCEPTS OF CHEMISTRY CHEMISTRY

(ii) Heating 1 g of silver metal in a current of dry chlorine gas till the metal is completely
converted into its chloride. It is found to weigh 1. 32 g. Illustrate the law of constant
composition by the above data.
0.5
Solution: % of Ag in AgCl in 1st case = 100 = 75.76%
0.66
% of Cl = 24.24%
1
% of Ag in AgCl in 2nd case = 100 = 75.76%
1.32
% of Cl = 24.24%
Since the % age of metal and chloride in both the cases is same. The law of definite proportion is
justified.
Example 11: The mass of copper oxide obtained by heating 2.16 g metallic copper with nitric acid and
subsequent ignition was 2.70.g In another experiment, 1.15 g of copper oxide on reduction
yielded 0.92 g copper. Show that the results illustrate the law of definite proportions.
Solution.
(a) Mass of copper = 2.70 g
Mass of copper = 2.16 g
Mass of oxygen = 2.70 - 2.16 = 0.54 g
Mass of copper 2.16 g 4
= =
Mass of oxygen 0.54 g 1
(b) Mass of copper oxide = 1.15 g
Mass of copper = 0.92 g
Mass of oxygen = 1.15 g – 0.92 g =0.23 g
Mass of copper 0.92 g 4
= =
Mass of oxygen 0.23 g 1
Since the ratio of masses of copper and oxygen in both the cases is the same, the law of definite
proportion is justified.
Example 12: When 50 g of ammonia is heated it gives 41.18 g of Nitrogen. When 10 g of Nitrogen is
combined with required amount of hydrogen it produces 12.14g ammonia. Show that the given
data follows the law of constant compositions.
Solution: If 50g of Ammonia gives 41.18g of Nitrogen, then the percentage of Nitrogen in ammoniais
41.18
100 = 82.36% .
50
If 10g of Nitrogen gives 12.14 g of Ammonia then percentage of Nitrogen in ammonia is
10
100 = 82.37% .
12.14
Law of Multiple Proportions (Dalton, 1804)

According to this law, when two elements A and B combine to form more than one chemical compound then
different weights of A, which combine with a fixed weight of B, are in a proportion of simple whole number.
E.g.: Nitrogen forms as many as five stable oxides. The analysis of these oxides (N2O, NO, N2O3,
N2O4 and N2O5) reveals that for 28 gm nitrogen, the weight of oxygen that combines is in the
ratio 16 : 32 : 48 : 64 : 80i.e., 1 : 2 : 3 : 4 : 5 in N2O, NO, N2O3, N2O4 and N2O5 respectively.

15
CHEMISTRY SOME BASIC CONCEPTS OF CHEMISTRY

Example 13: Two oxides of a certain metal were separately heated in a current of hydrogen until constant
weights were obtained. The water produced in each case was carefully collected and weighed. 2
grams of each oxide gave respectively 0. 2517 grams and 0.4526 grams of water. Show that these
results establish the Law of Multiple Proportions.
Solution: Step: 1
To calculate the mass of oxygen in each oxide:
Here, we are given:
Mass of each oxide = 2.0 g
Mass of water produced in case I = 0.2517 g
Mass of water produced in case II = 0. 4526 g
i.e., 18 g of water contain oxygen = 16 g
16
 0.2517 g of water contains oxygen =  0.2517 g = 0.2237 g
18
Step 2: To calculate the mass of oxygen which would combine with 1 g of metal in each oxide.
In Case I:
Mass of metal oxide =2g
Mass of oxygen = 0.2237 g
Mass of metal = 2 – 0.2237 = 1.7763 g
Mass of oxygen which combines with 1.7763 g of metal = 0.2237 g
0.2237
Mass of oxygen which combines with 1 g of metal g = 0.1259 g
1.7763
In Case II:
Mass of metal oxide = 2 g
Mass of oxygen = 0.4023 g
 Mass of metal = 2 - 0.4023 = 1.5977 g of metal = 0.4023 g
0.4023
 Mass of oxygen which combines with 1 g of metal = g = 0.2518 g
1.5977
Step 3: To compare the masses of oxygen which combine with the same mass of metal in the two oxides.
The masses of oxygen which combine with 1 g of metal in the two oxides are respectively 0.
1259 and 0.2518 g. these masses are in the ratio 0.1259: 0.2518 or 1:2
Since this is a simple ratio, so the above results establish the Law of Multiple Proportions.
Example 14: Carbon and oxygen are known to form two compounds. The carbon content in one of these
compounds is 42.9% while in the other, it is 27.3% Show that this data is in the agreement with
the law of multiple proportions.
Solution. In the first compound,
Mass of carbon = 42.9 g
Mass of oxygen = 100 – 42.9 = 57.1 g
In the second compound,
Mass of carbon = 27.3 g
Mass of oxygen = 100 – 27.3 =72.7 g
In the first compound,
Mass of carbon combining with 57 .1 g of oxygen = 42.9 g
42.9
Mass of carbon combining with 1.0 g of oxygen = 0.75 g
57.1
In the second compound,

16
SOME BASIC CONCEPTS OF CHEMISTRY CHEMISTRY

Mass of carbon combining with 72.7 g of oxygen = 27.3 g


27.3
Mass of carbon combining with 1.0 g of oxygen = 0.375 g
72.3
The ratio of mass of carbon combining with fixed mass of oxygen (i.e., 1 g) is 0.75: 0.375 or 2:1.
This is a simple ratio and therefore illustrates the law of multiple proportions.
Example 15: Sodium and oxygen combine to form 2 compounds of which one is Na2O. The percentage of
sodium in the other compound is 59%. Find the formula of this compound.
2  23
Solution: Percentage of sodium in Na2O is 100 = 74.2% and percentage of oxygen is 25.8%.
62
Percentage of sodium in other compound is 59% while that of oxygen is 41%. This means that in
the first compound (Na2O) if we take 100 gm then 25.8 gm of oxygen will be present therefore
74.2
the mass of sodium combining with 1g of oxygen would be = 2.87 g . Similarly in the second
25.8
59
compound the mass of sodium combining with one gm of oxygen is = 1.44 g. The ratio of
41
masses of sodium combining with the fixed mass of oxygen is 2.87: 1.44 = 2:1. Therefore
formula of the other compound is Na2O2.

Law of Reciprocal Proportions (Richter, 1792-94)

If two elements A and B combine separately with third element C to form two different compounds and
if A and B also combine together to form a compound then they do so in a ratio
of their masses equal or multiple or submultiples of ratio of their masses which
combine with a definite mass of C.
E.g.: Formation of H2S, H2O and SO2 can be done as follows,
(i) Hydrogen combines with sulphur forming hydrogen sulphide; 2gm.
of hydrogen reacts with 32gm of sulphur.
(ii) Hydrogen combines oxygen forming water; 2 gm. of hydrogen reacts with 16 gm. of oxygen.
(iii) Sulphur combines with oxygen forming sulphur dioxide; 32 gm. of sulphur reacts with 32 gm. of
oxygen i.e., in the ratio 32 : 32. This ratio is double of the ratio weights of these elements which
combine with 2 gm. of hydrogen. i.e., 32/16 : 32/32 = 2 : 1
Example 16: Ammonia contains 82.35% of nitrogen and 17.65% of hydrogen. Water contains 88.80% of
oxygen and 11.10% of hydrogen. Nitrogen trioxide contains 63. 15% of oxygen and 36.85% of
nitrogen. Show that these data illustrate the law of reciprocal proportions.
Solution. In NH3, 17.65 g of H combine with N = 82.35 g
82.65
 1 g of H combine with N = g = 4.67 g
17.65
In H2O, 11.10 g of H combine with O = 88.90 g
88.90
 1 g of H combine with O = g = 8.01g
11.10
Ratio of the masses of N and O which combine with fixed mass (1g) of H = 4.67:8.01 = 1: 1.72
In N2O3, ratio of masses of N and O which combine with each other = 36.85: 63.15 = 1: 1.74
Thus, the two ratios are the same. Hence, it illustrates the law of reciprocal proportions.

17
CHEMISTRY SOME BASIC CONCEPTS OF CHEMISTRY

Gay Lussac’s Law of Gaseous Volumes (Gay Lussac, 1808)


According to this law, when gases combine, they do so in volume which bears a simple ratio to each
other and also to the product formed provided all volumes are measured under similar conditions.
1
E.g.: (i) Reaction between hydrogen and oxygen. H2( g ) + O2( g ) ⎯⎯
→ H2O( v )
2
One volume of H2reacts with half volume of O2to give one volume of H2O.
(ii) Reaction between nitrogen and hydrogen. N2( g ) + 3H2( g ) ⎯⎯
→ NH3( v)
One volume of N2 reacts with three volumes of H2 to give two volumes of NH3.

Dalton’s atomic theory

1. All matter consists of indivisible particles called atoms.


2. Atoms of the same element are similar in shape and mass, but differ from the atoms of other elements.
3. Atoms cannot be created or destroyed.
4. Atoms of different elements may combine with each other in a fixed, simple, whole number ratios to
form compound atoms. Explains law of constant composition.
5. When atoms combine with one another to form compound atoms, they do so in simple whole no. ratios.
6. Atoms of 2 elements can combine in more than one ratio to form two or more compounds.Explains
law of multiple proportions.
7. The atom is the smallest unit of matter that can take part in a chemical reaction.
Explanation of laws by Dalton’s theory
1. Matter is made up of atoms which can neither be created nor be destroyed
2. As atoms combine with each other in simple ratio, therefore, all the ratios involved are simple which may
be same or some simple multiple of each other. This explains law of Reciprocal Proportions.
Drawbacks (Modern Atomic Theory)
1. The indivisibility of an atom was proved wrong: an atom can be further subdivided into protons,
neutrons and electrons. However, an atom is the smallest particle that takes part in chemical reactions.
2. Atoms of some elements vary in their masses and densities. These atoms of different masses are called
isotopes. For example, chlorine has two isotopes with mass numbers 35 and 37.

18
SOME BASIC CONCEPTS OF CHEMISTRY CHEMISTRY

3. Dalton also claimed that atoms of different elements are different in all respects. This has been
proven wrong in certain cases: argon and calcium atoms each have an atomic mass of 40 a.m.u.
These atoms are known as isobars.
4. Simple whole number ratios to form compounds is not observed in complex organic compounds like
sugar (C12H22O11).
5. The theory fails to explain the existence of allotropes; it does not account for differences in
properties of charcoal, graphite, diamond.
6. Atom is no longer indestructible.
Berzelius Hypothesis
Equal volumes of all gases under similar conditions of temp. and pressure contain equal no. of atoms
Avogadro’s Hypothesis/Law/Principle
Equal volumes of all gases under similar conditions of T & P contain equal no. of molecules.
Applications:
1. In the calculation of atomicity of Elementary Gases (No. of atoms contained in a molecule of gas)
2. To find the relationship between molecular mass and vapour density of a gas. Molecular mass = 2  V.D.
3. To find the relationship between mass and volume of a gas: Molecular mass = 2  V.D. = Mass of 1L
of gas at STP/0.089= 22.4 Mass of 1L of gas at STP = Mass of 22.4L of the gas at STP.
22.4L of any gas at STP (1 atm, 0 oC) weigh equal to the molecular mass of the gas expressed in gms.
Gram-Molecular Volume (GMV) Law

VAPOUR DENSITY
Density of vapour at same temp. & pressure
Vapour Density =
Density of H 2 gas at same temp. and pressure
1
Vapour density = × molecular mass
2
Example 17: The vapour density of gas A is four times that of B. If molecular mass of B is M, then molecular
mass of A is -
(A) M (B) 4M
(C) M/4 (D) 2M
Solution:(B) V.DA = 4 V.DB
MA M
=4 B
2 2
Example 18: Density of air is 0.001293g/c.c. Its V.D. is
(A) .001293 (B) 1.293
(C) 14.48 (D) None of the above.
Solution:(C) 0.001293 g/cc
1cc → 0.001293 g
22400 cc→ (0.001293  22400) g
28.9 g = mol. Mass
M 28.9
Now, V.D = = = 14.48
2 2

19
CHEMISTRY SOME BASIC CONCEPTS OF CHEMISTRY

Example 19: 22.4l of water vapour at NTP, when condensed to water, occupies an approx. Vol. of:
(A) 18l (B) 1l
(C) 1 ml (D) 18 ml
Solution:(D) 18g of water vapours occupy 22.4 l of volume
Now, 18g of water vapours condenses to form 18g of water
Now, density = 1 g/cc
 18g of water occupies 18 cc = 18 ml
Example 20: Density of chlorine relative to air is
(A) 2.44 (B) 3
(C) found exp. (D) 4
Solution:(A) Mol. Mass of air is 29 g
Relative density of chlorine =  Mass of Cl2  /  Mass of air  [Under same conditions of T and P]
 Vol. of Cl2   Vol. of air 
71 29 71
= / = = 2.44
22.4 22.4 29
Example 21: What is the V.D. of SO2 with respect to CH4
( MW )SO 2
Solution: V.D. =
( MW )CH 4

64
V.D. = =4
16
Example 22: 11.2 litre of the particular gas at S.T.P. weighs 16 gram. What is the V. D. of gas?
Solution: Wt.of11.2 litre = 16 gram.
11.2 16
moles = = M = 32 gm mole
22.4 M
32
V.D.= = 16
2

ATOMIC MASS & MOLE CONCEPT


Concept of Mole

(i) Definition: One mole is amount of a substance that contains


as many particles or entities as there are atoms in exactly 12
gram of the carbon (12C – isotope).
(ii) 1 mole  collection of 6.022 × 1023 particles or entities.
(iii) 1 mole atoms  1 gram-atom  gram atomic mass
(iv) 1 mole molecules  1 gram-molecule  gram molecular mass
(v) 1 mole ions  1 gram-ion  gram ionic mass

20
SOME BASIC CONCEPTS OF CHEMISTRY CHEMISTRY

Relative Atomic Masses:No. of times an atom of that element is heavier than an atom of C taken as 12.
1 amu = 1/12th of the mass of an atom or C-12 isotope
Note: The atomic mass of an element is the average relative mass of its atoms as compared with an atom
of C-12 taken as 12.

Some Important Atomic Masses:


H = 1, He = 4, C = 12, N = 14, O = 16, F = 19, Na = 23, Mg = 24, Al = 27, P = 31, S = 32, Cl = 35.5,
K = 39, Ca = 40, Ar = 40, Cr = 52, Mn = 55, Fe = 56, Cu = 63.5, Zn = 65, Br = 80, Ag = 108, I = 127,
Ba = 137, Pb = 207
Gram atomic mass: The atomic mass of an element expressed in gm
Molecular mass: No. of times the molecule of the substance is heavier than 1/12th the mass of an atom of C-
12 isotope.
or
Average relative mass of its molecules as compared with an atom of C-12 isotope taken as 12.
Gram molecular mass: The molecular mass of a substance expressed in gram.
Formula mass & Gram Formula mass: For ionic compounds

HOW BIG IS A MOLE?


One mole of marbles would cover the entire Earth (oceans included) for a depth of three miles. One mole
of $100 bills stacked one on top of another would reach from the Sun to Pluto and back 7.5 million times. It
would take light 9500 years to travel from the bottom to the top of a stack of 1 mole of $1 bills.

Atomic Weight & Atomic Mass Unit (amu)


(i) The atomic weight (or atomic mass) of an element may be defined as the average relative weight (or
mass) of an atom of the element with respect to the (1/12)thmass of an atom of carbon (mass number 12)
Weight of an atom of the element
Thus, atomic weight =  12
Weight of an atom of C (mass no. 12)
(ii) If we express atomic weight in grams, it becomes gram atomic weight (symbol g-atom).
(iii) 1 g-atom of any element contain NA number of atoms.
(iv) The atomic mass unit (amu or u) is defined as the (1/12)th of the mass of single carbon atom of mass
number 12.
Thus, 1 amu or u = 1.667 × 10–24 g = 1.667 × 10–27kg.

Molecular Weight and Formula Weight


(i) Molecular weight is defined as the weight of a molecule of a substance compared to the (1/12)th of
the mass of a carbon atom (mass number = 12).
(ii) In ionic compounds, as for example, NaCl, CaCl2, etc. there are no existence of molecules. For ionic
compounds, instead of "molecular weight" we use a new term known as "formula weight".
"Formula weight" is defined as the total weights of atoms present in the formula of the compound.
Example 23: Calculate the molecular mass of glucose (C6H12O6) molecule.
Solution: Molecular mass of glucose (C6H12O6) = 6 (12.011 amu) + 12 (1.008 amu) +6 (16.00 amu)
= 72.066 amu + 12.096 amu + 96.000 amu = 180.162 amu
Example 24: Calculate the formula mass of the following ionic compounds:
(i) Silver nitrate (ii) Anhydrous copper sulphate
[Atomic mass: Ag = 108, Cu = 63.5, S = 32, N = 14, O = 16]

21
CHEMISTRY SOME BASIC CONCEPTS OF CHEMISTRY

Solution:
(i) Formula of silver nitrate is AgNO3.
So, the formula mass of AgNO3
= Atomic mass of silver + Atomic mass of nitrogen + 3  Atomic mass of oxygen
= 108 +14 + 3  16 = 170 u
(ii) Formula of anhydrous copper sulphate is CuSO4
So, its formula mass
= Atomic mass of copper + Atomic mass of sulphur + 4  Atomic mass of oxygen
= 63.5 + 32 + 4  16 = 159.5 u
Example 25: Which of the following, in each case, has higher formula mass?
(i)Calcium carbonate and lead nitrate (ii) Barium chloride and magnesium sulphate
[Atomic mass: Ca = 40, C = 12, O = 16, Pb = 207, Ba = 137, Cl = 35.5, Mg = 24, N = 14, S = 32]
Solution:
(i) Formula mass of calcium carbonate (CaCO3) = 40 + 12 + 3  16 = 100 u
Formula mass of lead nitrate Pb(NO3)2 = 207 + 2 (14 + 3  16) = 331 u
So, lead nitrate has higher formula mass
(ii) Formula mass of barium chloride (BaCl2) = 137 + 2  35.5 = 208
Formula mass of magnesium sulphate (MgSO4) = 24 + 32 +4  16 = 120
So, barium chloride has higher formula mass.

Average Atomic Mass & Average Molecular Mass

(i) Average atomic mass: Let us consider, an element X, is available in the earth as isotopes of
nX
a1
, n X a 2 ,……, n X a n , the percentage abundance of the given isotopes in earth are x1, x2, ........, xn
respectively.
a1 x1 + a2 x2 + ...... + an xn
 The average atomic mass of X =
100

Example 26: Given that abundances of isotopes 54Fe, 56Fe and 57Fe are 5%, 90% & 5%, respectively, atomic
mass of Fe is-
(A)55.85 (B) 55.95
(C) 55.75 (D) 56.05
54  5 + 56  90 + 57  5 270 + 5040 + 285
Solution: (B) A= = = 55.95
100 100

Example 27: Calculate average atomic wt. of silicon if relative abundance is 92.23% Si28, 4.77%Si29, 3% Si30
92.23  28 + 4.77  29 + 3  30
Solution: Av at wt = = 28.1 amu
100
Example 28: Calculate % abundance of Ag109 if it is known that silver exist in only two isotopes Ag107&
Ag109& average atomic weight = 108.5
Solution: Let the % abundance of Ag109 be x
x 109 + (100 − x ) 107
 108.5 =
100
 10850 − 10700 = 2x  x = 75%

22
SOME BASIC CONCEPTS OF CHEMISTRY CHEMISTRY

(ii) Average molecular mass: Let us consider, in a container,


n1 moles of substance X1 (mol. wt M1) present
n2 moles of substance X2 (mol. wt M2) present
........................................................................
nn moles of substance Xn (mol. wt Mn) present hence, the total number of moles of substance present
in the container = n1 + n2 + ....... + nn
Total mass of the substance present in the container = n1M1 + n2M2 +….. + nnMn
j=n

n M
j=1
j j

M avg = j=n

n
j=1
j

Example 29: Dry air has a molar composition as 20% O2& 80% N2. Calculate average molecular wt. of dry air.
20  32 + 80  28
Solution: Av. Molecular wt = = 28.8
100
Use of Mole, GAM and GMM

(i) GAM = 1 gram-atom = 1 mole-atoms (GAM → gram atomic mass)


(ii) GMM = 1 gram-molecule = 1 mole molecules (GMM → gram molecular mass)
(iii) Molar mass: Mass of one mole of particles or entities of a substance is known as molar mass of a
substance.
Given mass w
(iv) No. of moles = = (When mass of substance is given)
Molar mass M
Given no. of particles N
(v) No. of moles = = (When no. of particles of substance are given) Here NA =
NA NA
Avogadro's No.
Volume of gas (in litres )
(vi) At STP: Number of moles (for ideal gas) =
22.711

Note:
1. According to IUPAC recommendations STP refers 273.15 K (or 0ºC) temperature and 1
bar pressure.
2. In old books STP refers 273.15 K and 1 atm pressure. Volume of 1 mole ideal gas at STP
is considered as 22.4 L.

23
CHEMISTRY SOME BASIC CONCEPTS OF CHEMISTRY

Example 30: The total number of electrons present in 1.6 gm. of methane is
(A) 6.02  1023 (B) 6.02  1022
(C) 6.02  1021 (D) 4.02  1020
Solution:(A) Number of GMM of methane in 1.6 gm. of methane
Mass of methane 1.6
= = = 0.1
Mol. mass of methane 16
Number of electrons in 1 molecule of methane (CH4) = 6 + 4 = 10
Hence, Number of electrons in 1.6 gm. of methane
= Number of GMM  6.02  1023  10 electrons = 0.1  6.02  1023 = 6.02  1023
Example 31: KClO3 on heating decomposes to KCl and O2. The volume of O2at STP liberated by 0.1 mole is
(A) 4.36 L (B) 3.36 L
(C) 2.36 L (D) None of these
Solution:(B) 2KClO3→ 2 KCl + 3O2
2 moles of on heating produces = 67.2 L of at STP
0.1 mole of on heating produces = 3.36 L of at STP
Example 32: 4.4 g of an unknown gas occupies 2.24 litres of volume at NTP. The gas may be
(A) Carbon dioxide (B) Carbon monoxide
(C) Oxygen (D) Sulphur dioxide
Solution:(A) Mass of 2.24 litres gas = 4.4 gm.
 Mass of 22.4 litres gas = 44 gm.
Here, out of four given gases, the molecular mass of only carbon dioxide is 44 gm.
Example 33: The number of gram molecules of oxygen in 6.02  1024 CO molecules is
(A) 10 gm molecules (B) 5 gm molecules
(C) 1 gm molecules (D) 0.5gm molecules
Solution:(B) 6.02  10 molecules = 1 mole of CO
23

 6.02  1024CO molecules = 10 moles of CO


= 10 gram atoms of Oxygen atom =5 gram molecules of O2
Example 34: 1 c.c of N2O at NTP contains:
1.8 6.02
(A) 1022 atoms (B) 1023 molecules
224 22400
1.32
(C) 1023 electrons (D) All the above
224
Solution:(D) 22400 c.c. = 6.02  1023 molecules
6.02 1023 3  6.02 1023
1 c.c. of N2O = molecules atoms (Since N2O has three atoms)
22400 22400
1.32
and 1023 electrons (Because number of electrons in are 22)
224
Example 35: The number of moles of oxygen in one litre of air containing 21% oxygen by volume under
standard conditions is
(A) 0.186 mole (B) 0.21 mole
(C) 0.0093 mole (D) 2.10 mole
Solution:(C) 100 ml of air at STP contains 21 ml of oxygen. 1000 ml of air at STP contains 210 ml of oxygen
So,No. of moles of oxygen = 0.0093 mole

24
SOME BASIC CONCEPTS OF CHEMISTRY CHEMISTRY

Example 36: The molecular mass of H2SO4 is 98 amu. Calculate the number of moles of each element in 294 g
of H2SO4.
Solution: Gram molecular mass of H2SO4 = 98 gm
294
Moles of H2SO4 = = 3moles
98
H2SO4 H S O
one molecule 2 atoms one atom 4 atoms
1  NA 2  NA atoms 1  NA atoms 4  NA atoms
 one mole 2 mole one mole 4 mole
 3 mole 6 mole 3 mole 12 mole
Example 37: Calculate the volume in litre of 20 g hydrogen gas at STP.
Given mass 20gm
Solution: No. of moles of hydrogen gas = = = 10 mol
Molecular mass 2gm
Volume of hydrogen gas at STP = 10 22.4 lt = 224 lt.
Example 38: How many g atom and no. of atoms are there in (a) 60 g carbon (b) 224.4 g Cu?
Given: At. Weight of C and Cu are 12 and 63.6 respectively. Avogadro’s no. = 6.02  1023.
wt wt.  Av.No.
Solution: g atom = and No. of atoms =
at.wt at.wt
60
(a) For 60 g C : g atom = =5
12
60  6.02 1023
No. of atoms = = 30.11023
12
224.4
(b) For 224.4 g Cu : g atom = = 3.53
63.6
224.4  6.02 1023
No. of atoms = = 21.24 1023
63.6
Example 39: Find the number of g atoms and weight of an element having 2  10 23
atoms. At. Weight of
element is 32.
Solution: NA atoms have 1 g atom
2 1023
2 1023 atoms have = = 0.33g atom
6.022 1023
NA atoms of elements weigh 32 g
2 1023  32
 2 10 atoms of element weigh =
23
= 10.628g
6.022 1023
Example 40: How many mole and molecules of O2 are there in 64g O2? What is the mass of one molecule of
O2?
Solution: Moles of O2 in 32 g O2 = 1
64 1
 In 64 g O2 moles = = 2 mole
32
32 g O2 contains 6.022 1023 molecules

25
CHEMISTRY SOME BASIC CONCEPTS OF CHEMISTRY

6.022 1023  64
 64 g O2 contains = = 12.044 1023 molecules
32
 NA molecules of O2 weigh 32 gm
32
 1 molecule of O2 weighs = = 5.3138 10−23 gm .
6.022 1023
Example 41: From 200 mg of CO2, 1021 molecules are removed. How many g and mole of CO2 are left?
Solution: 6.022 1023 molecules of CO2 = 44 g
44 1021
 1021 molecules of CO2 = g = 7.31  10–2 = 73.1 mg
6.022 1023
 CO2 left = 200 – 73.1 = 126.9 mg
wt. 126.9 10−3
Also Mole of CO2 left = = = 2.88 10−3
M.wt 44
Example 42: Arrange the following in order of increasing mass
(i)3.0115  1023 molecules of white phosphorus ( P4 ) (ii) 10 moles of H2 gas
(iii) 1 g molecule of anhydrous Na2CO3 (iv) 33.6 L of CO2 gas at S.T.P.
(A) ii i iv  iii (B) iii  vi iii
(C) i ii  iv  iii (D) i iv  iii  ii
Solution:(A)
(i) At. wt. of P = 31 and atomicity of P in white P is 4
Mol.wt. of white P = 31  4 = 124
 6.023  1023 molecules of white P weigh 124 g
124
 3.0115  1023 molecules of white P weigh  3.015 = 62g
6.023
(ii) Wt. of 1 mole of H2 gas = 2 g
Wt. of 10 moles of H2 gas = 2  10 = 20 g
(iii) 1 g molecule of anhydrous Na2CO3 = Mol wt. of Na2CO3 in g = 106 g
(iv) At STP 22.4 lit CO2 weighs 44 g
44
 33.6 lit CO2 weighs  33.6g = 66g
22.4
So, the correct choice , ii i iv  iii
Example 43: The number of millimoles contained in 0.160 g of NaOH is :
(A) 0.04 (B) 0.4
(C) 4 (D) 40
wt in gm 0.16 1000
Solution:(C) Number of millimoles = 1000 = =4
mol wt 40

26
SOME BASIC CONCEPTS OF CHEMISTRY CHEMISTRY

FORMULA SHEET FOR MOLE CALCULATIONS:


S. No. Items Formula
Wt. in gm
1. Molecules
Molecular mass
Wt. in gm
2. Atom
Atomic mass
Volume at STP
3. Gases
Standard molar Volume at STP
Number of particles
4. Any Particle
Avogadro number
5. Moles of A in AxBy x
For Gases at any P(in atm), PV
6. n= where R = 0.0821 lit-atm/mol K
V(in litres) & T(in K) RT
Minimum Molecular Weight:

It is the molecular weight of a compound shown by presence of minimum number of atoms


[i.e. for monomer = 1, Dimer = 2, Trimer = 3, Tetramer = 4]
e.g. Insulin contains 3.4% S, Find its MMW.
100g insulin contains 3.4g S (sulphur). Assuming it to be monomer [as nothing specified] one sulphur
atom32 amu (atomic mass) ,We can write , 3.4 g S is contained by 100g insulin
 100 
32g S is contained by   32  = MMW
 3.4 
Example 44: A compound contains 3.2% of oxygen. The minimum mol wt. of the compound is
(A) 300 (B) 440
(C) 350 (D) 500
Solution:(D) The compound must contain at least one oxygen atom
So, a minimum of 1 g atom of oxygen will be present in 1 g molecule i.e., 1 mole of the
compound.
If M is the mol. wt. of the compound then since 16 is the atomic mass of oxygen, so minimum of
16 g of oxygen will be present in M g of the compound
16 16 100
Thus, % of oxygen =  100 or 3.2 =
M M
or M = 500

STOICHIOMETRY
The word ‘Stoichiometry’ is derived from two Greek words - Stoicheion(meaning element) and
metron(meaning measure). Stoichiometry, thus, deals with the calculation of masses (sometimes volumes
also) of the reactants and the products involved in a chemical reaction. Before understanding how to calculate
the amounts of reactants required or those produced in a chemical reaction, let us study what information is
available from the balanced chemical equation of a given reaction. Let us consider the combustion of
methane. A balanced equation for this reaction is as given below:
CH4 (g) + 2O2 (g) → CO2 (g) + 2 H2O(g)
The above balance reaction gives the following information:

27
CHEMISTRY SOME BASIC CONCEPTS OF CHEMISTRY

• For every 1 mole of CH4, 2 mole of O2 will be required to produce 1 mole of CO2 and 2 moles of H2O.
this signifies Mole – Mole relation
• For every 16 gms of CH4 , 64 gms of O2 will be required to produce 44gms of CO2 and 36 gms of H2O
this signifies Mass – Mass relation
• Ratio of moles of CO2 : H2O at any time = 1 : 2
• There will be no change in total mass of all reactants and products at any time for any chemical reaction.
• For the above reaction only, there will be no change in total number of moles of all reactants and
products.
In order to solve the problems based on chemical calculations the following steps, in general, are quite
helpful.
• Write the balanced chemical equation.
• Write the atomic mass/molecular mass/moles/molar volumes of the species involved in calculations.
• Calculate the result by applying unitary method.

INTERPRETATION OF BALANCED CHEMICAL EQUATIONS


Once we get a balanced chemical equation then we can interpret a chemical equation by following:
1. Mass – mass analysis
2. Mass – volume analysis
3. Volume – volume analysis

Mass – Mass analysis:

Consider the reaction, 2KClO3 → 2KCl + 3O2


According to stoichiometry of the reaction,
Mass – mass ratio: 2  122.5 : 2  74.5 : 3  32
Mass of KClO3 2 122.5
or =
Mass of KCl 2  74.5
Mass of KClO3 2 122.5
=
Mass of O2 3  32

Example 45: 367.5 gram KClO3 (M = 122.5) is heated. How many gram KCl and oxygen is produced.
Solution: Balanced chemical equation for heating of KClO3 is
2KClO3 → 2KCl + 3O2
Mass – mass ratio: 2  122.5 gm 2 74.5 gm : 3  32 gm

Mass of KClO3 2 122.5 367.5 122.5


=  =
Mass of KCl 2  74.5 W 74.5
WKCl = 3 x 74.5 = 223.5 gm
Mass of KClO3 2 122.5 367.5 2 122.5
=  =
Mass of O2 3  32 W 3  32
Woxygen = 144 gm

28
SOME BASIC CONCEPTS OF CHEMISTRY CHEMISTRY

Example 46: How many grams of oxygen (O2) are required to completely react with 0.200 g of hydrogen (H2)
to yield water (H2O)? Also calculate the amount of water formed. (At. Mass H = 1; O = 16).
Solution: The balanced equation for the reaction is
2H2 + O2 ⎯⎯→ 2H2O
2 mol 1 mol 2 mol
4g 32 g 36 g
Now, 4g of H2 require oxygen = 32 g
32
0.200 g of H2 require oxygen =  0.200 = 1.6g
4
Again, 4g of H2 produce H2O = 36 g
36
0.200 g of H2 produce H2O =  0.200 = 1.8g
4
Example 47: What mass of zinc is required to produce hydrogen by reaction with HCl which is enough to
produce 4 mol of ammonia according to the reactions.
Zn + 2HCl ⎯⎯→ ZnCl2 + H2
3H2 + N2⎯⎯→ 2NH3
Solution: The given equations are Zn + 2HCl ⎯⎯→ ZnCl2 + H2
3H2 + N2⎯⎯→ 2NH3
From the equations it is clear that
2 mol of NH3 require = 3 mol of H2;3 mol of H2 require = 3 mol of Zn
Thus, 2 mol of NH3 require = 3 mol of Zn = 3  65 g of Zn
3  65
 4 mol of NH3 require =  4 = 390 g of Zn.
2
Mass – Volume analysis:

Now again consider decomposition of KClO3


2KClO3 → 2KCl + 3O2
Mass Volume ratio, 2  122.5 gm : 2  74.5 gm : 3  22.4 lt at S.T.P.
we can use two relation for volume of oxygen.
Mass of KClO3 2 122.5
= …(i)
volume of O2 at STP 3  22.4lt
Mass of KCl 2  74.5
and = …(ii)
volume of O2 at STP 3  22.4lt

Example 48: Calculate the volume of O2 and volume of air needed for combustion of 1 kg carbon at
STP.(Assume air contains 20% oxygen)
Solution: C + O2 ⎯⎯
→ CO2
12 g C requires O 2 = 22.4 litre of O 2 = 1 mole of O 2 = 32 g of O 2
22.4 1000
 1000 g C requires O2 = litre
12
= 1866.67 litre O 2
 Vair = 5  VO2 = 5 1866.67 = 9333.35litre

29
CHEMISTRY SOME BASIC CONCEPTS OF CHEMISTRY

Example 49: What volume of oxygen at N.T.P. can be produced by 6.125 g of potassium chlorate according to
the reaction 2KClO3⎯⎯→ 2KCl + 3O2.
Solution: The given chemical equation is:
2KClO3 ⎯⎯→ 2KCl + 3O2
2 mol 2 mol 3 mol
2  122.5 g 3  22.4L at N.T.P
Now 245 g of KClO3 produce oxygen at N.T.P. = 3  22.4 L
3  22.4
6.125 g of KClO3 produce oxygen = =  6.125 = 1.68 L at N.T.P.
245
Example 50: Assuming that petrol is octane (C8H18) and has density 0.8 g/ml, 1.425 litre of petrol on complete
combustion will consume
(A) 50 mole of O2 (B) 125 mole of O2
(C) 100 mole of O2 (D) 200 mole of O2
Solution: Mass of octane = 1.425  10  0.8 g
3

1425  0.8
Moles of octane = = 10 moles
114
25
C8H18 + O2⎯⎯→ 8CO2 + 9H2O
2
From the equation it can be seen
25
For 1 mole octane oxygen required = moles
2
25
 for 10 mole octane oxygen required =  10 = 125 moles
2
Volume – Volume Relationship:

It relates the volume of gaseous species (reactants or product) with the volume of another gaseous species
(reactant or product) involved in a chemical reaction.
Example 51: What volume of oxygen gas at NTP is necessary for complete combustion of 20 litre of propane
measured at 0oC and 760 mm pressure.
Solution: The balanced equation is
C3H8 + 5O2 = 3CO2 + 4H 2O
1vol 5 vol
1litre 5litre
1 litre of propane requires = 5 litre of oxygen
20 litre of propane will require = 5  20 = 100 litre of oxygen at 760 mm pressure and 0oC.
Example 52: The percentage by volume of C3H8 in a mixture of C3H8, CH4 and CO is 36.5. Calculate the
volume of CO2 produced when 100 mL of the mixture is burnt in excess of O2.
Solution: →3CO2 + 4H2O ( l )
C3H8 + 5O2 ⎯⎯
→ CO2 + 2H2O ( l )
CH4 + 2O2 ⎯⎯
1
CO + O2 ⎯⎯
→ CO2
2

30
SOME BASIC CONCEPTS OF CHEMISTRY CHEMISTRY

Let a mL, b mL and c mL be volumes of C3H8 ,CH4 and CO respectively in 100 mL given
simple, then a + b + c = 100 and a = 36.5
Now CO2 is formed as a result of combustion of mixture.
 1 vol. C3H8 gives 3 vol. CO 2 
 
 Vol. of CO2 formed = 3a + b + c  1vol. CH 4 gives 1 vol. CO 2 
 1 vol. CO gives 1 vol. CO 2 

= 3 36.5 + (100 − 36.5) = 173 mL

PRINCIPLE OF ATOM CONSERVATION (POAC):


POAC is based on law of mass conservation if atoms are conserved, moles of atoms shall also be conserved
hence mass of atoms is also conserved.
This principle is fruitful for the students when they don’t get the idea of balanced chemical equation in the
problem. This principle can be understand by the following example.
Consider the decomposition of KClO3(s) →KCl (s) + O2 (g) (unbalanced chemical reaction)
Apply the principle of atom conservation (POAC) for K atoms.
Moles of K atoms in reactant = moles of K atoms in products
or moles of K atoms in KClO3= moles of K atoms in KCl.
Now, since 1 molecule of KClO3 contains 1 atom of K
or 1 mole of KClO3contains 1 mole of K, similarly,1 mole of KCl contains 1 mole of K.
Thus, moles of K atoms in KClO3= 1 × moles of KClO3
and moles of K atoms in KCl = 1 × moles of KCl.
So, moles of KClO3 = moles of KCl
The above equation gives the mass-mass relationship between KClO3 and KClwhich is important in
stoichiometric calculations.
Again, applying the principle of atom conservation for O atoms,
moles of O in KClO3= 3 × moles of KClO3
moles of O in O2 = 2 × moles of O2
3 × moles of KClO3= 2 × moles of O2
wt. of KClO3 vol of O2 at NTP
or 3 = 2
mol. wt. of KClO3 standard molar vol (22.4 lt.)
The above equations thus gives the mass-volume relationship of reactants and products.
❖ Please note that the balanced reaction is essential in Mole method of solving.Only while using POAC
(which would be applicable only in certain cases), balanced reaction is not required.
❖ You can use POAC for all atoms in the compound only if all reactants and products are known.
Example 53: 27.6 g K2CO3 was treated by a series of reagents so as to convert all of its carbon to
K2Zn3[Fe(CN)6]2. Calculate the weight of the product. [mol. wt. of K2CO3 = 138 and mol. wt. of
K2Zn3 [Fe(CN)6]2= 698]
Solution: Here we have no knowledge about series of chemical reactions but we know about initial
reactant and final product accordingly
K2CO3 ⎯⎯⎯ → K2Zn3 [Fe(CN)6]2
Several
Steps

Since C atoms are conserved, applying POAC for C atoms,


moles of C in K2CO3 = moles of C in K2Zn3 [Fe(CN)6]2

31
CHEMISTRY SOME BASIC CONCEPTS OF CHEMISTRY

1 × moles of K2CO3 = 12 × moles of K2Zn3 [Fe(CN)6]2


(1 mole of K2CO3 contains 1 moles of C)
wt. of K 2CO3 wt. of the product
= 12 
mol wt. of K 2CO3 mol wt. of product
27.6 698
wt. of K2Zn3[Fe(CN)6]2 =  = 11.6 g
138 12
Example 54: A sample of KClO3 on decomposition yielded 448 mL of oxygen gas at NTP Calculate:
(i) weight of oxygen produced, (ii) weight of KClO3 originally taken
(iii) weight of KCl produced(K = 39, Cl = 35.5 and O = 16)
Solution:
448
(i) Mole of oxygen = = 0.02 Wt. of oxygen = 0.02  32 = 0.64gm
22400
(ii) KClO3 → KCl + O2
Applying POAC for O atoms, Moles of O atoms in KClO3 = moles of O atoms in O2
3 (moles of KClO3) = 2 (moles of O2)
(1 mole of KClO3 contains 3 moles of O and 1 mole of O2 contains 2 moles of O)
wt.of KClO3 vol. at NTP(litre)
3 = 2
mol. wt. of KClO3 22.4
wt.of KClO3 vol. at NTP (litre)
3 = 2 Wt. of KClO3 = 1.634 g
122.5 22.4
(iii) Again applying POAC for K atoms,
Moles of K atoms in KClO3 = 1 x moles of KCl
(1 mole of KClO3 contains 1 mole of K and 1 mole of KCl contains 1 mole of K)
wt. of KClO3 wt.of KCl
1 = 1 Wt. of KCl = 0.9937 g.
mol. wt. of KClO3 mol. wt. of KCl
Example 55: In a gravimetric determination of P, an aqueous solution of dihydrogen phosphate ion H 2 PO −4 is
treated with a mixture of ammonium and magnesium ions to precipitate magnesium ammonium
phosphate, Mg ( NH4 ) PO4 .6H2O . This is heated and decomposed to magnesium

pyrophosphate, Mg 2 P2O7 which is weighed. A solution of H 2 PO 4 yielded 1.054 g of
Mg 2 P2O7 . What weight of NaH2 PO4 was present originally?
Solution: NaH2PO4 + Mg2+ + NH4+ → Mg ( NH4 ) PO4 .6H2O ⎯⎯⎯
heated
→ Mg2P2O7
Since P atoms are conserved, applying POAC for P atoms, moles of P in NaH2 PO4 = moles of
P in Mg 2 P2O7
1 × moles of NaH2 PO4 = 2 × moles of Mg2P2O7
( 1 mole of NaH2 PO4 contains 1 mole of P and 1 mole of Mg 2 P2O7 contains 2 moles of P)
wt. of NaH 2 PO4 wt. of Mg 2 P2O7
=2×
mol. wt. of NaH 2 PO4 mol. wt. of Mg 2 P2O7
wt. of NaH 2 PO4 1.054
=2×
120 222
Wt. of NaH2 PO4 = 1.14 g.

32
SOME BASIC CONCEPTS OF CHEMISTRY CHEMISTRY

Example 56: What weight of CO is required to form Re2 ( CO)10 from 2.50 g of Re2 O7 according to the
unbalanced reaction: Re2 O7 + CO → Re2 ( CO)10 + CO2 (Re = 186.2, C = 12 and O = 16)
Solution: Suppose the relative moles of each reactant and product are as follows (just for convenience)
Re2 O7 + CO → Re2 ( CO )10 + CO2
a moles b moles d moles
cmoles

Applying POAC for Re atoms,


Moles of Re in Re2 O7 = moles of Re in Re2 ( CO)10
2 × moles of Re2O7 = 2 × moles of Re2 ( CO )10
2a = 2c or a=c …(i)
Applying POAC for C atoms,
Moles of C atoms in CO = moles of C in Re2 ( CO)10 + moles of C in CO2
1 × moles of CO = 10 × moles of Re2 ( CO)10 + 1 × moles of CO2
or b = 10c + d …(ii)
Applying POAC for O atoms,
Moles of O in Re2 O7 + moles of O in CO
= moles of O in Re2 ( CO)10 + moles of O in CO2
7 × moles of Re2O7 + 1 × moles of CO = 10 × moles of Re2 (CO)10 + 2 × moles of CO2
or 7a + b = 10 c + 2d …(iii)
From the eqns. (i), (ii), (iii), we get, 17a = b
i.e., 17 × moles of Re2O7 = moles of CO
2.50 wt. of CO in g  mol. wt. of Re2 O7 = 484.4 
17 × = . 
484.4 28  mol. wt. of CO = 28 
Wt. of CO = 2.46 g.
Example 57: 1.84 g of a mixture of CaCO3 and MgCO3 was heated to a constant weight. The constant weight
of the residue was found to be 0.96 g. Calculate the percentage composition of the mixture. (Ca
= 40, Mg = 24, C = 12, O = 16)
Solution: On heating CaCO3 and MgCO3, one of the products, CO2, escapes out.
We have, CaCO3 + MgCO3 → CaO+ MgO + CO2 
xg (1.84− x ) g yg ( 0.96− y ) g
Applying POAC for Ca atoms, Moles of Ca atoms in CaCO3 = moles of Ca atoms in CaO
1 × moles of CaCO3 = 1 × moles of CaO
x y CaCO3 = 100 
=  CaO = 56  …(i)
100 56  
Again applying POAC for Mg atoms,
Moles of Mg in MgCO3 = moles of Mg in MgO
1 × moles of MgCO3 = 1 × moles of MgO
1.84 − x 0.96 − y  MgCO3 = 84 
= …(ii)
84 40  MgO = 40 
From eqns. (i) and (ii), we get x = 1 g, y = 0.84 g
1
% of CaCO3 = × 100 = 54.34 %
1.84
% of MgCO3 = 45.66%

33
CHEMISTRY SOME BASIC CONCEPTS OF CHEMISTRY

CALCULATION OF PERCENTAGE COMPOSITION FROM FORMULA


The percentage of any element or constituent in a compound is the number of parts by mass of that element
or constituent present in 100 parts by mass of the compound. It can be calculated by the following two steps:
Step 1: Calculate the molecular mass of the compound from its formula by adding the atomic masses of the
elements present.
Step 2: Calculate the percentage of the element or the constituent by applying the following relation:
Percentage of the element or constituent
No. of parts by mass of the element or constituent
= 100
Mol. mass of the compound
Example 58: The minimum quantity of H2S needed to precipitate 63.5 gm of Cu2+ will be nearly.
(A) 63.5 gm (B) 31.75 gm
(C) 34 gm (D) 32 gm
Solution:(C) Since the ppt is of CuS, hence all the S atoms will be conserved
wH2S 63.5
 = moles of Sulphur = moles of Copper =
34 63.5
 wH 2S = 34 gm

Example 59: Calculate the percentage composition of the various elements in MgSO4.
Solution: Mol. Mass of MgSO4 = 24 + 32 + 4  16 = 120
No. of parts by mass of Mg 24
% of Mg = 100 = 100 = 20%
Mol. mass of MgSO4 120
No. of parts by mass of S 32
% of S = 100 = 100 = 26.67 %
Mol. mass of MgSO4 120
No. of parts by mass of O 64
% of O = 100 = 100 = 53.33%
Mol. mass of MgSO4 120
Example 60: Calculate the percentage of water of crystallisation in the sample of blue vitriol (CuSO4.5H2O).
Solution: Mol. Mass of CuSO4.5H2O = 63.5 + 32 + 4  16 + 5  18 = 249.5
No. of parts by mass of H2O = 5  18 = 90
90
 % of H2O = 100 = 36.07 %
249.5
Example 61: 13.4g of a sample of unstable hydrated salt: Na2SO4nH2O was found to contain 6.3g of water.
Determine the number of water of crystallisation.
(A) 6 (B) 5
(C) 7 (D) 8
wt. of water wt. of water in the compound
Solution: =
wt. of sample mol. wt. of the compound
6.3 n 18
=  n=7
13.4 142 + 18n

34
SOME BASIC CONCEPTS OF CHEMISTRY CHEMISTRY

Example 62: Calculate the percentage of cation in ammonium dichromate.


Solution: Molecular formula of ammonium dichromate is (NH4)2Cr2O7
Mol. Mass of (NH4)2Cr2O7= 2  (14 + 4) + 2  52 + 7  16 = 252
No. of parts by mass of cation viz NH4 = 2  (14 + 4) = 36
+

36 100
 % of NH+4 = 100 = = 14.29%
252 7
DETERMINATION OF MOLECULAR & EMPERICAL FORMULAE
The molecular formula of a compound may be defined as the formula which gives the actual number of atoms
of various elements present in the molecule of the compound. For example, the molecular formula of the
compound glucose can be represented as C6H12O6. A molecule of glucose contains six atoms of carbon,
twelve atoms of hydrogen and six atoms of oxygen.
In order to find out molecular formula of a compound, the first step is to determine its empirical formula from
the percentage composition.
Mass of that element
Mass % of an element = × 100
Molar mass

Empirical Formula:
The empirical formula of a compound may be defined as the formula which gives the simplest whole number
ratio of atoms of the various elements present in the molecule of the compound. The empirical formula of the
compound glucose (C6H12O6) is CH2O which shows that C,H and O are present in the simplest ratio of 1:2:1
Empirical formula mass of substance is equal to the sum of atomic masses of all the atoms in the empirical
formula of the substance. Molecular formula is a whole number multiple of empirical formula.
Thus Molecular formula = (Empirical formula) n;where n = 1,2,3…
Molecular Formula Molecular Mass
n= =
Empirical Formula Empirical Mass

Steps for writing the empirical formula:


The percentage of the elements in the compound is determined by suitable methods and from the data
collected, the empirical formula is determined by the following steps:
• Divide the percentage of each element by its atomic mass. This will give the relative number of moles of
various elements present in the compound.
• Divide the quotients obtained in the above step by the smallest of them so as to get a simple ratio of
moles of various elements.
• Multiply the figures, so obtained by a suitable integer if necessary in order to obtain a whole number ratio.
• Finally write down the symbols of the various elements side by side and put the above number as the subscripts
to the lower right hand corner of each symbol. This will represent the empirical formula of the compound.
Steps for writing the molecular formula:
• Calculate the empirical formula as described above.
• Find out the empirical formula mass by adding the atomic masses of all the atoms present in the
empirical formula of the compound.
• Divide the molecular mass (determined experimentally by some suitable method) by the empirical
formula mass and find out the value of n.

35
CHEMISTRY SOME BASIC CONCEPTS OF CHEMISTRY

Example 63: A substance, on analysis, gave the following percentage composition: Na = 43. 4%,C = 11.3%, O
= 45.3%. Calculate its empirical formula. {Na = 23, C = 12, O = 16]
Solution:
Atomic Relative number of Simple ratio Simplest
Element SYMBOL % age
Mass moles of moles whole no. ratio
43.4 1.88
Sodium Na 43.4 23 = 1.88 =2 2
23 0.94
11.3 0.94
Carbon C 11.3 12 = 0.94 =1 1
12 0.94
45.3 2.83
Oxygen O 45.3 16 = 2.83 =3 3
16 0.94
Therefore, the empirical formula is Na2CO3.
Example 64: A compound has the following composition: Mg = 9.76%, S = 13.01%, O = 26.01%, H 2O =
51.22%. What is its empirical formula? [Mg = 24, S = 32, O = 16, H = 1]
Solution:
Atomic
Relative number Simple Simplest Wholeno.
Element Symbol % age
Mass of moles ratioof moles ratio
9.76 0.406
Magnesium Mg 9.76 24 = 0.406 =1 1
24 0.406
13.01 0.406
Sulphur S 13.01 32 = 0.406 =1 1
32 0.406
26.01 1.625
Oxygen O 26.01 16 = 1.625 =4 4
16 0.406
51.22 2.846
Water H2O 51.22 18 = 2.846 =7 7
18 0.406
Hence, the empirical formula is MgSO4. 7H2O.
Example 65: A sample of pure compound is found to have Na = 0.0887 mole, O = 0.132 mole, C = 2.65 
1022atoms. The empirical formula of the compound is
(A) Na2CO3 (B) Na3O2C5
22
(C) Na0.0887O0.132C2.65  10 (D) NaCO
Solution:(A)
 6.02  1023 atoms of C = 1 mole of C
1 2.65 1022 2.65
 2.65  1022 atoms of C = mole = = 0.044 mole
6.02 10 23
6.02 10
Element Relative number of moles Simplest ratio of moles
0.887
Na 0.0887 =2
0.044
0.132
O 0.132 =3
0.044
0.044
C 0.044 =1
0.044
Thus, the empirical formula of the compound is Na2CO3.

36
SOME BASIC CONCEPTS OF CHEMISTRY CHEMISTRY

Example 66: An organic compound containing C, H and N gave the following on analysis: C = 40%, H =
13.3% and N = 46.67%. Its empirical formula would be
(A) CHN (B) C2H2N
(C) CH4N (D) C2H7N
Solution:(C) Calculation of empirical formula
Thus, the empirical formula is CH4N.
Relative number of
Percentage At. Mass Simplest Simplest whole
Element Symbol atoms = Percentage /
of element of elements atomic ratio number atomic ratio
At. mass
40 3.33
Carbon C 40 12 = 3.33 =1 1
12 3.33
13.3 13.33
Hydrogen H 13.3 1 = 13.3 =4 4
1 3.33
46.67 3.33
Nitrogen N 46.67 14 = 3.33 =1 1
14 3.3
Example 67: What is the simplest formula of the compound which has the following percentage composition.
Carbon 80%, Hydrogen 20%. If the molecular mass is 30, calculate its molecular formula.
Solution:
Relative number of Simple ratio of Simples whole
Element % age At. Mass
moles moles no.ratio
80 6.66
C 80 12 = 6.66 =1 1
12 6.66
20 20
H 20 1 = 20 =3 3
1 6.66
 Empirical formula is CH3
 Empirical mass = 12  1 + 1  3 = 15
Molecular mass 30
n= = =2
Empirical formula mass 15
Molecular formula = Empirical formula  2 = CH3 2 = C2H6
Example 68: 2.746 gm of a compound gave on analysis 1.94 gm of silver, 0.268 gm of sulphur and 0.538 gm
of oxygen. Find the empirical formula of the compound. (At masses: Ag = 108, S = 32, O = 16)
Solution: To calculate percentage composition.
Percentage composition of the compound is calculated as under:
1.94 0.268 0.538
Silver = 100 = 70.65% , Sulphur = 100 = 9.75% , Oxygen = 100 = 19.6%
2.746 2.746 2.746
To calculate empirical formula:
Relative number of Simplest whole
Element % age At. Mass Simplest ratio of moles
moles no.ratio
70.65 0.654
Ag 70.65 108 = 0.654 =2 2
108 0.305
9.75 0.305
S 9.75 32 = 0.305 =1 1
32 0.305
19.6 1.22
O 19.6 16 = 1.22 =4 4
16 0.305
 Empirical formula is Ag2SO4

37
CHEMISTRY SOME BASIC CONCEPTS OF CHEMISTRY

Example 69: An organic substance containing C, H and O gave the following percentage composition:
C = 40.687%, H = 5.085% and O = 54.228%. The vapour density of the compound is 59. The
molecular formula of the compound is
(A) C4H6O4 (B) C4H6O2
(C) C4H4O2 (D) None of these
Solution:(A)
At. Mass Relative number of Simplest Simplest whole
Percentage
Element Symbol of atoms = Percentage atomic number atomic
of element
elements / At. mass ratio ratio
40.687 3.390
Carbon C 40.687 12 = 3.390 =1 2
12 3.389
5.085 5.085
Hydrogen H 5.085 1 = 5.085 = 1.5 3
1 3.389
54.228 3.389
Oxygen O 54.228 16 = 3.389 =1 2
16 3.389
 Empirical formula is C2H3O2
 Empirical formula mass of = 59
Also, Molecular mass = 59  2 = 118
Molecular formula is C4H6O4

LIMITING REAGENT
In many situations, an excess of one or more substance is available for chemical reaction. Some of these
excess substances will therefore be left over when the reaction is complete; the reaction stops immediately as
soon as one of the reactant is totally consumed.
The substance that is totally consumed in a reaction is called limiting reagent because it determines or limits
the amount of product. The other reactant present in excess are called as excess reagent.
Let us consider a chemical reaction which is initiated by passing a spark through a reaction vessel containing
10 mole of H2 and 7 mole of O2.
Moles before reaction 10 7 0
Moles after reaction 0 2 10
The reaction stops only after consumption of 5 moles of O2 as no further amount of H2 is left to react with
unreacted O2. Thus,H2 is a limiting reagent in this reaction.
How to find Limiting Reagent:
Step 1: Divide the given moles of reactant by the respective stoichiometric coefficient of that reactant.
Step 2: See for which reactant this division come out to be minimum. The reactant having minimum
value is limiting reagent for you.
Step 3: Now once you find limiting reagent then your focus should be on limiting reagent.
Example 70: How much magnesium sulphide can be obtained from 2.00 g of magnesium and 2.00 g of sulphur
by the reaction Mg + S⎯⎯→MgS? Which is the limiting reagent? Calculate the amount of the
reactants which remains unreacted.
Solution: First of all each of this masses are converted into moles:
2.00
2.00 g of Mg = = 0.0824 moles of Mg
24.3
2.00
2.00 g of S = = 0.0624 moles of S
32.1

38
SOME BASIC CONCEPTS OF CHEMISTRY CHEMISTRY

From the equation, Mg + S ⎯⎯→MgS, it follows that one mole of Mg reacts with one mole of
S. We are given more moles of Mg than of S. Therefore, Mg is in excess and some of it will
remain unreacted when the reaction is over. S is the limiting reagent and will control the amount
of product. From the equation we note that one mole of S gives one mole of MgS, so 0.0624 mole
of S will react with 0.0624 mole of Mg to form 0.0624 mole of MgS.
Molar mass of MgS = 56.4 g
 Mass of MgS formed = 0.0624  56.4 g = 3.52 g of MgS
Moles of Mg left unreacted = 0.0824 –0.0624 moles of Mg
= 0.0200 moles of Mg
Mass of Mg left unreacted = moles of Mg  molar mass of Mg
= 0.0200  24.3 g of Mg = 0.486 g of Mg

Example 71: 4 mole of MgCO3 is reacted with 6 moles of HCl solution. Find the volume of CO2 gas
produced at STP. The reaction is MgCO3 + 2HCl → MgCl2 + CO2 + H2O
Solution: From the reaction MgCO3 + 2HCl → MgCl2 + CO2 + H2O
Given moles 4 mole 6 mole
Given mole ratio 2 : 3
Stoichiometric
Coefficient ratio 1 : 2
There should be one limiting reagent.
To find the limiting reagent, divide the given moles by stoichiometric coefficient.
MgCO3 HCl
4 6
=4 =3
1 2
HCl is limiting reagent.
moles of HCl moles of CO2 produced
=
2 1
 moles of CO2 produced = 3 moles
 volumes of CO2 produced at S.T.P, = 3  22.4 = 67.2L

Example 72: What weight of AgCl will be precipitated when a solution containing 4.77 g NaCl is added to a
solution of 5.77g of AgNO3 ?
Solution: AgNO3 + NaCl ⎯⎯
→ AgCl + NaNO3
5.77 4.77
mmoles. mixed 1000 1000
170 58.5
= 33.94 = 81.54 0 0
mmoles. left 0 47.60 33.94 33.94
 mmoles of AgCl formed = 33.94
w
1000 = 33.94
143.5
WAgCl = 4.87 g

39
CHEMISTRY SOME BASIC CONCEPTS OF CHEMISTRY

Example 73: When a solution containing 4.77 gm. of NaCl is added to a solution of 5.77 gm. of AgNO3, the
weight of precipitated AgCl is
(A) 11.70 gm (B) 9.70 gm
(C) 4.86 gm (D) 2.86 gm
Solution:(C) AgNO3 + NaCl ⎯⎯
→ AgCl + NaNO3
Moles before mixing
5.77 4.77
0 0
108 + 14 + 48 58.5
= 0.0339 = 0.0815
(Here AgNO3 is limiting reactant, thus)
Moles after mixing
0 0.0815 – 0.0339 0.0339 0.0339
= 0.0476
Moles of AgCl formed = 0.0339
Mass of AgCl formed = Mol. mass  No. of moles =1435  0.0339 = 4.864 gm
Example 74: The volume of oxygen at STP required to completely burn 30 ml of acetylene at STP is
(A) 100 ml (B) 75 ml
(C) 50 ml (D) 25 ml
Solution:(B) The balanced chemical equation for the reaction can be written as:
C2 H 2 + 5 / 2O2 ⎯⎯
→ 2CO2 + H 2O
1 Vol. 5/2 Vol.
1 ml. 5/2 ml.
30 ml. 30  5/2 = 75 ml
Hence, volume of the oxygen at STP required to burn 30 ml of acetylene at STP = 75 ml.
Example 75: What is the volume (in litres) of oxygen at STP required for complete combustion of
32 g of CH4?
(A) 44.8 (B) 89.6
(C) 22.4 (D) 179.2
Solution:(B) According to Avogadro’s hypothesis, volume occupied by one mole of any gas at STP is 22.4
litres.
CH4( g ) + 2O2( g ) ⎯⎯
→ CO2( g ) + 2H2O(l )
1 mole 2 moles
2 moles 4 moles
2  16 gm. = 32 gm. 4  22.4 litres = 89.6 litres

THEORETICAL YIELD AND PERCENTAGE YIELD


The theoretical yield is the maximum quantity of product that can be obtained, according to the reaction
stoichiometry, from a given quantity of a specified reactant.
The percentage yield of a product is the percentage of its theoretical yield achieved in practice.
actual yield
Percentage yield = 100%
theoretical yield
Suppose we find that in an actual combustion of 100 g of octane, only 92 g of carbon dioxide is produced,
then
92g of CO2
Percentage yield of CO2 = 100% = 30%
308g CO2

40
SOME BASIC CONCEPTS OF CHEMISTRY CHEMISTRY

If the percentage yield is very close to 100%, then we say that the reaction is complete or that it is
quantitative.

The role of a limiting reactant: only as much product (the finished cars) can be
produced as the limiting reactant (the car bodies) allows: Some quantity of
excess reagent (wheels) remains even after completion of reaction.

CONCENTRATION TERMS

(i) Density () = Mass of the substance


volume of the substance
In c.g.s. and MKS units, density is expressed in g/cm3 or g/ml & kg/m3 resp.
Density of any substance
(ii) Relative density=
Density of referencesubstance
Density of any subs tance
(iii) Specific gravity =
Density of H 2 O at 4º C
weight of solute
(iv) Weight by weight percentage (% w/w) or percentage by weight =  100
weight of solution
weight of solute
(v) Weight by volume percentage (%w/v) or percentage by strength=  100
volume of solution
volume of solute
(vi) volume by volume percentage (%v/v)or percentage by volume=  100
volume of solution
(Moles of solute )
(vii)mole percentage (% mol/mol) or percentage by mole =  100
Moles of solute + Moles of solvent

Note:
1. Do remember, for the calculation of strength (% w/w, %w/v etc) the solute must be
completely dissolved into the solution, otherwise, the given terminologies will be invalid.
2. If, anything is not specified, % strength generally means % by mass.

No. of parts of solute


(viii) Parts per million (PPM) =  10 6
Total no. of parts of all compounds in solution

41
CHEMISTRY SOME BASIC CONCEPTS OF CHEMISTRY

Note:
1. PPM is generally expressed as w/w (mass to mass)
2. PPM can also be expressed as w/v (mass to volume) or V/V (volume to volume)

(ix) gram per litre (gm/lit): It is the amount of solute in gm dissolved in 1 litre (1000 ml) of solution.
Number of moles of formula units of solute
(x) Formality =
volume of solution (in litres )

Number of moles of solute


(xi) Molality =
mass of solvent in kg
No. of moles of solute
(xii)Molarity =
Volume of solution (in litres )

10  x  d
Note: Molarity = ;
M1
w
Here x = % of solute, d = density of solution in gm/mL, M1 = molar mass of solute
w

Example 76: What volume of a 3.0 M HCl solution be mixed with 500 mL of a 7 M HCl solution to prepare a
HCl solution whose molarity will be 4.0?
Solution: Let V mL of 3.0 M HCl solution is taken, then
3 V + 500 × 7 = 4 × ( 500 + V)  V = 1500 mL
Example 77: Calculate the molarity of NaOH in the solution prepared by dissolving its 4 g in enough water to
form 250 mL of the solution.
No. of moles of solute
Solution: Since molarity (M) =
Volume of solution in litre
Mass of a NaOH/Mol or mass of NaOH
=
0.250 L
0.1mol 4 g / 40 g
= = = 0.4 mol L−1 = 0.4M
0.250 L 0.250 L
Example 78: 149 gm of potassium chloride (KCl) is dissolved in 10 L of an aqueous solution. Determine the
molarity of the solution (K = 39, Cl = 35.5)
Solution: Molecular mass of KCl = 39 + 35.5 = 74.5 gm
14.5 gm
Moles of KCl = =2
74.5 gm
2
Molarity of the solution = = 0.2 M
10
n solute n solvent
(xiii) Mole fraction:Xsolute= , Xsolvent =
n solute + n solvent n solute + n solvent

Note: Xsolute + Xsolvent = 1

42
SOME BASIC CONCEPTS OF CHEMISTRY CHEMISTRY

Example 79: Calculate the molarity of a solution of ethanol in water in which mole fraction of ethanol is
0.040.
n ( C2 H5OH )
Solution: xC2H5OH = = 0.040 (Given)
n ( C2 H5OH ) + n ( H 2O )
The aim is to find number of moles of ethanol in 1L of the solution which is nearly = 1L of water
(because solution is dilute)
1000 g
No. of moles in 1L of water = = 55.55 moles
18 g mol−1
Substituting n (H2O) = 55.55 in eqn. (i). we get
n ( C2 H5OH )
= 0.040 or 0.96n ( C2H5OH) = 55.55  0.040
n ( C2 H5OH ) + 55.55
or ( C2H5OH) = 2.31mol
Hence, molarity of the solution = 2.31 M

RELATION BETWEEN CONCENTRATION TERMS


1000 X1 1000M 1000 d X 1
(i) m = (ii) m= (iii) M =
(1 − X1 ) M 2 1000d − MM1 X1M1 + (1 − X 1 ) M 2
Here m = molality; M = molarity; d = density of solution in gm/mL; X1 = mole fraction of solute; M1 =
molar mass of solute; M2 = molar mass of solvent
w 
(iv) PPM = %  concentration  × 104 (v) Gram per litre = M × M1
w 
 w w  w
= % d
v  W 
(vi) Gram per litre = 10 × % (vii) %
 v 

Note: All these above relations (i – vii) are applicable only to binary solutions.

Strength
Amount of solute in g dissolved per L of solution. E.g. 10g NaOH dissolved to make 1L soln. has
strength 10g L-1.

Mass %
%w/w wt. of parts of solute in g dissolved per 100 g of solution

Volume %
%w/v wt. of parts of solute in g dissolved per 100 ml of solution

Volume %
% v/v volume of parts of solute in ml dissolved per 100 ml of solution

43
CHEMISTRY SOME BASIC CONCEPTS OF CHEMISTRY

Note: % w/w = % v/v dsolute/dsolution


PPM
wsolute/wsolution 106 = % strength  104
MOLARITY
M1V1 = M2V2
MIXING 2 SOLUTIONS
M = M1V1 + M2V2 / V1 + V2; M = (n1M1V1 – n2M2V2)/ V1 + V2
MOLALITY
Relationship between M & m: m = M/(d – M  MB/1000)

Example 80: Determine molarity of a solution obtained by mixing 50 mL of a 0.26 M H2SO4 solution with
another 150 mL 0.48 M H4SO4 solution.
Solution: It is a case of mixing of two solutions of different molarities. Applying the mixing formula:
M1V1 + M 2 V2 50  0.26 + 150  0.48
 M3 = = = 0.425 M
V3 50 + 150

Example 81: What volume of a 5.00 M H2SO4 solution should be added to a 150 mL 1.0 M H2SO4 solution
to obtain a solution of sulphuric acid of molarity 2.5?
Solution: It is again a case of mixing of 2 solutions. Let us assume that V mL of stock solution of H2SO4
is added.
 5V + 150 × 1.0 = 2.5 × (150 + V)  2.5 V = 225
225
 V= = 90 mL
2.5
Example 82: A 150 mL 0.25 M NaCl solution, 250 mL 0.45 M CaCl2 solution and a 100 mL 0.60 M AlCl3
solution are mixed together and diluted to a final volume of 750 mL by adding enough water.
Determine molarity of chloride ion Cl − ( ) in solution assuming that all three salts are completely
soluble as well as completely dissociated.
Solution: First we need to calculate total mmoles of Cl− from the three salt solutions:
mmoles of Cl− ion from NaCl = 37.5
mmoles of Cl− ion from CaCl2 = 2 × mmoles of CaCl2 = 2 × 112.5 = 225
mmoles of Cl− ion from AlCl3 = 3 × mmoles of AlCl3 = 3  60 = 180
 Total mmoles of Cl− in final solution = 37.5 + 225 + 180 = 442.5
mmoles of Cl– 442.5
 Molarity of Cl = –
= = 0.59 M
mL of solution 750
Example 83: 0.5 g of a substance is dissolved in 25 g of a solvent. Calculate the percentage amount of the
substance in the solution.
Solution: Mass of substance = 0.5 g Mass of solvent = 25 g
0.5
 Percentage of the substance (w/w) = 100 = 1.96
0.5 + 25

44
SOME BASIC CONCEPTS OF CHEMISTRY CHEMISTRY

Example 84: 20 cm3 of alcohol is dissolved in 80cm3 of water. Calculate percentage of alcohol in solution.
Solution: Volume of alcohol = 20cm3
Volume of water = 80cm3
20
 % of alcohol = 100 = 20 %
20 + 80
Example 85: What are the final concentration of all the ion when following are mixed 50 ml of 0.12 M
Fe(NO3)3+100 ml of 0.1 M FeCl3+100 ml of 0.26 M Mg(NO3)2?
50  0.12  3 + 100  0.26  2
Solution: [NO3–] =
250
18 + 52 70
= = = 0.28
250 250
[Cl–] = 0.12 M
[Mg++] = 0.104 M
[Fe3+] = 0.064 M
Example 86: Calculate the molarity of water.
Solution: H2O → 18 gm
= 1 mole
Volume of water = 1 Litre
Mass = 1000 gm
1000
Mole =
18
1000
Molarity of water = 55.55 M
18
Example 87: CaCO3 reacts with aq. HCl to give CaCl2 and CO2 according to reaction
CaCO3( s) + 2HCl( aq) ⎯⎯
→ CaCl2 + CO2 + H2O
How much mass of CaCO3 is required to react completely with 100 ml of 0.5 m HCl?
Solution: millimole of HCl
100 × 0.5 = 50
2 mole of HCl reacts 1 moles CaCO3
1
1 mole of HCl reacts → moles of CaCO3
2
1
50 mmole of HCl reacts →  50 = 25 mmol of CaCO3
2
25
mole of CaCO3 =
1000
25
mass of CaCO3 = × 100 = 2.5 gm.
1000
Example 88: 255 gm of an aqueous solution contains 5 gm of urea. What is the concentration of the solution in
terms of molality? (Mol. wt. of urea = 60)
Solution: Mass of urea = 5 gm
Molecular mass of urea = 60
5
Number of moles of urea = = 0.083
60

45
CHEMISTRY SOME BASIC CONCEPTS OF CHEMISTRY

Mass of solvent = (255 – 5) = 250 gm


Number of moles of solute
Molality of the solution = 1000
Mass of solvent in gram
0.083
= 1000 = 0.332
250
Example 89: The density of 3 M solution of NaCl is 1.25 g mL–1. Calculate molality of the solution.
Solution: M = 3 mol L–1
Mass of NaCl in 1 L solution = 3 × 58.5 = 175.5 g
Mass of 1L solution = 1000 × 1.25 = 1250 g (since density = 1.25 mL–1)
Mass of water in solution = 1250 – 175.5 = 1074.5 g
No. of moles of solute 3 mol
Molarity = = = 2.79 m
Mass of solvent in kg 1.0745 kg
Example 90: 255 gm of an aqueous solution contains 5 gm of urea. What is the concentration of thesolution in
terms of molality. (Mol. wt. of urea = 60)
Solution: Mass of urea = 5 gm
5
Molecular mass of urea = 60 Number of moles of urea = = 0.083
60
Mass of solvent = (255 – 5) = 250 gm
Number of moles of solute 0.083
 Molality of the solution = × 1000 = 1000 = 0.333
Mass of solvent in gram 250
Example 91: The molarity and molality of a solution are M and m respectively. If the molecular weight of the
solute is M, calculate the density of the solution in terms of M, m and M.
Solution: Let weight of solute be w g and weight of solvent is W g and volume of solution is V mL.
w 1000
 M= …(1)
M  V
w 1000
m= …(2)
M  W
w+W
D= …(3)
V
MMV
By Eq. (1) w= …(4)
1000
w 1000 MMV 1000
By Eq. (2) W= =
M  m 1000  M  m
MV
by Eq. (4) W= …(5)
m
MV MMV
+
1 M 
 By Eq. (3) D= m 1000 D = M +
V  m 1000 

46
SOME BASIC CONCEPTS OF CHEMISTRY CHEMISTRY

EQUIVALENT CONCEPT
CaH2 AlH3 NH3
  
2H – 2Ca 3H – 1Al 3H – 1N
2g – 40g 3g – 27g 3g – 14g
1g – 10g (Eq. wt.) 1g – 9g (Eq. wt.) 1g – 14 g (Eq. wt.)
3

98
NaOH + H2SO4→ NaHSO4 + H2O Eq. wt. of H2SO4 =
1
98
NaOH + H3PO4→ Na2HPO4 + H2O Eq. wt. of H3PO4 =
2
Eq. mass of an element in a compound

A1O1→ A2O2 BO2→ B2O4 AxOy→ A1Oy→ A 2 O 2 y


At. massAt. mass At. mass x
Eq. wt. of A = = EB = At. mass
Valency 2 4 EA =
2y / x
Wt.
Equivalence =
Eq. wt.
aA + bB→cC + dD

Equivalence of A = Equivalence of B = Equivalence of C = Equivalence of D E.g. Ca + O2 → CaO

Equivalence of Ca = Equivalence of O2=Equivalence of CaO


x y 2
= =
20 8 28
Example 92: 4.5 g Al reacts with ___ g O2 to give ____ g Al2O3.
Solution: Al + O2→ Al2O3
Equivalents of Al = Equivalence of O2 = Equivalence of Al2O3
4.5 w w
= =
9 8 17
Example 93: Calculate eq. mass of ozone ( E O3 ) in the following reaction:
2O3→ 3O2
Solution: 96g Oxygen is obtained from 96 g of Ozone,
So, 8g of oxygen is obtained from 8 g of Ozone,
So, EO3 = 8
Note: Amount which reacts or releases 8g oxygen is the eq. mass of a substance).
Example 94: Calculate the eq. mass of alum for the following change
K2SO4.Al2(SO4)3.24H2O + FeCl3→ K2SO4.FeAl(SO4)3.24H2O + AlCl3

47
CHEMISTRY SOME BASIC CONCEPTS OF CHEMISTRY

Solution: Al3+ is displaced by Fe3+ and +3 unit charge is displaced.


M Alum
EAlum =
3
Law of chemical Equivalence:

Equal equivalents of substances react with each other to give same equivalents of substances.
A+B→C+D
eq. of A = eq. of B = eq. of C = eq. of D

Salts

Equivalent weight = Molecular weight/’n factor’ where ‘n factor’ = Net cationic or anionic charge
1. Oxides: MgO, Li2O, Al2O3, CaO, Na2O, FeO, Fe2O3, BaO, K2O, ZnO,Mn2O3, etc.
2. Carbonates: CaCO3, MgCO3, Na2CO3, H2CO3, FeCO3, CuCO3, Li2CO3, BaCO3, etc.
3. Nitrates: NaNO3, Mg(NO3)2, Ca(NO3)2, Al(NO3)3, Cu(NO3)2, etc.

Using Equivalent concept

E.g.: In a reaction, Mg + O2→MgO, eq. of Mg = eq. of O2 = eq. of MgO, 12 g Mg loses 1 mole of e &
8 g O2 needs 1 mole of e
Now, total electrons lost by Mg = total electrons gained by O2. Using unitary method, we
getWMg/12 = WO2/8 = WMgO/20

Acids
Equivalent weight = Molecular weight/’n factor’ where ‘n factor’ = Replaceable H+ ions
1. HCl 2. H2SO4 3. H3PO4 4. H3PO3 5. H3PO2
6. H2SO3 7. H2CO3 8. HNO3 9.CH3COOH 10. (COOH)2
11. HClO4 12. NaHCO3 13. HClO3 14. HOCl 15. NaHSO4

Bases
Equivalent weight = Molecular weight/’n factor’ where ‘n factor’ = Replaceable OH- ions
1. NaOH 2. Ca(OH)2 3. Al(OH)3 4. Ba(OH)2
5. NH4OH 6. KOH 7. Zn(OH)2 8. CsOH

Normality

No. of Equivalents per L of soln.


Example 95: Equivalent mass of H3BO3 is:
(A) M/3 (B) M/1
(C) M/2 (D) None
Solution: (B)
Example 96: What is Ew of K2SO4.Al2(SO4)3.24H2O?
MW MW
(A) (B)
8 6
M M
(C) W (D) W
4 2
Solution: (A)

48
SOME BASIC CONCEPTS OF CHEMISTRY CHEMISTRY

Example 97: The normality of 0.3 M Phosphorus acid H3PO3 is:


(A) 0.1 (B) 0.9
(C) 0.3 (D) 0.6
Solution:(D) N=M2 [ H3PO3 is dibasic acid]

SPECIAL TOPICS
Volatile Chloride Method
2 V.D.
Valency, x =
( E + 35.5)
Dulong &Petit’s Method
Specific heat (in Cal)  Approx. Atomic wt. = 6.4
Example 98: The specified heat of a metal is 0.24 cal/g. This metal forms a hydrated sulphate which contains
8.1% metal and 43.2% SO 24− by mass. Determine the formula of hydrated sulphate.
Solution: Let the formula of hydrated sulphate = M2(SO4)x.yH2O
6.4 6.4
At. Mass of M = = = 26.67  27
Sp. heat 0.24
8.1 43.2
g eq. of M = g. eq. of SO 24− = , EM = 9
E M 46 / 2
At. mass 27
Valency of M = = =3
EM 9
So, formula of metal sulphate = M2(SO4)3 Formula of metal sulphate hydrate = M2(SO4)3.yH2O
Mass of H2O
Let us calculate ‘y’ % of H2O = 100
Mass of M2 ( SO4 )3 . yH2O
y.18 18 y
48.7 = 100 48.7 = 100, y = 18
Mass of M 2 (SO4 )3 + 18y 342 + 18 y
So hydrated sulphate = M2(SO4)3.18H2O
Example 99: Equivalent weight of an element is 4. Its chloride has a vapour density of 59.25. Then valency of
the element is:
(A) 4 (B) 3
(C) 2 (D) 1
2 V.D 2  59.25
Solution:(B) x=  x= =3
E + 35.5 4 + 35.5
Example 100: 1. 0 g of metal Nitrate gave 0.86 g of metal sulphate. Calculate the equivalent weight of metal.
(A) 38 (B) 62
(C) 48 (D) 96
Solution: (A) neq = neq
(nitrate) (sulphate)
1 0.86
=
EM + 62 EM + 48
 EM + 48 = EM 0.86 + 53.32

49
CHEMISTRY SOME BASIC CONCEPTS OF CHEMISTRY

 ( 0.14) EM = 5.32
55.32
 EM = = 38
0.14
Example 101: Specific heat of a solid element is 0.1 Cal/g °C and its equivalent weight is 31.8. It’sexact atomic
weight is
(A) 31.8 (B) 63.6
(C) 318 (D) 95.4
6.4
Solution: (B) Approx. At. wt. = = 64.
0.1
Now, Valency = 2.
Hence, Exact Atomic wt. = 63.6
Example 102: Specific heat of an element is 0.214 cal/g °C. Approx. atomic wt. is
(A) 0.6 (B) 12
(C) 30 (D) 65
6.4
Solution: (C) Approx. At. wt. = = 30
0.214
Example 103: Specific heat of a metal M is 0.25. Its eq. wt. is 12. It’s correct at wt
(A) 25.6 (B) 36
(C) 24 (D) 12
6.4
Solution:(C) Approx. At. wt. = = 25.6.
0.25
Now, Valency = 2.
Hence Exact Atomic wt. = 24
Cannizzaro’s Method
Smallest wt. of an element present in 1 mole of its various compounds gives either at. wt. of element or
its simple multiple.
Example 104: % of X in their compounds having molecular masses 222, 154, 300, 126 resp. are 27.9%, 20.2%,
43.7% & 24.6% resp. What is the atomic mass of X?
Solution: Cal. value of X in each compound. X comes out to be P = 31

Isomorphism
1. Valancies of 2 elements are same.
2. Masses of 2 elements that combine with the same mass of other elements in their respective
isomorphous compounds, are in ratio of their atomic masses. Mass of A/Mass of B = MA/MB
Example 105: 71g Cl combines with metal giving 111g of its chloride. It is Isomorphous with MgCl 2.6H2O.
Atomic mass of metal is-
(A) 20 (B) 30
(C) 40 (D) 69
Solution:(C) MA/24 = 40/24, therefore, MA = 40, which implies it is Ca
Example 106: Metal M forms a sulphate which is Isomorphous with MgSO4.7H2O. If 0.6538 g of metal M
displaced 2.16 gm of silver from silver nitrate solution, then the atomic weight of the metal M is
(A) 32.61 (B) 56.82
(C) 65.38 (D) 74.58

50
SOME BASIC CONCEPTS OF CHEMISTRY CHEMISTRY

Solution: (C) EquivalentsM = EquivalentsAg implies 0.6538/E = 2.16/108,


hence E = 32.69.
Valency =2, At. wt. = 65.38
Example 107: Carbonate of M is isomorphous with MgCO3 and contains 6.091% of C. Atomic weight of the
metal is nearly
(A) 48 (B) 68.5
(C) 137 (D) 120
Solution: (C) For MCO3, 197g has 12g C which is 6.091%.
Hence, M = 137

Hardness of Water

Temp. hardness due to HCO3-& permanent hardness due to chlorides, SO42-& NO3- of Ca & Mg
Unit of Hardness: Since the hardness of raw water is expected to be only a small quantity, it is generally
expressed in parts per million (ppm). Parts per million is a unit of salt (solute) per unit million unit
weights of solution (water containing dissolved salt). In dilute solutions where the density is
approximately one, 1 ppm = 1 mg/litre. To express hardness in terms of equivalents of CaCO3, all the
hardness in terms of equivalents of CaCO3, all the hardness causing impurities are first converted in
terms of their respective weight equivalents of CaCO3 and the sum of these is expressed in ppm.
The equivalent are as follows: 100g of CaCO3 = 111g of CaCl2 = 136g of CaSO4 = 95g of MgCl2 = 120g
of MgSO4 = 162g of Ca(HCO3)2 = 146g of Mg(HCO3)2 = 164g of Ca(NO3)2 = 44g of CO2

Note: CaCO3 is chosen as a standard to make the calculations easy as it has M. wt. = 100

Example 108: 1L sample of hard water contains 1mg of CaCl2& 1mg of MgCl2. Find hardness of water in terms
of PPM of CaCO3?
Solution: 1L = 1000g,
now Eq CaCl + Eq MgCl = Eq CaCO , cal. WCaCO .
2 2 3 3

−3 −3
10 10 W
+ =
111/ 2 95 / 2 100 / 2
 1 1 
 W = 102 10−3  +  = 10−1 ( 9 + 10 ) 10−3
 111 95 
= 19 10−4
19 10−4
PPM = 3
106 = 1.9 ppm
10
OLEUM (H2SO4 + SO3)

( x − 100)  80
% of free SO3 =
18
Here x = strength of oleum sample in percentage. (x is always greater than 100)

Oleum Labelling

Fuming (acid with excess gas) H2SO4


%SO3 = (l – 100)  40/9where 109% means 100g Oleum + 9g H2O

51
CHEMISTRY SOME BASIC CONCEPTS OF CHEMISTRY

Example 109: What is the volume of free SO3 obtained from 100 gm of oleum (considered as solution of SO3
in H2SO4) that is labelled with"109% H2SO4" at the pressure of 1 atm and 273K?

40
Solution: %SO3 = (109 – 100) = 40%,
9
1
hence WSO = 40g. Moles = , hence V = 11.2L
3
2
Volume Strength of H2O2
Volume Strength = 11.2  M = 5.6  N
Example 110: Find volume Strength of 1.5N H2O2 soln. Also cal. Vol. Strength of 17g/L H2O2 soln.
Solution: 5.6  1.5 = 8.4 17g/L = 1N, hence 5.6

EUDIOMETRY
Eudiometry or “gas analysis” involves calculations based on gaseous reactions in which at least two
components are gases & their amount is given in terms of volumes measured at same pressure
&Temperature. The relationship amongst gases, when they react with one another, is governed by two laws,
namely Gay-Lussac law and Avogadro’s law.

Avogadro’s Law

In 1812, Amedeo Avogadro stated that samples of different gases which contain the same number of
molecules (any complexity, size, shape) occupy the same volume at the same temperature and pressure. It
follows from Avogadro’s hypothesis that V  n (when T and P are constant).
Gaseous reactions for investigation purposes are studied in a closed graduated tube open at one end and the other
closed end of which is provided with platinum terminals for the passage of electricity through the mixture of gases.
Such a tube is known as Eudiometer tube and hence the name Eudiometry also used for Gas analysis.
During Gas analysis, the Eudiometer tube filled with mercury is inverted over a trough containing mercury.
A known volume of the gas or gaseous mixture to be studied is next introduced, which displaces an
equivalent amount of mercury. Next a known excess of oxygen is introduced and the electric spark is passed,
whereby the combustible material gets oxidised. The volumes of carbon dioxide, water vapour or other
gaseous products of combustion are next determined by absorbing them in suitable reagents. For example, the
volume of CO2 is determined by absorption in KOH solution and that of excess of oxygen in an alkaline
solution of pyrogallol. Water vapour produced during the reaction can be determined by noting contraction in
volume caused due to cooling, as by cooling the steam formed during combustion forms liquid (water) which
occupies a negligible volume as compared to the volumes of the gases considered. The excess of oxygen left
after the combustion is also determined by difference if other gases formed during combustion have already
been determined. From the data thus collected a number of useful conclusions regarding reactions amongst
gases can be drawn.
(a) Volume-volume relationship amongst Gases or simple Gaseous reactions.
(b) Composition of Gaseous mixtures.
(c) Molecular formulae of Gases.
(d) Molecular formulae of Gaseous Hydrocarbons.
The various reagents used for absorbing different gases are
O3⎯→ turpentine oil O2⎯→ alkaline pyrogallol NO ⎯→ FeSO4 solution
Cl2, CO2, SO2⎯→alkali solution (NaOH, KOH, Ca(OH)2, HOCH2CH2NH2, etc.)
NH3→ Water H2O → CuSO4, CaCl2 CO → Ammonical Cu2Cl2

52
SOME BASIC CONCEPTS OF CHEMISTRY CHEMISTRY

GENERAL ASSUMPTIONS:
In all problems, it is assumed that the sparking occurs at room temperature. This implies that water formed
would be in liquid state and that nitrogen gas is inert towards oxidation. Also, if a gas is added externally, it
will be assumed to be in excess unless otherwise stated.

Determination of Molecular formula of Hydrocarbon using Eudiometry

A known amount of hydrocarbon is taken into an Eudiometry tube. O2 gas is then inserted to cause complete
combustion of hydrocarbon & the reaction mixture is cooled back to the original room temperature. This
gives 1st volume contraction VIC. The resultant gaseous mixture is then passed through alc. KOH which gives
second volume contraction VII c. These data can help to calculate the molecular formula of the hydrocarbon
as explained below.
Step 1: Write down the balanced chemical reaction.
 y y
Cx H y +  x +  O2 → xCO2 + H 2O
 4 2
Step 2: Write down Volume of components before the reaction
VHC VO2 0 −
Step 3: Write down volume of components after the reaction using Avogadro’s Law,
(after identifying limiting Reagent)
 y
0 VO2 −  x +  VHC xVHC −
 4
Step 4: Use the data given
VIC = Vbefore – Vafter [due to change in volume because of reaction]
  y   y
= VHC + VO2 − VO2 −  x + VHC + xVHC  = VHC 1 +  =
  4   4
This will give the value of y
VIIC = due to change in volume because of absorption of CO2 in alc KOH. VIIC = xVHC
Hence, both x & y can be calculated.
Example 111: A gaseous hydrocarbon requires 6 times its own volume of O2 for complete oxidation and
produces 4 times its volume of CO2. What is its formula?
Solution: The balanced equation for combustion
 y y
Cx H y +  x +  O2 → xCO2 + H 2O
 4 2
 y y
1 vol.  x +  vol x + = 6 , or 4x + y = 24..............(1)
 4 4
Again x = 4 since evolved CO2 is 4 times that of hydrocarbon
 16 + y = 24 or y = 8
 Formula of hydrocarbon C4H8

Note: On cooling if volume of gaseous mixture decreases then this is because of condensation of H2O(V).

53
CHEMISTRY SOME BASIC CONCEPTS OF CHEMISTRY

Example 112: 10 ml gaseous CxHy + 80ml O2 at NTP is given. Remaining gas vol. is 70ml which further
reduces to 50ml on passing through KOH. Find formula?
Solution: Vol. of CO2 used = 20ml and that of O2 used = 30ml. On solving we get, C2H4
 y y
Cx H y +  x +  O 2 → x CO2 + H 2 O
 4 2
10 80
 y
0 80 −  x + 10 10 x −
 4
Now, 10x = 20  x=2
Also, remaining volume = 70
 y
 70 = 80 −  x + 10 + 20
 4
 y y
 x + 10 = 30  2+ = 3
 4 4
y
=1  y=4
4
Example 113: 100 ml of CH4 and C2H2 were exploded with excess of O2. After explosion and cooling, the
mixture was treated with KOH, where a reduction of 165 ml was observed. Therefore the
composition of the mixture is -
(A) CH4 = 35 ml; C2H2 = 65 ml (B) CH4 = 65 ml ; C2H2 = 35 ml
(C) CH4 = 75 ml; C2H2 = 25 ml (D) None
Solution:(A) Let CH4 = y ml then POAC on C, 1y + 2(100 – y) = 165 then y = 35ml

Victor Mayer Method

Example 114: In victor Mayer’s method 0.2g volatile compound on volatilisation gave 56ml of vapour at STP.
Its molecular wt. is-
(A) 40 (B) 60
(C) 80 (D) 120
0.2 56
Solution:(C) =
M 22400
0.2  22400
 M= = 80
56
Answer is 80

Ionic Strength:
Ionic strength of a soln. is a measure of conc. of ions in that soln.

, where ci is conc. of ith species and zi is the charge on ith species

54
SOME BASIC CONCEPTS OF CHEMISTRY CHEMISTRY

Example 115: Cal. ionic strength of soln. containing 0.2 M NaCl& 0.1 M Na2SO4
Solution: For Na+ = ½(0.2 + 2  0.1) 22 = 0.8

Specific Heat Method


For mono-atomic gas, ϒ = 5/3
Diatomic gas, ϒ = 7/5
Polyatomic gas, ϒ = 4/3
Example 116: V.D. of a gas is 16. Ratio is specific heat at const. P to specific heat at const. Vol. is 1.4, then its
atomic weight is-
(A) 8 (B) 16
(C) 24 (D) 32
Solution:(B) Molecular wt. = 2  V.D. = 2  16 = 32.
Gas is diatomic as ϒ = 1.4.
Therefore, atomic wt. is 16

Silver salt method: (for organic acids)

Basicity of an acid:No. of replaceable H+ atoms in an acid (H contained to more electronegative atom is


acidic)
Procedure: Some known amount of silver salt (w1 gm) is heated to obtain w2 gm of while shining
residue of silver. Then if the basicity of acid is n, molecular weight of acid would be and molecular
weight of acid = Msalt– n(107)
This is one good practical application of POAC.

Chloroplatinate salt method: (for organic bases)

Lewis acid: electron pair acceptor


Lewis base:electron pair donor
Procedure:Some amount of organic base is reacted with H2PtCl6 and forms salt known as
chloroplatinate. If base is denoted by B then salt formed
(i) with monoacidic base = B2H2PtCl6
(ii) with diacidic base = B2(H2PtCl6)2
(iii) with triacidic base = B2(H2PtCl6)3
The known amount (w1 gm) of salt is heated and Pt residue is measured. (w2 gm). If acidity of base is
1 W 
'n'then and Mbase =  1  195 − 410  Acidity of base
2 W2 

55
CHEMISTRY SOME BASIC CONCEPTS OF CHEMISTRY

1. Which of the following mixtures, is a composition of Iodised salt?


(A) NaCl and I2 (B) NaCl and KI
(C) NaCl and KI3 (D) NaI and KI
2. Which of the following substances is deliquescent?
(A) NaCl (pure) (B) CaCl2
(C) KBr (D) CuSO4
3. Which of the following substances exhibit polymorphism?
(A) ZnS (B) NaCl
(C) KBr (D) CuSO4
4. Which among the following is not a pure substance?
(A) Diamond (B) Oxygen
(C) Sodium chloride (D) Iodized table salt
5. When sulphuric acid is added carefully to water what is formed:
(A) Compound (B) element
(C) Homogenous mixture (D) Heterogeneous mixture
6. Presence of which of the following in the air makes it a heterogeneous mixture.
(A) Oxygen (B) Argon
(C) Nitrogen (D) Dust Particles
7. The smallest particle that exhibits all the properties of a substance is:
(A) Atom (B) Molecule
(C) Fermions (D) ion
8. A pure substance which contains only one type of atoms is called
(A) an element (B) a compound
(C) a solid (D) a liquid

9. The speed of 80 miles per hour expressed in ms–1 is


(A) 20 ms–1 (B) 135ms– 1
(C) 3.5  107ms–1 (D) 35.5ms–1
10. A length of 65 expressed in m is:
(A) 100 m (B) 1.9558 m
(C) 160312 m (D) 200.10 m
11. What temperature is −40 F on the kelvin scale?
o

(A) 233 K (B) 213 K


(C) 212 K (D) 200 K
12. Which of the following is the greatest length?
(A) 0.005 km (B) 180 inch
(C) 300 cm (D) 6feet
13. The units nanometer, Fermi, angstrom and attometre, arranged in decreasing order will read as –
(A) angstrom, nanometer, Fermi, attometre (B) fermi, attometre, angstrom, nanometer
(C) nanometer, angstrom, fermi, attometre (D) attometre, angstrom, fermi, nanometer

56
SOME BASIC CONCEPTS OF CHEMISTRY CHEMISTRY

14. The number of significant figures in Avogadro’s number (6.022 1023) are:
(A) 27 (B) 3
(C) 4 (D) 10
15. 81.4 g sample of ethyl alcohol contains 0.002g of water. The amount of pure ethyl alcohol to the proper
number of significant figures is:
(A) 81.4g (B) 71.40g
(C) 91.4g (D) 81g
16. Mass of Hydrogen atom is 1.008g. the value expressed in 2 significant figures is:
(A) 1.008  102 (B) 1.0
(C) 1.01 (D) 1
–1
17. Molecular mass of glucose is 180g mole . The value expressed in four significant figures is:
(A) 180.0 (B) 1.80 104
(C) 18.00  10 5
(D) 1.8  105
18. 12.35g of a substance has volume of 20.153 ml. The density of the substance in correct number of
significant digits is.
(A) 0.6128 g/ml (B) 6.1  10– 2 g/ml
(C) 0.61 g/ml (D) 0.0612 g/ml
19. Given the numbers: 161 cm, 0.161 cm; 0.0161 cm. Number of significant figures for three numbers is:
(A) 3, 4, 5 (B) 3, 3, 3
(C) 3, 3, 4 (D) 3, 4, 4
20. The significant figures in 34.12 are
(A) 2 (B) 5
(C) 6 (D) 4
21. The number of significant figures in 60.0001 are
(A) 5 (B) 6
(C) 3 (D) 2
22. Use the approximate rules to report the result of the following computation(43.221 – 1.6)/11.2
(A) 4 (B) 3.7
(C) 3.72 (D) 3.716
23. Answer to the following addition problem should be expressed as:
(10.11111 + 1.02 + 100.00001 +0.04000)
(A) 111.2 (B) 111.1711
(C) 111.17 (D) 111.171

24. What is the weight of sodium chloride decomposed by 9.8 g of sulphuric acid if 12g of sodium hydrogen
sulphate and 3.65g of hydrogen chloride are produced in the same reaction?
(A) 2.925g (B) 4.9g
(C) 7.6g (D) 5.85g
25. Which illustrates law of mass conservation?
(A) AgNO3 + NaCl → AgCl + NaNO3 (B) 4 Be9 + 2 He4 → 6 C12 + 0 n1
(C) Both (D) None
26. H2S contains 5.88% hydrogen, H2O contains 11.11% hydrogen while SO2 contains 50% sulphur. These
figures illustrate the law of:
(A) conservation of mass (B) constant proportions
(C) multiple proportions (D) reciprocal proportions

57
CHEMISTRY SOME BASIC CONCEPTS OF CHEMISTRY

27. Which does not illustrate law of multiple proportion


(A)SO2 + SO3 (B) NO2 + N2O
(C) MgO + Mg(OH)2 (D) NO + N2O5
28. There are two oxides of sulphur. They contain 50% and 60% of oxygen respectively by weight. The
weights of sulphur which combine with 1g of oxygen
(A)1 : 1 (B) 2 : 1
(C) 1 : 2 (D) 3 : 2
29. 2 elements X (at. Mass 16) & Y (at. Mass 14) combine to form compounds A, B and C. The ratio of
different masses of Y which combine with a fixes mass of X in A, B & C is 1:3:5. If 32 parts by mass of
X combines with 84 parts by mass of Y in B, then in C, 16 parts by mass of X will combine with:
(A) 14 parts by mass of Y (B) 42 parts by mass of Y
(C) 70 parts by mass of Y (D) 84 parts by mass of Y

30. Of two oxides of iron, the first contained 22% and the second contained 30% of oxygen by weight. The
ratio of weight of iron in the two oxides that combine with the same weight of oxygen, is
(A) 3 : 2 (B) 2 : 1
(C) 1 : 2 (D) 1 : 1
31. Law of multiple proportions is illustrated by one of the following pairs
(A) H2S and SO2 (B) NH3 and NO2
(C) Na2S and Na2O (D) N2O and NO
32. Which of the following set of compounds will illustrate law of reciprocal proportions?
(A)CuS, CuO, Cu2O (B) CuS, SO3, CuO
(C) HCl, P2O5, PCl5 (D) All
33. “Under similar conditions of temperature & pressure equal volumes of all gases contain equal number of
molecules” is
(A) Gay Lussac’s Law (B) Law of multiple proportions
(C) Avogadro’s hypothesis (D) Law of constant proportion
34. Two containers P and Q of equal volume (1 litre each) contain O2 and SO2 respectively at 300 K and 1
atmosphere. Then
(A) Number of molecules in P is less than that in Q (B) No. of molecules in Q is less than that in P
(C) Number of molecules in P and Q are same (D) Either (A) or (B)
35. Important postulate of Dalton’s atomic theory is:
(A) Atom contains electrons, protons and neutrons
(B) Atom can neither be created nor destroyed nor divisible
(C) All the atoms of an element are not identical
(D) All elements are available in nature in the formof atoms
36. Which of them is limitation of Dalton’s Atomic Theory?
(A) Atoms of the same element are identical in all respects
(B) An atom is the smallest particle that takes part in a chemical reaction
(C) Atoms of same or different elements combine together to form compound atoms.
(D) None of these.

37. The weight of a molecule of the compound C60H122 is


(A)1.4  10−21 g (B)1.09  10−21 g
(C)5.025  1023 g (D)16.023  1023 g

58
SOME BASIC CONCEPTS OF CHEMISTRY CHEMISTRY

38. 19.7 kg of gold was recovered from a smuggler. The atoms of gold recovered are: (Au = 197)
(A) 10 (B) 6.02 × 1023
(C) 6.02 × 1024 (D) 6.02 × 1025
39. Total no. of protons in 36 ml of water at 4°C (where  of water = 1 g/ml) is
(A) 20 (B) 16
(C) 20 NA (D) 16NA
40. In which of the following pairs both members have same no. of atoms
(A) 1 gm O2, 1 gm O3 (B) 1 gm N2, 2 gm N
(C) Both (D) None
41. Which of the following has greatest no. of oxygen atoms?
(A) 10 mole water (B) 200gm of C12H22O11
(C) 144 gm O3 (D) 2.5 mole H2SO3
42. Which of the following samples contains the smallest no. of atoms?
(A) 1 g of CO2 (g) (B) 1 g of C8H18 (l)
(C) 1 g of C2H6 (g) (D) 1 g of LiF (s)
43. Which of the following weighs the least?
(A) 0.2 g atom of N (at. wt. N=14) (B) 31023 atoms of C (at. wt. C = 12)
(C) 1 mole atoms of S (at. wt. S=32) (D) 7 g silver (at. wt. Ag=108)
44. Which one of the following statements is not correct?
(A) One gm atom of silver equals 108 gms
(B) One mole of CO2 and NH3 at NTP occupies same volume
(C) One mole Ag weighs more than one mole of Zn
(D) One gm molecule of CO2 is 44 times heavier than one molecule of CO2
45. The molecular wt. of green vitriol is M0. The wt. of 10−3 NA molecules of it is
(A) M0 gm (B) M0 mg
(C) 103 M0 gm (D) 10−3 M0 mg
46. Maximum no. of hydrogen atoms are in:
(A) 16g of CH4 (B) 31.2g of C6H4
(C) 34.2g of C12H22O11 (D) 36g of C6H12O6
47. Which of the following samples contains 2.0  10 atoms?
23

(A) 8.0 g O2 (B) 3.0 g Be


(C) 8.0 g C (D) 19.0 g F2
-7 3
48. Volume of a gas NTP is 1.12 × 10 cm . The number of molecules in it is:
(A) 3.01 × 1012 (B) 3.01 × 1024
23
(C) 3.01 × 10 (D) 3.01 × 1020
49. 3.0115 × 109 molecules of X weigh 1.0 × 10– 12 g. The molecular mass of X is.
(A) 250 amu (B) 200 amu
(C) 380 amu (D) 100 amu
50. A sample contains 200 atoms of hydrogen, 0.05 gm atom of nitrogen, 10−20 gm atom of oxygen. What is
the approximate no. of total atoms?
(A) 200 (B) 6223
22
(C) 310 (D) none of these
51. Compute the value of x in terms of numbers of moles and also in terms of mass gram.
X = 9.4 g of phenol (C6H5OH) + 6.02 × 1023 molecules of phenol – 0.2 g mole of phenol
(A) 0.9 mol, 84.6 g (B) 0.0978 mol, 9.2 g
(C) 0.1 mol, 9.4 g (D) 6.02 × 1023 molecules, 94 g

59
CHEMISTRY SOME BASIC CONCEPTS OF CHEMISTRY

52. If 1 ml of water contains 20 drops, then number of molecules in a drop of water is


(A) 6.023 × 1023 molecules (B) 1.376 × 1026molecules
21
(C) 1.672 × 10 molecules (D) 4.346 × 1020 molecules
53. Volume of a mixture of 6.02 × 1023 oxygen atoms and 3.01 × 1023 hydrogen molecules at NTP is :
(A) 28.0 litre (B) 33.6 litre
(C) 11.2 litre (D) 22.4 litre
54. At NTP the volume of 0.44 g of CO2 is the same as that of
(A) 0.02 of hydrogen gas (B) 0.085 g of NH3
(C) 320 mg of gaseous SO2 (D) each of the above
3+
55. The charge on 1 gram ion of Al is (e represents magnitude of charge on 1 electron)
(A) 1/27 NAe coulomb (B) 1/3 NAe coulomb
(C) 1/9 NAe coulomb (D) 3 NAe coulomb
56. Chlorophyll contains 2.68% of Mg by mass. Calculate the number of Mg atoms in 3 g of chlorophyll.
(A) 2.01  1021 atoms (B) 6.023  1023 atoms
(C) 1.7  1020 atoms (D) 2.8  1022 atoms

57. A mixture contains n moles of H2 and 2n moles of CH4. Ratio of no. of C:H atoms in the mixture is:
(A) 1/5 (B) 2/3
(C) 4/5 (D) 1/3
58. Which of the following substances contains the greatest mass of chlorine?
(A) 5 gm Cl2 (B) 60 gm NaClO3
(C) 0.10 mole of KCl (D) 0.5 mole of Cl2
59. The wt. of NO having the same no. of oxygen atom present in 9.2 gm of NO2 is
(A) 9.2 gm (B) 6 gm
(C) 12 gm (D) 24 gm
60. The no. of moles of carbon dioxide which contain 8g of oxygen is
(A) 0.50 mole (B) 0.20 mole
(C) 0.40 moles (D) 0.25 moles
61. A sample of ammonium phosphate, (NH4)3 PO4 contains 3.18 mol of hydrogen atoms. The number of
moles of oxygen atoms in the sample is:
(A) 0.265 (B) 0.795
(C) 1.06 (D) 3.18
62. An ore contains 1.34% of the mineral argentite, Ag2S, by weight. How many grams of this ore would
have to be processed in order to obtain 1.00 g of pure solid silver, Ag?
(A) 74.6 g (B) 85.7 g
(C) 134.0 g (D) 171.4 g
63. 8 g of sulphur is burnt to form SO2 which is oxidised by Cl2 water. The solution is treated with BaCl2
solution. The amount of BaSO4 precipitated is
(A) 1 mole (B) 0.5 mole
(C) 0.24 mole (D) 0.25 mole
64. A mixture of KBr and NaBr weighing 0.560 gm was treated with aqueous Ag+ and all the bromide ion
was recovered as 0.970 gm of pure AgBr. The weight of KBr in the sample is
(A) 0.25 gm (B) 0.212
(C) 0.36 (D) 0.285
65. One mole of potassium chlorate is thermally decomposed and excess of aluminium is burnt in the
gaseous product. How many mole of aluminium oxide are formed:
(A) 1 (B) 1.5
(C) 2 (D) 3

60
SOME BASIC CONCEPTS OF CHEMISTRY CHEMISTRY

66. An element, X, have three isotopes X 20, X 21 and X 22. The percentage abundance of X 20is 90% and its
average atomic mass of the element is 20.11. The percentage abundance of X 21 should be
(A) 9% (B) 8%
(C) 10% (D) 0%
18 16
67. O /O ratio in some meteorites is greater than that used to calculate average atomic mass of oxygen on
earth. The average mass of an atom of O in these meteorites is ……. that of a terrestrial O atom.
(A) equal to (B) greater than
(C) less than (D) None of these
68. Insulin contains 3.4% sulphur. What will be the minimum molecular weight of insulin?
(A) 94.176 (B) 1884
(C) 941.176 (D) 976
69. 2g mixture of Na2CO3 & NaHCO3 suffered a loss of 0.12g on heating. % of Na2CO3 in mixture is:
(A) 83.8 (B) 16.2
(C) 38.8 (D) 61.2
70. 5 moles of CH4is burned with 8 moles of O2 than calculate mole of CO2formed and remaining moles of
excess reagent
(A) 4, 1 (B) 1, 4
(C) 0, 5 (D) 5, 0

71. 10 moles
N 2 and 15 moles of H 2 were allowed to react over a suitable catalyst. 10 moles of NH3 were
N H
formed. The remaining moles of 2 and 2 respectively are:
(A) 5 moles, 0 mole (B) 0 moles, 5 mole
(C) 9 moles, 12 mole (D) 0 moles, 0 mole
72. Equal weights of X (atomic weight = 36) and Y (atomic weight = 24) are reacted completely to form the
compound X2Y3. Which of the following is/are correct?
I. X is the limiting reagent. II. Y is the limiting reagent. III. No reactant is left over.
IV. Number of moles of X2Y3 formed is half the number of moles of X taken.
(A) I, II (B) III, IV
(C) II, I (D) II, III

73. Cortisone is a molecular substance containing 21 atoms of carbon per molecule. The weight percentage
of carbon in cortisone is 69.98%. What is the molecular weight of cortisone?
(A) 176.5 (B) 252.2
(C) 287.6 (D) 360.1
74. A compound contains 28% nitrogen and 72% metal by mass 3 atoms of the metal combine with 2 atoms
of nitrogen. The atomic mass of metal is
(A) 36 (B) 20
(C) 24 (D) 36
75. An organic compound on analysis was found to contain 0.014% of nitrogen. If its molecule contains two
N atoms, then the molecular mass of the compound
(A) 200 (B) 2000
(C) 20,000 (D) 200000
76. The combustion of 4.24 mg of an organic compound produces 8.45 mg of CO2 and 3.46 mg of water. The
mass percentages of C and H in the compound respectively are.
(A) 54.4, 9.1 (B) 9.1, 54.4
(C) 27.2, 18.2 (D) 182.2, 27.2

61
CHEMISTRY SOME BASIC CONCEPTS OF CHEMISTRY

77. Simplest formulae of a compound containing 20% of element X (atomic weight 10) and 80% of element
Y (atomic weight 20) is
(A) XY (B) X2Y
(C) XY2 (D) X2Y3
78. The hydrated salt Na2SO410H2O undergoes X% loss in weight on heating and becomes anhydrous. The
value of X will be
(A) 10 (B) 45
(C) 56 (D) 70
79. An oxide of iodine (I =127) contains 25.4 g of iodine and 8 g of oxygen. Its formula could be
(A) I2O3 (B) I2O
(C) I2O5 (D) I2O7
80. A compound of magnesium contains 21.9% magnesium, 27.8% phosphorus and 50.3% oxygen. What
will be the simplest formula of the compound?
(A) Mg2P2O7 (B) MgPO3
(C) Mg2P2O2 (D) MgP2O4
81. The element A at wt.=75 and B at wt. =32 combine to form a compound X. If 3 mol of B combine with 2
mol of A to give 1 mol of X, the weight of 5 mol of X is
(A) 246 gm (B) 1230 amu
(C) 1.23 kg (D) None of these

82. A copper sulphate solution contains 1.595% of CuSO4 by weight. Its density is 1.2 g/ml, Its molarity will
be
(A) 0.12 (B) 0.06
(C) 1.20 (D) 1.595
83. 25.0 ml of HCl solution gave, on reaction with excess AgNO3 solution 2.125 g of AgCl. The molarity of
HCl solution is
(A) 0.25 (B) 0.6
(C) 1.0 (D) 0.75
84. The density of a 3.6 M sulphuric acid solution that is 29%H2SO4(M =98g mol–1) by mass will be:
(A) 1.22 (B) 1.45
(C) 1.64 (D) 1.88
85. Two samples of HCl, A and B having concentrations 0.5 M and 0.1 M respectively. The volumes of A
and B required to make 2L of 0.2 M HCl are:
(A) 0.5 L of A + 1.5 L of B (B) 1.5 L of A + 0.5 L of B
(C) 1.0 L of A + 1.0 L of B (D) 0.75 L of A + 1.25 L of B
86. According to the reaction, CaCO3 ( s ) + 2HCl ( aq ) → CaCl2 ( aq ) + CO2 ( g ) + H2O (l ) . The mass of
CaCO3 required to react completely with 25 mL of 0.75 M HCl is
(A) 0.1 g (B) 0.84 g
(C) 8.4 g (D) 0.94 g

87. A toothpaste has 0.2 gL–1 of fluoride. The concentration of the fluoride in ppm will be:
(A) 250 (B) 200
(C) 400 (D) 100

62
SOME BASIC CONCEPTS OF CHEMISTRY CHEMISTRY

88. What will be the molality of chloroform in the water sample which contains 15 ppm chloroform by mass?
(A) 1.25  10–4 m (B) 2.5  10–4 m
(C) 1.5  10–3 m (D) 1.25  10–5 m
89. Mole fraction of ethanol in an ethanol water mixture is 0.25. Hence the percentage concentration of
ethanol by weight of the mixture is
(A) 25% (B) 75%
(C) 46% (D) 54%
90. An aqueous solution of urea (NH2CONH2) is 3.0 molal. The mole fraction of urea is:
(A) 0.25 (B) 0.33
(C) 0.66 (D) 0.05
91. 0.25 molar aqueous solution of NaOH is found to have density of 1.25 g/cc. What is its molality and the
mole fraction of the solute?
(A) m = 0.3, xNaOH = 0.2 (B) m = 0.202, xNaOH = 0.00363
(C) m = 0.1, xNaOH = 0.011 (D) m = 0.05, xNaOH = 1.47

92. What is the basicity of the following oxo-acids of Phosphorous in the respective order:
I → H3PO4 (orthophosphoric acid) II → H3PO3 (orthophosphorous acid)
III → H3PO2 (Hypophosphorous acid)
(A) 3, 2, 1 (B) 1, 2, 3
(C) 3, 3, 3 (D) 2, 1, 3
93. If half mole of oxygen combine with Al to form Al2O3, the weight of Al used in the reaction is:
(A) 27 g (B) 40.5 g
(C) 54 g (D) 18 g
94. A metal oxide is reduced by heating it in a stream of hydrogen. It is found that after compete reduction
3.15 gm of the oxide have yielded 1.05 gm of the metal. We may conclude that -
(A) Atomic weight of the metal is 4 (B) Equivalent weight of the metal is 8
(C) Equivalent weight of the metal is 4 (D) Atomic weight of the metal is 8
95. 2 oxides of M contains 50% & 40% of M resp. Formula of 1st oxide is MO. Formula of 2nd oxide is-
(A) MO2 (B) M2O3
(C) M2O (D) M2O5
96. A hydrocarbon C10Hx requires 32.5 moles of O2for combustion of 2.5 moles. Calculate value of x?
(A) 24 (B) 32
(C) 12 (D) 22
97. Hydrogen evolved at NTP on complete reaction of 27 gm of Al with excess of aq. NaOH would be
(Chemical reaction: 2Al + 2NaOH + 2H2O ⎯→ 2NaAlO2 + 3H2)
(A) 22.4 lit (B) 44.8 lit
(C) 67.2 lit (D) 33.6 lit

98. What volume of 1M NaOH solution should be added to 12 g NaH2PO4 to convert it into Na3PO4? (at. wt
of P = 31)
(A) 100 ml (B) 200 ml
(C) 80 ml (D) 300 ml
99. 0.45g of acid (mol. wt = 90) was neutralized by 20 ml of 0.5 (M) NaOH. Basicity of given acid is-
(A) 1 (B) 2
(C) 3 (D) 4

63
CHEMISTRY SOME BASIC CONCEPTS OF CHEMISTRY

100. 0.84 g of a metal carbonate reacts with 40 mL of N/2 H2SO4. The equivalent mass of metal carbonate is:
(A) 84 g (B) 64 g
(C) 42 g (D) 38 g
101. 0.5 gm of a base was completely neutralised by 100 ml. of 0.2 N acid. Equivalent weight of base is -
(A) 50 (B) 100
(C) 25 (D) 125
102. 0.126 g of an acid requires 20 ml of 0.1 N NaOH for complete neutralisation. Eq. wt. of the acid is -
(A) 45 (B) 53
(C) 40 (D) 63

103. Aq. solution of 6.3g oxalic acid dihydrate is made upto 250 mL. Volume of 0.1M NaOH required to
completely neutralize 10 mL of this solution is:H2C2O4.2H2O + 2NaOH →Na2C2O4 + 4H2O
(A) 40 mL (B) 20 mL
(C) 10 mL (D) 4 mL
104. Equivalent weight of an element is 4. It’s chloride has a V.D. 59.25. Then valency of the element is -
(A) 4 (B) 3
(C) 2 (D) 1
105. Chloride of a metal contains 71% chlorine by weight and the vapour density of it is 50. Atomic weight of
the metal will be:
(A) 29 (B) 58
(C) 35.5 (D) 71
106. Specific heat of a solid element is 0.1 Cal/g °C and its equivalent weight is 31.8. It’s exact atomic weight is:
(A) 31.8 (B) 63.6
(C) 318 (D) 95.4
107. Carbonate of M is isomorphous with MgCO3 and contains 6.091% of C. Atomic weight of the metal is
nearly:
(A) 48 (B) 68.5
(C) 137 (D) 120

108. 1L sample of hard water contains 1.11 mg of CaCl2& 0.95 mg of MgCl2. Find hardness of water in terms
of PPM of CaCO3?
(A) 2 PPM (B) 1 PPM
(C) 3 PPM (D) None
109. Volume of free SO3 obtained from 100g of Oleum that is labelled with"109% H2SO4" at NTP is?
(A) 11.2 L (B) 22.4 L
(C) 33.6 L (D) None
110. Find volume Strength of 1.5N H2O2 soln.
(A) 8.4 (B) 5.6
(C) 11.2 (D) None
111. Calculate Vol. Strength of 17g/L H2O2 soln.
(A) 5.6 (B) 8.4
(C) 11.2 (D) None
112. A gaseous hydrocarbon requires 6 times its own volume of O2 for complete oxidation and produces 4
times its volume of CO2. What is its formula?
(A) C4H8 (B) C2H6
(C) C3H6 (D) None

64
SOME BASIC CONCEPTS OF CHEMISTRY CHEMISTRY

113. What is the SI prefix corresponding to the following multiples?


(a) 10– 6 (b) 106 (c) 109 (d) 10– 2
114. Which is a more precise measure of the volume of water in a measuring cylinder? 15.0 mL or 15.000 mL
115. Matter in its solid state has a definite shape and volume and is rigid. In light of the fact stated above,
account for the following observations:
(a) A sponge is a solid but can be compressed.
(b) Salt powder acquires the shape of the container; into which it is poured.
(c) A rubber band can be stretched.
116. What is the mass of phosphoric acid (H3PO4) obtained from 284 g of phosphorous pentaoxide(P2O5).
Solve by POAC.
117. A 10.0 g of sample of mixture of calcium chloride and sodium chloride is treated with Na 2CO3 to
precipitate the calcium as calcium carbonate. This CaCO3 is heated to convert all calcium to CaO and
final mass of CaO is 1.62 g. Calculate the percentage by mass of CaCl2in the original mixture.
118. A welding fuel gas contains carbon and hydrogen only. Burning a small sample of it in oxygen gives 3.38
g carbon dioxide, 0.690 g of water and no other products. A volume of 10.0 L (measured at STP) of this
welding gas is found to weigh 11.6 g. Calculate (i) empirical formula, (ii) molar mass of the gas, and (iii)
molecular formula.
119. A compound which contains one atom of X and two atoms of Y for each three atoms of Z is made by mixing
5.00 g of X, 1.15  1023 atoms of Y and 0.03 mole of Z atoms. Given that only 4.40g of compound results.
Calculate the atomic weight of Y if the atomic weight of X and Z are 60 and 80 a.m.u. respectively.
120. Calcium carbonate reacts with aqueous HCl to give CaCl2 and CO2 according to the reaction,
CaCO3 ( s ) + 2HCl ( aq ) → CaCl2 ( aq ) + CO2 ( g ) + H2O (l )
What mass of CaCO3 is required completely with 25 mL of 0.75 M HCl?
121. If the density of methanol is 0.793 kg L-1, what is its volume needed for making 2.5 L of its 0.25 M solution?
122. Chlorine is prepared in the lab by heating MnO2 with aqueous hydrochloric acid, according to the reaction:
4HCl (aq) + MnO2 (s) → 2H2O (l) + MnCl2 (aq) + Cl2 (g)
What vol. of 0.05 M HCl will react with 10 g MnO2? (Atomic mass of Mn = 55u, O = 16 u, H = 1 u,Cl = 35.5 u).
123. Calculate the molarity of a solution of ethanol in water in which the mole fraction of ethanol is 0.040.
124. A sample of drinking water was found to be severely contaminated with chloroform, CHCl 3 supposed to
be carcinogenic in nature. The level of contamination was 15 ppm (by mass).
(i) Express this in per cent by mass.
(ii) Determine the molality of chloroform in the water sample.

65
CHEMISTRY SOME BASIC CONCEPTS OF CHEMISTRY

66
SOME BASIC CONCEPTS OF CHEMISTRY CHEMISTRY

1. What is efflorescence?
2. What are the general properties, characteristic of all forms of matter?

3. Which of the following is a homogenous mixture?


(A) Steel (B) Butter
(C) Smoke (D) Mist
4. What of the following is a compound?
(A) Marble (B) toothpaste
(C) Brass (D) Honey
5. Which of the following statements about a compound is incorrect?
(A) A molecule of a compound has atoms of different elements.
(B) A compound can be separated into its constituent elements.
(C) The ratio of atoms of different elements in a compound is fixed.
(D) A compound retains the physical properties of its constituent elements
6. Molecules of which of the following gases will be homo-nuclear?
(A) Ammonia (B) Phosphine
(C) Oxygen (D) Carbon monoxide
0
7. A measured temperature on Fahrenheit scale is 200 F. What will be the reading on Celsius scale?
(A) 400 C (B) 940 C
(C) 93.30 C (D)300 C
8. How many substances among the following are pure substances?
Gold, Tin, diamond, Ice, distilled water, sand, milk, air, brass
(A) 3 (B) 4
(C) 5 (D) 6

9. Express the following in the SI units.


(i) 5feet 4 inches (the average height of an Indian female)
(ii) 0.74 Å (Bond length of the hydrogen molecule)
(iii) 140 pounds (the average weight of an Indian male)
(iv) 45oC (the peak summer temperature in Jaipur)
(v) 90 miles per hour (the typical speed of Rajdhani Express)
10. Convert the following into basic units:
(i) 28.7 pm (ii) 15.15s (iii) 25365mg.

67
CHEMISTRY SOME BASIC CONCEPTS OF CHEMISTRY

11. Match the following prefixes with their multiples:


Prefixes Multiples
(i) Micro 106
(ii) Deca 109
(iii) Mega 10-6
(iv) Giga 10-15
(v) Femto 10
12. A train covers 912 miles in 27 hours. Calculate the speed in m s– 1.

13. The highest temperature among the following is:


(A) 2030 F (B) 278 K
0
(C) 105 C (D) All are equal
14. Which of the following system of units is not based on units of mass, length and time alone
(A) SI (B) MKS
(C) FPS (D) CGS
15. The boiling point of a substance is 50oC or
(A) 90oF (B) 122oF
(C) 58oF (D) 212oF
o o
16. The correct value of room temperature (25 C) in F scale is
(A) 98.6oF (B) 32oF
o
(C) 77 F (D) 107oF
17. S.I. unit of temperature is
(A) degree Celsius (B) degree Fahrenheit
(C) Kelvin (D)None of these
18. Which of the following is the smallest quantity?
(A) 3924 mg (B) 3.924 10–5 g
(C) 3.924 103g (D) 3.92410– 6 kg
19. Accuracy & Precision depends upon:
(A) Skill of the operator (B) Device used
(C) On both (D) Can’t predict
20. The number 0.302040 has ______ significant figures.
(A) 3 (B) 4
(C) 6 (D)5
21. Which value is more precise:
(A) 5.0 (B) 5.00
(C) Both are same (D) Can’t say
22. A measured temperature on Fahrenheit scale is 2000 F. What will be the reading on Celsius scale?
(A) 400 C (B) 940 C
0
(C) 93.3 C (D)300 C
23. For the prefix Giga, by what factor is the base unit multiplied
(A) 106 (B) 10– 6
(C) 10 9
(D) 10– 9

68
SOME BASIC CONCEPTS OF CHEMISTRY CHEMISTRY

24. How many significant figures should be present in the answer of the following calculations?
0.02856  298.15  0.112
(i) (ii) 5  5.364 (iii) 0.0125 + 0.7865 + 0.0215
0.5785
25. Express the following in scientific notation:
(i) 0.0048 (ii) 234,000 (iii) 8008 (iv) 500.0 (v) 6.0012
26. Round up the following up to three significant figures:
(i) 34.216 (ii) 10.4107 (iii) 0.04597 (iv) 2808
27. Define diffusion as a property of matter. In which state of matter is it more significant?

28. The number 0.302040 has ______ significant figures.


(A) 3 (B) 4
(C) 6 (D) 5
29. Which value is more precise:
(A) 5.0 (B) 5.00
(C) Both are same (D) Can’t say
30. In the scientific notation of 0.00123451, the exponent of 10 will be
(A) 5 (B) –3
(C) –5 (D) –4
31. The number of significant figure is 10.3406 g in
(A) 2 (B) 3
(C) 1 (D) 6

32. If there are equal volume of oxygen gas and ozone gas, what is the ratio of the number of atoms of oxygen in
the two gases and the ratio of the number of molecules of oxygen and the number of molecules of ozone
33. Copper sulphate crystals contain 25.45 % Cu and 36.07 % H2O. If law of constant proportions is true,
then calculate the mass of copper required to obtain 40 g of crystalline copper sulphate.

34. After a chemical reaction, the total mass of reactants and products
(A) Is always increased (B) Is always decreased
(C) Is not changed (D) Is always less or more
35. The percentage of copper and oxygen in samples of CuO obtained by different methods were found to be
the same. This illustrates the law of
(A) Constant proportions (B) Conservation of mass
(C) Multiple proportions (D) Reciprocal proportions
36. Two samples of lead oxide were separately reduced to metallic lead by heating in a current of hydrogen. The
weight of lead from one oxide was half the weight of lead obtained from the other oxide. The data illustrates
(A) Law of reciprocal proportions (B) Law of constant proportions
(C)Law of multiple proportions (D) Law of equivalent proportions
37. Chemical equation is balanced according to the law of
(A) Multiple proportion (B) Reciprocal proportion
(C) Conservation of mass (D) Definite proportions

69
CHEMISTRY SOME BASIC CONCEPTS OF CHEMISTRY

38. Sample of calcium carbonate (CaCO3) has the following percentage composition: Ca = 40%; C = 12%;
O = 48%. If the law of constant proportions is true, then the weight of calcium in 4 g of a sample of
calcium carbonate obtained from another source will be
(A) 0.016 g (B) 0.16 g
(C) 1.6 g (D) 16 g
39. n g of substance X reaction with m g of substance Y to form p g of substance R and q g of substance S.
This reaction can be represented as, X + Y = R + S. The relation which can be established, in the
amounts of the reactants and the products will be
(A) n − m = p − q (B) n + m = p + q
(C) n = m (D) p = q
40. Which of the following is the best example of law of conservation of mass
(A) 12 g of carbon combines with 32 g of oxygen to form 44 g of CO2
(B) When 12 g of carbon is heated in a vacuum there is no change in mass
(C) A sample of air increases in volume when heated at constant pressure but its mass remains unaltered
(D) The weight of a piece of platinum is the same before and after heating in air
41. A sample of pure carbon dioxide, irrespective of its source contains 27.27% C and 72.73% O. The data
support
(A) Law of constant composition (B) Law of conservation of mass
(C) Law of reciprocal proportions (D) Law of multiple proportions

42. Two oxides of lead were separately reduced to metallic lead by heating in hydrogen atmosphere. Mass of
yellow oxide used = 7.19g. Loss of mass due to reduction = 0.5g. Mass of brown oxide used = 7.69g.
Loss of mass after reduction =1 gm Prove the law of multiple proportions
43. Dalton’s Atomic theory remained unchallenged for a century, but later in the light of the new
observations it was replaced by the modern atomic theory. How is modern atomic theory different from
Dalton’s atomic theory?
44. What is Avogadro’s Hypothesis and how is it different from Berzelius hypothesis?
45. How can we calculate the atomicity of an elementary gas using Avogadro’s hypothesis?

46. Which one of the following pairs of compounds illustrates the law of multiple proportion
(A) H2O, Na2O (B) MgO, Na2O
(C) Na2O, BaO (D) SnCl2, SnCl4
47. Two elements X and Y have atomic weights of 14 and 16. They form a series of compounds A, B, C, D and E
in which the same amount of element X, Y is present in the ratio 1 : 2 : 3 : 4 : 5. If the compound A has 28
parts by weights of X and 16 parts by weight of Y, then the compound of C will have 28 parts weight of X and
(A) 32 parts by weight of Y (B) 48 parts by weight of Y
(C)64 parts by weight of Y (D) 80 parts by weight of Y
48. Carbon and oxygen combine to form two oxides, carbon monoxide and carbon dioxide in which the ratio
of the weights of carbon and oxygen is respectively 12 : 16 and 12 : 32. These figures illustrate the
(A) Law of multiple proportions (B) Law of reciprocal proportions
(C) Law of conservation of mass (D) Law of constant proportions
49. The law of multiple proportions is illustrated by the two compounds
(A) Sodium chloride and sodium bromide (B) Ordinary water and heavy water
(C) Caustic soda and caustic potash (D) Sulphur dioxide and sulphur trioxide

70
SOME BASIC CONCEPTS OF CHEMISTRY CHEMISTRY

50. In compound A, 1.00 g nitrogen unites with 0.57 g oxygen. In compound B, 2.00 g nitrogen combines
with 2.24 g oxygen. In compound C, 3.00 g nitrogen combines with 5.11 oxygen. These results obey the
following law
(A) Law of constant proportion (B) Law of multiple proportion
(C) Law of reciprocal proportion (D) Dalton’s law of partial pressure
51. Which one of the following sets of compounds correctly illustrate the law of reciprocal proportions
(A) P2O3, PH3, H2O (B) P2O5, PH3, H2O
(C) N2O5, NH3, H2O (D) N2O, NH3, H2O
52. Law of multiple proportion was proposed by
(A) Lavoisier (B) Proust
(C) Dalton (D) Gay Lussac
53. Four one litre flasks are separately filled with the gases CO2, F2, NH3 and Heat same room temperature
and pressure. Ratio of total no. of molecules of these gases present in different flasks would be
(A) 1 : 1 : 1 : 1 (B) 1 : 2 : 2 : 3
(C) 3 : 2 : 4 : 1 (D) 2 : 1 : 3 : 2

54. Volume of 88g of CO2 under a similar set of conditions of temperature and pressure is V L. Find the volume of:
(a) 64g of O2 (b) 17g of NH3 (c) 64g of CH4
55. Calculate the volume of gas at NTP occupied by
(i) 14 g nitrogen (ii) 1.5 g moles of carbon dioxide (iii) 1021 molecules of oxygen
56. Which one of the following will have largest number of atoms?
(i) 1 g Au (s) (ii) 1 g Na (s) (iii) 1 g Li (s) (iv) 1 g of Cl2 (g)
57. Calculate the number of atoms in each of the following:
(i) 52 moles of Ar (ii) 52 u of He (iii) 52 g of He.
21
58. If 10 molecules are removed from 200 mg of CO2, then what is number of moles of CO2 left?
59. Calculate the mass per cent of different elements present in sodium sulphate Na2SO4?
60. What is the mass of ‘P’in 2.2 g of P4S3?

61. A polymer contains 4 atoms of S per molecule. What is its minimum molecular mass if it contains 32%
of Sulphur by mass? (Atomic weight of S = 32)
(A) 100 (B) 200
(C) 300 (D) 400
62. Number of atoms in 4.9 g of H2SO4 is
(A) 0.35 (B) 0.35 NA
(C) 7 (D) 7 NA
63. Calculate no. of molecules in 11200 ml of O2 at NTP
(A) NA (B) 0.5NA
(C) 1 (D) 0.5
– 2−
64. Calculate the total number of e in 32 g of O 2 ion
(A) 18 (B) 18NA
(C) 16 (D) 32NA
65. Which is heaviest?
(A) 2g atoms of oxygen (B) 2g atoms of nitrogen
(C) NA molecules of CO2 (D) NA molecules of NO2

71
CHEMISTRY SOME BASIC CONCEPTS OF CHEMISTRY

66. Gram molecular volume of oxygen at STP is


(A) 3200 cm3 (B) 5600 cm3
3
(C) 22400 cm (D) 11200 cm3
67. 1 L of oxygen gas at STP will weigh
(A) 1.43g (B) 2.24g
(C) 11.2g (D) 22.4g
68. The maximum number of molecules is present in :
(A) 15 L of H2 at STP (B) 5L of N2 at STP
(C) 0.5g of H2 gas (D) 10g of O2 gas
69. The total number of electrons in 72 mL of water (density 1 gmL– 1) is
(A) 4 NA (B) 40 × NA
(C) 2 NA (D) 30 × NA
– 23
70. One atom of an element ‘X’ weighs 6.664 × 10 g. The number of gram atoms in 40 kg of it is:
(A) 10 (B) 100
(C) 10000 (D) 1000
71. How many moles of magnesium phosphate, Mg3 ( PO4 )2 will contain 0.25 mole of O atoms?

(A) 0.02 (B) 3.125 10−2


(C) 1.25 10−2 (D) 25 10−2
72. Number of atoms present in 4.25 g of NH3 is: [AFMC 2010]
(A) 6.023 10 23
(B) 4  6.023 10 23

(C) 1.7 1024 (D) 4.25  6.023 1023


73. One atom of an element weighs 1.8  10–22 g. Its atomic mass is: [MANIPAL 2009]
(A) 29.9 (B) 154
(C) 108.36 (D) 18
74. A polymer contains 4 atoms of S per molecule. What is its minimum molecular mass if it contains 32%
of Sulphur by mass? (Atomic weight of S = 32)
(A) 100 (B) 200
(C) 300 (D) 400
2+
75. Total no. of electrons present in 48 g of Mg are:
(A) 24 NA (B) 2 NA
(C) 20 NA (D) None of these
76. Which has more number of molecules?
(A) 150 cc of H2 gas at STP (B) 50 cc of gas at STP
(C) 250 cc of N2 gas at NTP (D) All have same molecules
77. Which has more number of molecules?
(A) 16g of S2 (B) 16g of S4
(C) 16g of S8 (D) All have same number of molecules
78. At NTP, volume of 0.32g of O2 is same as that of
(A) 6.02  1023 molecules of O3 (B) 0.14 g of nitrogen gas
(C) 6.02  10 molecules of SO2
21
(D) 17g of NH3
79. Which of the following has smallest number of molecules?
(A) 11.2 L of O2 at NTP (B) 8.0 g of O2
(C) 0.1 mole of O2 (D) 2.24  104 ml of O2

72
SOME BASIC CONCEPTS OF CHEMISTRY CHEMISTRY

80. The ratio of masses of oxygen and nitrogen in a particular gaseous mixture is 1:4. The ratio of number of
their molecules is
(A) 3 : 16 (B) 1 : 4
(C) 7 : 32 (D) 1 : 8
81. If we take 2.2g of CO2, 6.02  10 atoms of nitrogen and 0.03 g atoms of sulphur, then the molar ratio of
21

C, N and S atoms will be


(A) 1 : 2 : 5 (B) 5 : 1 : 2
(C) 2 : 5 : 3 (D) 5 : 1 : 3
82. Calculate number of atoms present in 5.6 l of diatomic gas at NTP.
NA NA
(A) (B)
4 2
NA
(C) NA (D)
8

83. Calculate the mass of carbon tetrachloride which can be produced by the reaction of 10 g of carbon with
100 g of chlorine. Determine the mass of excess reagent left unreacted
84. In the reaction12 P + 3Q2→ 6 P2Q Which is the limiting reagent if
(i) 3 mole of P react with 2 mole of Q.
(ii) 300 atoms of P react with 100 molecules of Q

85. For a reaction A + 2B → C, the amount of C formed by starting the reaction with 5 moles of A and 8
moles of B is
(A) 5 moles (B) 8 moles
(C) 16 moles (D) 4 moles
86. When 22.4 L of H2(g) is mixed with 11.2 L of Cl2(g),each at S.T.P., the moles of HCl (g) formed is equal to
(A) 1 mol of HCl (g) (B) 2 mol of HCl (g)
(C) 0.5 mol of HCl (g) (D) 1.5 mol of HCl (g)
87. In the reaction, 4NH3 ( g ) + 5O2 ( g ) → 4NO ( g ) + 6H2O ( g ) , when 1 mole of ammonia and 1 mole of
O2 are made to react to completion.
(A) 1.0 mole of H2O is produced (B) 1.0 mole of NO will be produced
(C) All the oxygen will be consumed (D) All the ammonia will be consumed
88. If 40 g of CaCO3 is treated with 40 g of HCl. Which of the reactants will act as limiting reagent?
(A) CaCO3 (B) HCl
(C) Both are equal. (D) Cannot be calculated.
89. 10 g of hydrogen and 64 g of oxygen were filled in a steel vessel and exploded. Amount of water
produced in this reaction will be [CBSE AIPMT 2009]
(A) 3 mol (B) 4 mol
(C) 1 mol (D) 2 mol
90. How many moles of lead (II) chloride will be formed from a reaction between 6.5 g of PbO and 3.2 g of HCl?
[CBSE AIPMT 2008]
(A) 0.044 (B) 0.333
(C) 0.011 (D) 0.029

73
CHEMISTRY SOME BASIC CONCEPTS OF CHEMISTRY

91. Calculate the amount of carbon dioxide that could be produced when
(i) 1 mole of carbon is burnt in air
(ii) 1 mole carbon is burnt in 16 g of dioxygen
(iii) 2 moles of carbon are burnt in 16 g of dioxygen.
92. A 5.00 g sample of natural gas containing methane CH4 and ethylene, C2H4 was burned in excess of
oxygen, yielding 44/3 g CO2 and some H2O as product. What % of the sample was ethylene?
93. 100 ml of CH4 and C2H2 were exploded with excess of O2. After explosion and cooling, the mixture was
treated with KOH, where a reduction of 165 ml was observed. Therefore the composition of the mixture is?

94. How much volume of oxygen at STP in litres is required to burn 4g of methane gas completely?
(A) 11.2 (B) 5.6
(C) 2.8 (D) 8
95. What is the yield of NH3 when 50.0 g each of N2 and H2 react with each other?
(A) 17.0 g (B) 30.5 g
(C) 34.0 g (D) 60.7
96. Gaseous mixture contains equimolecular proportion of C2H2& C2H4. Weight of 4.48 l of this mixture at NTP is
(A) 2.3 g (B) 23 g
(C) 5.4 g (D) 2.7 g
97. Air contains nearly 20% oxygen by volume. The volume of air needed for complete combustion of 100
5
ml of acetylene will be: C2 H2 ( g ) + O2 ( g ) → 2CO2 ( g ) + H 2O ( l )
2
(A) 500 ml (B) 100 ml
(C) 250 ml (D) 1250 ml
98. 0.75 mol of A4 reacts with 2 moles of O2 to give 1 mole of AXOY. Find x&y?
(A) x = 3, y = 4 (B) x = 4, y = 3
(C) x = 4, y = 4 (D) None
99. Polyethylene can be produced from calcium carbide according to the following sequence of reactions
CaC2 + H2O → CaO + HC  CH n ( CH  CH ) + nH2 → − ( CH2 − CH2 )n−
The mass of polyethylene which can be produced from 20.0 kg pure CaC2 is:
(A) 6.75 kg (B) 7.75 kg
(C) 8.75 kg (D) 9.75 kg
100. How much oxygen can be obtained by the thermal decomposition of 30.625 g of KClO3
(A) 16 g (B) 32 g
(C) 4 g (D) 8 g
101. In a reaction, 10g of MnO2, on heating completely gets converted to Mn3O4. The weight of Mn3O4
produced is [Given: Atomic weight of Mn = 55]
(A) 12.77 g (B) 8.77 g
(C) 9.77 g (D) 15.01 g

102. Determine the empirical formula of an iron which has 69.9% iron and 30.1 % dioxygen by mass?
103. The action of bacteria on meat and fish produces a poisonous compound called cadaverine. It has 58.77% of
C, 13.81% H and 27.42% of N. Its molar mass is 102 g/mol. Determine the molecular formula of cadaverine

74
SOME BASIC CONCEPTS OF CHEMISTRY CHEMISTRY

104. A gaseous hydrocarbon contains 85.7 % carbon and 14.3 % hydrogen. 1 litre of hydrocarbon weighs 1.26
g at NTP. Determine the molecular formula of the hydrocarbon.
105. Calculate the mass of carbon tetrachloride which can be produced by the reaction of 10 g of carbon with
100 g of chlorine. Determine the mass of excess reagent left unreacted

106. What is the empirical formula of a hydrocarbon if on complete combustion it gives 4.4 g of CO2 and 1.8 g
of H2O
(A) CH2 (B) CH
(C) CH4 (D) C2H2
107. A compound contains 69.5% oxygen and 30.5% nitrogen and its molecular weight is 92. The formula of
the compound is
(A) N2O (B) NO2
(C) N2O4 (D) N2O5
108. An element A is tetravalent and another element is divalent. The formula of the compound formed from
these elements will be
(A) A2B (B) AB
(C) AB2 (D) A2B3
109. An element A (at wt =75) and B (at wt = 25) combine to form a compound. The compound contains 75%
of A by weight. The formula of the compound will be
(A) A2B (B) A3B
(C) AB3 (D) AB
110. Simplest formula of a compound containing 50% of an element X (atomic wt. 10) & 50% Y (atomic
weight 20) is
(A) XY (B) X2Y
(C) XY2 (D) X2Y3
111. The empirical formula of the compound of carbon with hydrogen is CH2. 1L of this gaseous compound has
mass equal to that of 1 L N2 at STP. Thus, the molecular formula of the compound is:
(A) C3H6 (B) C2H4
(C) C4H8 (D) C5H10
112. The empirical formula of a compound is CH2O and its molecular mass is 180 g mol–1. What is the
molecular formula of the compound?
(A) C6H12O6 (B) C5H10O5
(C) C2H4O2 (D) C3H6O3
113. An organic compound on analysis gave C = 54.2%, H = 9.2% by mass. Its empirical formula is
(A) CHO2 (B) CH2O
(C) C2H8O (D) C2H4O
114. The empirical formula of compound is CH2O2, probable molecular formula of acid may be
(A) CH2O (B) CH2O2
(C) C2H4O2 (D) C3H6O4

115. How are 0.2 mole of CaCO3 different from 0.2 M CaCO3?
116. 350 mL of 0.25M HCl was added drop wise to sufficient amount of water to make the final volume 1.5
L. Find the molarity of the final sample.
117. Calculate the mass of sodium acetate (CH3COONa) required to make 500 mL of 0.375 molar aqueous
solutions. Molar mass of sodium acetate is 82.0248 g mol-1.

75
CHEMISTRY SOME BASIC CONCEPTS OF CHEMISTRY

118. How many moles and how many grams of sodium chloride are present in 500 cm3 of 0.25 M NaCl solution?
(A) 0.25 moles, 14.6g (B) 0.125 moles, 7.312g
(C) 1 mole, 58.5 g (D) 0.5 moles, 29.25 g
119. The final molarity of a solution by mixing 50 mL of 0.4 M HCl, 150 mL of 0.2 M HCl is:
(A) 0.5 M (B) 1 M
(C) 0.75 M (D) 0.25 M
120. If 250 mL of a 4 M solution is diluted to 1000 mL, what will be the molarity of the solution obtained?
(A) 1 M (B) 0.5 M
(C) 0.25 M (D) 0.01 M
121. 2.82 g of glucose (C6H12O6) is dissolved in 30 g of water. The mole fraction of glucose in the solution is
(A) 0.01 (B) 0.99
(C) 0.52 (D) 1.66
122. If 20 ml of a 0.4 N NaOH solution completely neutralize 40 ml of a dibasic acid, molarity of the acid
solution is:
(A) 0.1 M (B) 0.2 M
(C) 0.3 M (D) 0.4 M
123. What volume of water needs to be added to dilute 500 ml of 0.1 M solution into 0.08 M?
(A) 125 ml (B) 625 ml
(C) 50 ml (D) 250 ml
124. Mole fraction of urea in a solution with water is 0.2. What is molality of solution?
(A) 12.1 (B) 6.2
(C) 8.9 (D) 13.89
125. What mole fraction of glucose in 18% aqueous solution?
(A) 0.02 (B) 0.04
(C) 0.06 (D) 0.08
126. What volume of 0.2 M H2SO4 is equivalent to 50 mL of 0.4 M H2SO4?
(A) 50 mL (B) 100 mL
(C) 25 mL (D) 200 mL
–1
127. If the concentration of glucose (C6H12O6) in blood is 0.9 gL , what will be the molarity of glucose in blood?
(A) 5 M (B) 50 M
(C) 0.005 M (D) 0.5 M
128. In a binary solution, mole fraction of a solute is 0.4. If the no. of moles of solute is 0.2, no. of moles of solvent is
(A) 0.3 (B) 0.6
(C) 0.5 (D) 0.4
129. 25 mL of 3.0 M HCl are mixed with 75 mL of 4.0 M HCl. If the volumes are additive, the molarity of the
final mixture will be:
(A) 4.0 M (B) 3.75 M
(C) 4.25 M (D) 3.50 M
130. 0.2 M H2SO4 is diluted to 1000 times of its initial volume. Find the final molarity of the acid.
(A) 2  10–4 (B) 2  10–3
(C) 4  10–4 (D) 2  10–2
131. The mole fraction of solute in a 2.0 molal aqueous solution is:
(A) 1.77 (B) 1.87
(C) 0.347 (D) 0.0347

76
SOME BASIC CONCEPTS OF CHEMISTRY CHEMISTRY

132. A sample of HNO3 is 69% by mass and its density is 1.41 gL–1. Calculate the molarity and the molality of the sample.
133. What volume of 6 M HCl and 2 M HCl should be mixed to get 2 L of 3 M HCl?
134. The density of 3 molal solution of NaOH is 1.110 g mL–1. Calculate the molarity of the solution.

135. Which mode of concentration does not change with temperature?


(A) Molarity (B) Normality
(C) Molality (D) All of these.
136. Total weight of 0.2 m solution of H2SO4 in 1kg H2O:
(A) 1000 g (B) 1098.6 g
(C) 1019.6 g (D) 980.4 g
137. If 100 g of solution contains 60.25 g of solute, then what is the concentration of the solution in ppm?
(A) 602500 (B) 6.25
(C) 6.025 (D) 6025
138. Assuming 100% ionization, the solution having highest normality is
(A) 1 M H2SO4 (B) 1 M H3PO3
(C) 1 M H3PO4 (D) 1 M HNO3
139. Which does not change on dilution?
(A) Molarity of solution (B) Molality of solution
(C) Milli-moles and milli-equivalent of solute (D) Mole fraction of solute
140. What is molarity of 1 kg pure water?
(A) 1 (B) 5.55 M
(C) 55.5 M (D) 10 M
141. 0.2 moles of NaOH were dissolved in 92g H2O. What is % w/w of the solution?
(A) 9.2 % (B) 0.22 %
(C) 8 % (D) 20 %
142. 200 ml of 0.2M HCl reacted with 0.5M NaOH. What is the volume of NaOH required for complete
neutralization?
(A) 100 ml (B) 150 ml
(C) 200 ml (D) 80 ml
143. What is volume occupied by 1g eq. of O2 at NTP?
(A) 22.4 l (B) 11.2l
(C) 5.6l (D) 2.25l
144. If20 mL of a 0.4 N NaOH solution completely neutralize 40 mL of a dibasic acid, the molarity of the acid
solution is:
(A) 0.1 M (B) 0.2 M
(C) 0.3 M (D) 0.4 M
145. What volume of water is required to make 0.20 N solution of an acid from 250 mL of its 0.35 N solution.
(A) 187.5 mL (B) 100 mL
(C) 200 mL (D) 500 mL

77
CHEMISTRY SOME BASIC CONCEPTS OF CHEMISTRY

146. A 2N solution of calcium carbonate X ‘g’ of the salt dissolved in 500 mL of solution. The value of X is:
(A) 50 g (B) 200 g
(C) 250 g (D) 25 g
147. CaCO3 → CaO + CO2 . 10 g CaCO3 decomposes to give 2 l of CO2 at NTP. What is % purity of CaCO3?
(A) 10% (B) 89%
(C) 49% (D) 99%
148. 0.7 g of Na2CO3.xH2O were dissolved in water and the volume was made to 100 ml, 20 ml of this
solution required 19.8 ml of N/10 HCl for complete neutralization. The value of x is:
(A) 7 (B) 3
(C) 2 (D) 5
149. 98% solution of H2SO4 has density 1.75 g/cc. Calculate its molarity.
(A) 98 M (B) 19.8 M
(C) 9.8 M (D) 17.5 M
150. What is the % (by mass) composition of a solution containing 1000 g of sugar in 2000 g of water?
(A) 50% (B) 71%
(C) 25% (D) 33.33%
151. 1.25 g of a solid dibasic acid is completely neutralized by 25 ml of 0.25 M Ba(OH)2 sol. Molecular mass
of the acid is:
(A) 100 (B) 150
(C) 120 (D) 200

152. 1.091g of SnO2 was completely reduced when 0.86g of metallic tin was obtained. What is the E wt. of Sn?
153. 1g of chalk (CaCO3) is dissolved in 230ml of 0.1N HCl. Soln. requires 40ml of 0.1N NaOH to neutralize.
% purity is
154. When an element forms an oxide in which O is 20% of oxide by mass, equivalent mass of the element will be?

155. The equivalent weight of iron in Fe2O3 would be: [Atomic weight of Fe = 56]
(A) 18.6 (B) 26.66
(C) 56 (D) 112
156. A metal oxide has 40% oxygen. The equivalent weight of the metal is
(A) 12 (B) 16
(C) 24 (D) 48
157. Equivalent weight of Al2(SO4)3
M M
(A) (B)
2 6
M M
(C) (D)
4 8

78
SOME BASIC CONCEPTS OF CHEMISTRY CHEMISTRY

158. The equivalent wt. of a metal is double then of oxygen. How many times is the equivalent weight of its
oxide then the equivalent weight of the metal?
(A) 1.5 (B) 2
(C) 3 (D) 4
159. 0.71g of chlorine combines with certain weight of a metal giving 1.11 g of its chloride. The eq. wt. of the
metal is:
(A) 40 (B) 20
(C) 80 (D) None of these
160. Which of the following formulae expresses the law of equivalent correctly?
m1 E2
(A) = (B) E1E2 = m1m2
m2 E1

(C) m1E2 = E1m2 (D) ( m1 + m2 ) = ( E1 + E2 )

161. The normality of a 1 M solution of H3PO4 is:


(A) 0.5 N (B) 1.0 N
(C) 3.0 N (D) 1.5 N
162. 2 g of a base whose eq. wt. is 40 reacts with 3 g of an acid. Eq. wt. of acid is -
(A) 40 (B) 60
(C) 10 (D) 80

79
CHEMISTRY SOME BASIC CONCEPTS OF CHEMISTRY

1
 6.02 1023
197
1
 6.02 1023
23
1
 6.02 1023
7
2
 6.02 1023
71
3.1311025

6.023  1023  13

7.4  10−11 m



2.5365  10−2 Kg.

80
SOME BASIC CONCEPTS OF CHEMISTRY CHEMISTRY

4.8 10−3
2.34 105
8.008 103
5.000 102
6.0012 100

V
2
86

81
CHEMISTRY SOME BASIC CONCEPTS OF CHEMISTRY

1. Which of the following is not an allotrope of C(s):


(A) Diamond (B) Graphite
(C) Fullerene (D) Carbon dioxide
2. A mixture of iron filings & S was heated in a china dish and a black solid is obtained. Which is not true?
(A) A chemical change has taken place (B) On adding HCl, rotten eggsmell gas evolve
(C) Iron can be separated out using a magnet (D) Mixture is insoluble in carbon disulphide
3. On dividing 0.46 by 15.374, the actual answer is 0.029236. The correctly reported answer will be
(A) 0.02 (B) 0.029
(C) 0.0292 (D) 0.02924
4. The number of significant figure in  are:
(A) three (B) infinite
(C) zero (D) one
5. Which of the following is used to measure the volume of a standard solution?
(A)burette (B) pipette
(C) graduated cylinder (D) volumetric flask
6. 3ft 2 inch into cm makes
(A)91.44 cm (B) 96.52 cm
(C) 84.32 cm (D) 70.52 cm
7. The density of an element is 6.96 g cm–3. Convert to density in SI units
(A)6.96 (B) 69.600
(C) 6.96  102 (D) 6960
8. The value of 96 miles per hour in meters per minute will be
(A)42.7 (B) 2566
(C) 256.6 (D) 3.59
9. The value of 50 pm in the SI base unit will be
(A)50  1012 m (B) 5  10–13 m
(C) 50  10–12 m (D) 5  1013 m
10. What is the capacity of a box in SI units if it is 0.08m long, 1 cm wide and 400 mm deep?
(A) 3.2  10–3m3 (B) 3.2  10–5m3
(C) 3.2  10 m
–4 3
(D) 3.2  10–6m3
11. The body temperature of a normal healthy person is 98.4oF.The temperature on the Kelvin scale is
(A) 412K (B) 309.89 K
(C) 212 K (D) 315 K
12. Given P = 0.0030m, Q = 2.40m R = 3000m significant figures in P, Q, R are respectively.
(A) 2, 2, 1 (B) 2, 3, 4
(C) 4, 2, 1 (D) 4, 2, 3
13. Expressing 88 g of CO2 is terms of numbers of moles with three significant figures is
(A) 4.04 (B) 2.00
(C) 2.02 (D) 2.11
14. The atomic weight of silicon is 28.0855. Round this to 3 significant figures.
(A) 28.09 (B) 28.0
(C) 28.08 (D) 28.1

82
SOME BASIC CONCEPTS OF CHEMISTRY CHEMISTRY

15. 67.892 is rounded off to four significant figures as:


(A) 6.7892  10 (B) 67.89
(C) 6.788 (D) 6.700
16. Which of the following is a characteristic property of both mixtures and compounds?
(A) Their properties are same as those of their components
(B) Energy is released when they are formed
(C) Their masses are equal to sum of masses of their component
(D) They contain components in fixed proportion
17. The existence of different compound with similar chemical composition in the same crystalline forms is:
(A) Isomorphism (B) Polymorphism
(C) Allotropy (D) Isotropy
18. The correctly reported answer of the addition of 4.523, 2.3 and 6.24 will have significant figures
(A) Two (B) three
(C) Four (D) Five
19. The ability of a given substance to assume two or more crystalline structure is called
(A) amorphism (B) isomorphism
(C) polymorphism (D) isomerism
20. The significant figures in 1.0024, 1.240 and 0.0020 are respectively
(A) 5, 4, 2 (B) 3, 3, 1
(C) 5, 3, 4 (D) 5, 3, 2
21. Which of the following contains largest mass of hydrogen atoms?
(A) 5 moles C2H2O4 (B) 1.1 moles C3H8O3
(C) 1.5 moles C6H8O6 (D) 4 moles C2H4O2
22. What volume of 75% alcohol be weight (d = 0.8 g / cm3) must be used to prepare 150 cm3 of 30%
alcohol be weight (d = 0.9 g/ cm3)
(A) 67.5 ml (B) 56.25 ml
(C) 44.44 ml (D) None of these
23. What are the number of atoms if oxygen in 16 g of O3?
(A) 3 (B) 3NA
(C) 1 (D) NA
24. Calculate number of He atoms in 56u of He
(A) 14 (B) 28
(C) 14 NA (D) 28NA
25. Which is the chosen standard reference for comparing atomic masses on the unified scale?
(A) O-16 (B) C-12
(C) H-1 (D) All of these
2
26. The number of neutron in 5g of D2O (D is 1𝐻) are:
(A) 0.25 NA (B) 2.5 NA
(C) 1.1 NA (D) None of these
1
27. The quantity of matter which weighs equal to th of the mass of an atom of C-12 is?
12
(A) gram atomic mass (B) gram molecular mass
(C) 1u (D) None
28. 1 M HCl and 2 M HCl are mixed in volume ratio of 4:1. What is the final molarity of HCL solution?
(A) 1.5 (B) 1
(C) 1.2 (D) 1.8

83
CHEMISTRY SOME BASIC CONCEPTS OF CHEMISTRY

29. Al2(SO4)3 solution of 1 molal concentration is present in 1 l solution of 2.684 g/cc. How many moles of
BaSO4 would be precipitated on adding BaCl2 in excess?
(A) 2 moles (B) 3 moles
(C) 6 moles (D) 12 moles
30. Lavoisier gave
(A) Law of conservation of mass (B) Law of constant composition
(C) Law of reciprocal proportions (D) none
31. The law of constant proportions was enunciated by:
(A) Dalton (B) Berthelot
(C) Avogadro (D) Proust
32. Law of reciprocal proportions was put forward by
(A) Dalton (B) Richter
(C) Lavoisier (D)Proust
33. Determine the empirical formula of Kevlar, used in making bullet proof wests is 70.6% C, 4.2 % H, 11.8 % N
and 13.4 % O.
(A) C7H5NO2 (B) C7H5N2O
(C) C7H9NO (D) C7H5NO
34. A sample of sodium has a mass of 46 g. What is the main of the same number of calcium atom as sodium
atoms present in given sample?
(A) 46 g (B) 20 g
(C) 40 g (D) 80 g
35. The percentage of copper in 187.5 g of copper nitrate Cu(NO3)2is: [Aw of Cu = 63.5]
(A) 42.3% (B) 33.8%
(C) 63.5% (D) 50.12%
36. Number of moles in 1.8 g of H2O is equal to number of moles in :
I : 18 g of glucose (C6H12O6) ; II : 6 g of urea ; III : 34.2 g of sucrose (C12 H22 O11)
(A) I, II, III (B) I, II
(C) I, III (D) II, III
37. If 150 g of phosphoric acid is reacted with 100 g of Magnesium hydroxide, then how much of H3PO4 will
remain un-reacted? (The molecular weights of H3PO4 = 98 and Mg (OH)2 = 58.3)
(A) 112 g (B) 38 g
(C) 66.7 g (D) 16 g
38. What weight of SO2 can be made by burning Sulphur in 5.0 moles of oxygen
(A) 640 g (B) 160 g
(C) 80 g (D) 320 g
39. For the reaction Fe2O3 + 3CO → 2Fe + 3CO2 , the volume of carbon monoxide required to reduce one
mole of ferric oxide is
(A) 67.2 dm3 (B) 1.12 dm3
(C) 4.48 dm3 (D) 0.56dm3
40. 2NA number of atoms of A, combine with 4NA atoms of B. What is the empirical formula
(A) A2B4 (B) AB2
(C) AB (D) A2B2
41. Disilane Si2Hx is analyzed and found to contain 90.28 % by weight silicon. Value of x is (Si = 29)
(A) 2 (B) 3
(C) 4 (D) 6

84
SOME BASIC CONCEPTS OF CHEMISTRY CHEMISTRY

42. Certain compound has molecular formula X4O6 . If 10g of compound contains 5.62 g of X, atomic mass
of X is:
(A) 62.0 amu (B) 48.0 amu
(C) 32.0 amu (D) 30.8 amu
43. 637.2 g of NH3are allowed to react with 1142 g of CO2, by the following reaction:
2NH3 (g) + CO2 (g) → NH2CONH2 (aq) + H2O (l)What is true about the process?
(i) NH3 is the limiting reagent & CO2 is in excess. (ii)NH3 is in excess & CO2 is limiting reactant.
(iii) 1124 g of urea is formed. (iv) 319 g of CO2 is left un-reacted.
(A) (ii) and (iii) (B) (i) and (iii)
(C) (i) and (ii) (D) (iii) and (iv)
44. An impure sample of silver (1.5 g) is heated with S to form 0.124 g of Ag2S. What was the percent yield
of Ag2S?
(A) 21.6% (B) 7.2%
(C) 1.7% (D) 24.8%
45. If 0.5 mole of BaCl2 is mixed with 0.2 mole of Na3PO4 ; maximum number of Ba3(PO4)2 that can be
formed is:
(A) 0.70 (B) 0.50
(C) 0.20 (D) 0.10
46. What volume of oxygen gas (O2) measured at 0C and 1 atm, is needed to burn completely 1 L of
propane gas (C3H8)measured under the same conditions?
(A) 7 L (B) 6 L
(C) 5 L (D) 10 L
47. When the same amount of zinc is treated separately with excess of sulphuric acid and excess of NaOH,
the ratio of the volume of hydrogen evolved is:
Zn (X g) + excess H2SO4→ ZnSO4 + H2
Zn (X g) + 2NaOH (excess) → Na2ZnO2 + H2
(A) 1 : 1 (B) 1 : 2
(C) 2 : 1 (D) 9 : 4
48. Molarity equation of a mixture of solution of same substance is given by
(A) M1 + V1  M 2 + V2  M 3 + V3 = M1 + M 2 + M 3 (B) M1V1 + M 2V2 + M 3V3 = M (V1 + V2 + V3 )
M1 M 2 M 3 1 1 1 M1 M 2 M 3 1 1 1
(C) + + =M  + +  (D)   = M1    
V1 V2 V3  V1 V2 V3  V1 V2 V3  V1 V2 V3 
49. 320 mg of a sample of magnesium having a coating of its oxide required 20 ml of 0.1 M HCl for the
complete neutralization of the latter. The composition of the sample is:
(A) 87.5 % Mg & 12.5 % MgO (B) 12.5 % Mg & 87.5 % MgO
(C) 80 % Mg & 20 % MgO (D) 20 % Mg & 80 % MgO
50. 25 ml of a solution of barium hydroxide on titration with 0.1 molar solution of hydrochloric acid needed
35 ml of HCl. The molarity of barium hydroxide is:
(A) 0.28 (B) 0.35
(C) 0.07 (D) 0.14
51. If a metal chloride is isomorphous with MgCl2.8H2O then the valency of the metal is:
(A) 2 (B) 1
(C) 3 (D) None
52. 10 % w/w = ____ ppm.
(A) 10–3 (B) 105
–5
(C) 10 (D) 106

85
CHEMISTRY SOME BASIC CONCEPTS OF CHEMISTRY

53. A reacts with O to form A2O3. If 0.359 g of A reacts with 0.559 g of the compound, atomic weight of A
will be
(A) 51 (B) 43.08
(C) 49.7 (D) 47.9
54. A molal solution is one that contains 1 mol of solute in
(A) 1000 g of solvent (B) 1 L of solvent
(C) 1 L of solution (D) 22.4 L of solution
55. 16.8 L gas containing H2 and O2 is formed at NTP on electrolysis of water. What should be the weight of
the electrolysed water?
(A) 5 g (B) 9 g
(C) 10 g (D) 12 g
56. The volume of 0.05 M H3PO4 solution required to neutralize 100 mL of 0.5 M Ba(OH)2 solution is:
(A) 750 mL (B) 666.6 mL
(C) 199.5 mL (D) 117.15 mL
57. The amount of oxalic acid (hydrated) required to prepare 500 mL of its 0.1 N solution is:
(A) 0.315 g (B) 6.3 g
(C) 3.15 g (D) 63.0 g
58. 0.16 g of a dibasic and required 25 mL of deci-normal NaOH for complete neutralization. The molecular
weight of the acid is :
(A) 32 (B) 64
(C) 128 (D) 256
59. In victor Mayer’s method 0.2g volatile compound on volatilization gave 56ml of vapour at STP. Its
molecular wt. is-
(A) 40 (B) 60
(C) 80 (D) 120

86
SOME BASIC CONCEPTS OF CHEMISTRY CHEMISTRY

1. 4.5 g of compound A(MW = 90) was used to make 250 mL of its aqueous solution. The molarity of the
solution in M is x  10–1. The value of x is__________. [JEE (M) 2021]
2. Complete combustion of 1.80 g of an oxygen containing compound (CxHyOz) gave 2.64 g of CO2 and
1.08 g of H2O. The percentage of oxygen in the organic compound is: [JEE (M) 2021]
(A) 63.53 (B) 53.33
(C) 51.63 (D) 50.33
3. The number of significant figures in 50000.020  10–3 is [JEE (M) 2021]
4. The NaNO3 weighed out to make 50 mL of an aqueous solution containing 70.0mgNa+ per mL is g.
[Given: Atomic weight in g mol–1. Na : 23 ; N : 14 ; O : 16] [JEE (M) 2021]
5. The ratio of the mass percentages of 'C' & 'H' and 'C'& 'O' of a saturated acyclic organic compound °X'
are 4: 1 and 3: 4 respectively. Then, the moles of oxygen gas required for complete combustion of two
moles of organic compound ‘X’ is _____________. [JEE (M) 2020]
6. The mole fraction of glucose (C6H12O6) in an aqueous binary solution is 0.1. The mass percentage of
water in it, to the nearest integer, is [JEE (M) 2020]
7. 6.023  10 molecules are present in 10g of a substance x  . The molarity of a solution containing 5g of
22

substance 'x' in 2L solution is 10–3 [JEE (M) 2020]


8. The mass of ammonia in grams produced when 2.8kg of dinitrogen quantitatively reacts with 1 kg of
dihydrogen is [JEE (M) 2020]
9. A 100 mL solution was made by adding 1.43g of Na2CO3.  H2O. The normality of the solution is 0.1N.
The value of x is (The atomic mass of Na is 23g /mol) [JEE (M) 2020]
10. The minimum number of moles of O2 required for complete combustion of 1 mole of propane and 2
moles of butane is [JEE (M) 2020]
–1 35 37
11. The average molar mass of chlorine is 35.5gmol . The ratio Cl of Cl in naturally occurring chlorine is
close to [JEE (M) 2020]
(A) 01 : 01 (B) 02 : 01
(C) 03 : 01 (D) 04 : 01
12. Ferrous sulphate heptahydrate is used to fortify foods with iron. The amount (in grams) of the salt
required to achieve 10 ppm of iron in 100 kg of wheat is.
Atomic weight : Fe = 55.85; S = 32.00; O = 16.00 [JEE (M) 2020]
13. NaClO3 is used, even in spacecrafts, to produces O2. The daily consumption of pure O2 by a person is 492
L at 1 atm, 300 K. How much amount of NaClO3, in grams, is required to produce O2 for the daily
consumption of a person at 1 atm, 300 K ? [JEE (M) 2020]

87
CHEMISTRY SOME BASIC CONCEPTS OF CHEMISTRY

14. The molarity of HNO3 in a sample which has density 1.4 g/mL and mass percentage of 63% is ......
(Molecular Weight of HNO3 = 63) [JEE (M) 2020]
15. 10.30 mg of O2 dissolved into a liter of sea water of density 1.03 g/mL. The concentration of O 2 in ppm
is [JEE (M) 2020]
16. The first and second ionisation enthalpies of a metal are 496 and 4560 kJ mol–1, respectively. How many
moles of HCl and H2SO4, respectively, will be needed to react completely with 1 mole of the metal
hydroxide? [JEE (M) 2020]
(A) 1 and 0.5 (B) 2 and 0.5
(C) 1 and 2 (D)1 and 1
17. A solution of sodium sulfate contains 92 g of Na ions per kilogram of water. The molality of Na+ ions in
+

that solution in mol kg–1 is: [JEE (M) 2019]


(A) 16 (B) 8
(C) 4 (D) 12
18. For the following reaction, the mass of water produced from 445 g of C57H110O6 is: [JEE (M) 2019]
2C57H110O6(s) + 163O2(g) → 114CO2(g) + 110 H2O(l)
(A) 495 g (B) 490 g
(C) 890 g (D) 445 g
19. Amount of sugar (C12H22O11) required to prepare 2L of its 0.1M aqueous solution is:[JEE (M) 2019]
(A) 68.4 g (B) 17.1 g
(C) 34.2 g (D)136.8 g
20. 25 ml of the given HCl solution requires 30 mL of 0.1 M sodium carbonate solution. What is the volume of
this HCl solution required to titrate 30 mL of 0.2 M aqueous NaOH solution? [JEE (M) 2019]
(A) 25 mL (B) 50 mL
(C) 12.5 mL (D)75 mL
21. 50 mL of 0.5 M oxalic acid is needed to neutralize 25 mL of sodium hydroxide solution. The amount of
NaOH in 50 mL of the given sodium hydroxide solution is: [JEE (M) 2019]
(A) 4 g (B) 2 g
(C) 8 g (D) 1 g
22. A sample of NaClO3 is converted by heat to NaCl with a loss of 0.16 g of oxygen. The residue is
dissolved in water and precipitated as AgCl. Mass of AgCl (in g) obtained will be:
(Given: Molar mass of AgCl = 143.5 g mol–1) [JEE (M) 2018]
(A) 0.54 (B) 0.35
(C) 0.48 (D) 0.41
23. For per gram of reactant, the maximum quantity of N2 gas is produced in which of the following thermal
decomposition reactions? (Given: Atomic wt. – Cr = 52u, Ba = 137u) [JEE (M) 2018]
(A) 2NH4NO3(s) → 2N2(g) + 4H2O(g) + O2(g) (B) Ba(N3)2(s) → Ba(s) + 3N2(s)
(C) (NH4)2Cr2O7(s) → N2(g) + 4H2O(g) (D) 2NH3(g) → N2(g) + 3H2(g)
24. An unknown chlorohydrocarbon has 3.55% of chlorine. If each molecule of the hydrocarbon has one
chlorine atom only; chlorine atoms present in 1 g of chlorohydrocarbon are: [JEE (M) 2018]
(Atomic wt. of Cl = 35.5u; Avogadro’s constant = 6.023  10 mol )
23 –1

(A) 6.023  1021 (B) 6.023  1023


(C) 6.023  10 20
(D) 6.023  109

88
SOME BASIC CONCEPTS OF CHEMISTRY CHEMISTRY

25. The ratio of mass percent of C and H of an organic compound (CXHYOZ) is 6 : 1. If one molecule of the above
compound (CXHYOZ) contains half as much oxygen as required to burn one molecule of compound CXHY
completely to CO2 and H2O. The empirical formula of compound CXHYOZ is: [JEE (M) 2018]
(A) C3H6O3 (B) C2H4O
(C) C3H4O2 (D) C2H4O3
26. 1 gram of a carbonate (M2CO3) on treatment with excess HCl produces 0.01186 mole of CO2. The molar
mass of M2CO3 in g mol–1 is: [JEE (M) 2017]
(A) 1186 (B) 84.3
(C) 118.6 (D) 11.86
27. Number of atoms in the following samples of substances is the largest in [JEE (M) Online 2013]
(A) 4.0 g of hydrogen (B) 70.0g of chlorine
(C) 127.0 g of iodine (D) 48.0 g of magnesium
28. A gaseous hydrocarbon gives upon combustion 0.72 g of water and 3.08 g of CO2. The empirical formula
of the hydrocarbon is [JEE (M) Online 2013]
(A) C2H4 (B) C3H4
(C) C6H5 (D) C7H8
29. The density of 3M solution of sodium chloride is 1.252 g mL–1. The molality of the solution will be
(molar mass, NaCl = 58.5 g mol–1) [JEE (M) Online 2013]
(A) 2.60 m (B) 2.18 m
(C) 2.79 m (D) 3.00 m
30. 10 mL of 2M NaOH solution is added to 200 mL of 0.5 M of NaOH solution. What is the final
concentration? [JEE (M) Online 2013]
(A) 0.57 M (B) 5.7 M
(C) 11.4 M (D) 1.14 M
31. The molarity of a solution obtained by mixing 750 mL of 0.5 (M) HCl with 250 mL of 2(M) HCl will be
[AIEEE-2013]
(A) 1.75 M (B) 0.975 M
(C) 0.875 M (D) 1.78 M
32. The density (in g/mL) of a 3.60 M sulphuric acid solution that is 29% by mass will be [AIEEE-2007]
(A) 1.22 (B) 1.45
(C) 1.64 (D) 1.88
33. The reaction, [AIEEE-2007]
2Al(s) 6HCl(aq) 2Al3 (aq) 6Cl (aq) 3H 2 (g)
(A) 33.6 L H2(g) is produced regardless of temperature and pressure for every mole of Al that reacts
(B) 67.2 L H2(g) at STP is produced for every mole of Al that reacts
(C) 11.2 L H2(g) at STP is produced for every mole of HCl (aq) consumed
(D) 6 L HCl (aq) is consumed for every 3L H2 (g) produced
34. The density of a solution prepared by dissolving 120 g of urea (Mol. Mass = 60u) in 1000 g of water is
1.15 g/mL. The molarity of this solution is [AIEEE-2007]
(A) 1.02 M (B) 0.50 M
(C) 2.05 M (D) 1.78 M
35. Density of a 2.05M solution of acetic acid in water is 1.02 g/ml. The molality of the solution is:
[AIEEE-2006]
(A) 1.14 mol/kg (B) 3.28 mol/kg
(C) 2.28 mol/kg (D) 0.44 mol/kg

89
CHEMISTRY SOME BASIC CONCEPTS OF CHEMISTRY

36. If we consider that 1/6, in place of 1/12, mass of carbon atom is taken to be the relative atomic mass unit,
the mass of one mole of a substance will [AIEEE 2005]
(A) decrease twice (B) increase two fold
(C) remain unchanged (D) be function of molecular mass of substance
37. 6.02 ×1020 molecules of urea are present in 100 ml of its solution. The concentration of urea solution is
[AIEEE 2004]
(A) 0.001 M (B) 0.01 M
(C) 0.02 M (D) 0.1 M
38. To neutralise completely 20 mL of 0.1 M aqueous solution of phosphorous acid (H3PO3), the value of
0.1M aqueous KOH solution required is [JEE (M) 2004]
(A) 40 mL (B) 20 mL
(C) 10 mL (D) 60 mL
39. What volume of hydrogen gas at 273 K and 1 atm pressure will be consumed in obtaining 21.6 gm of
elemental boron (atomic mass = 10.8) from the reduction of boron trichloride by hydrogen?
[AIEEE 2003]
(A) 44.8 lit. (B) 22.4 lit.
(C) 89.6 lit. (D) 67.2 lit.
40. In an organic compound of molar mass 108 g/mol C, H and N atoms are present in 9 : 1 : 3.5 by weight.
Molecular formula can be: [AIEEE 2002]
(A) C6H8N2 (B) C7H10N
(C) C5H6N3 (D) C4H18N3

90
SOME BASIC CONCEPTS OF CHEMISTRY CHEMISTRY

1. Which one of the followings has maximum number of atoms? [NEET 2020]
(A) 1 g of Mg(s) [Atomic mass of Mg = 24] (B) 1 g of O2(g) [Atomic mass of O = 16]
(C) 1 g of Li (s) [Atomic mass of Li = 7] (D) 1 g of Ag(s) [Atomic mass of Ag = 108]
2. A mixture of 2.3 g formic acid and 4.5 g oxalic acid is treated with conc. H2SO4. The evolved gaseous
mixture is passed through KOH pellets. Weight (in g) of the remaining product at STP will be?
[NEET 2019]
(A) 2.8 g (B) 28 g
(C) 0.28 g (D) 0.028 g
3. In which case is number of molecules of water maximum? [NEET 2019]
(A) 0.00224 L of water vapours at 1 atm and 273 K (B) 0.18 mL of water
(C) 18 mL of water (D) 10–3 mol of water
4. In which case is number of molecules ofwater maximum? [NEET 2018]
(A) 0.00224 L of water vapours at 1 atm and273 K (B) 0.18 g of water
(C) 18 mL of water (D) 10–3 mol of water
5. Suppose the elements X and Y combine to form 2 compounds XY2 and X3Y2. When 0.1 mole of XY2weighs
10 g and 0.05 mole of X3Y2 weighs 9 g, the atomic weights of X and Y are [NEET 2016]
(A) 40, 30 (B) 60, 40
(C) 20, 30 (D) 30, 20
6. What is the mass of the precipitate formed when 50 mL of 16.9 % solution of AgNO3 is mixed with50 mL
of 16.9% solution of AgNO3 is mixed with 50 mL of 5.8% NaCl solution? [AIPMT 2015]
(Ag = 107.8, N = 14, O = 16, Na = 23, Cl = 35.5)
(A) 3.5 g (B) 7 g
(C) 14 g (D) 28 g
7. If Avogadro number NA, is changed from 6.022  10 mol–1 to 6.022  1020 mol–1, this would change
23

[AIPMT 2015]
(A) the mass of one mole of carbon
(B) the ratio of chemical species to each other in a balanced equation
(C) the ratio of elements to each other in a compound
(D) the definition of mass in units of grams
8. The number of water molecules is maximum in [AIPMT 2015]
(A) 1.8 gram of water (B) 18 gram of water
(C) 18 moles of water (D) 18 molecules of water
9. 20.0 g of a magnesium carbonate sample decomposes on heating to give carbon dioxide and 8.0 g
magnesium oxide. What will be the percentage purity of magnesium carbonate in the sample?
[AIPMT 2015 Cancelled]
(A) 75 (B) 96
(C) 60 (D) 84
10. A mixture of gases contains H2 and O2 gases in the ratio of 1:4 (w/w). What is the molar ratio of two
gases in the mixture? [AIPMT 2015]
(A) 4:1 (B) 16:1
(C) 2:1 (D) 1:4

91
CHEMISTRY SOME BASIC CONCEPTS OF CHEMISTRY

11. 1g of Mg is burnt with 0.56 g O2 in a closed vessel. Which reactant is left in excess and how much?
(At. wt. Mg = 24; O = 16) [AIPMT 2014]
(A) Mg, 0.16 g (B) O2, 0.16g
(C) Mg, 0.44g (D) O2, 0.28g
12. When 22.4 litres of H2(g)is mixed with 11.2 litres of Cl2(g), each at S.T.P, the moles of HCl(g) formed is
equal to [AIPMT 2014]
(A) 1 mol of HCl(g) (B) 2 mol of HCl(g)
(C) 0.5 mol of HCl(g) (D) 1.5 mol of HCl(g)
13. 6.02  10 molecules of urea are present in 100 mL of its solution. The concentration of solution is
20

[NEET 2013]
(A) 0.001 M (B) 0.1 M
(C) 0.02 M (D) 0.01 M
14. In an experiment it showed that 10 mL of 0.05M solution of chloride required 10 mL of 0.1 M solution of
AgNO3, which of the following will be the formula of the chloride (X stands for the symbol of the
element other than chlorine): [NEET Kar. 2013]
(A) X2Cl (B) X2Cl2
(C) XCl2 (D)XCl4
15. The number of atoms in 0.1 mol of a triatomic gas is (NA = 6.02  1023 mol–1) [AIPMT 2010]
(A) 6.026  10 22
(B) 1.806  10 23

(C) 3.600  10 23
(D) 1.800  1022
16. What is the [OH–] in the final solution prepared by mixing 20.0 mL of 0.050 M HCl with 30.0 mL of
0.10 M Ba(OH)2? [2009]
(A) 0.40 M (B) 0.0050 M
(C) 0.12 M (D) 0.10 M
17. 10g of hydrogen and 64g of oxygen were filled in a steel vessel and exploded. Amount of water produced
in this reaction will be: [2009]
(A) 3 mol (B) 4 mol
(C) 1 mol (D)2 mol
18. An organic compound contains carbon, hydrogen and oxygen. Its elemental analysis gave C, 38.71% and
H, 9.67%. The empirical formula of the compound would be: [2008]
(A) CH3O (B) CH2O
(C) CHO (D)CH4O
19. Concentrated aqueous sulphuric acid is 98% H2SO4 by mass and has a density of 1.80 g mL–1. Volume of
acid required to make one litre of 0.1 M H2SO4 solution is: [2007]
(A) 16.65 mL (B) 22.20 mL
(C) 5.55 mL (D)11.10 mL
20. Specific volume of cylindrical virus particle is 6.02  10–2 cc/gm, whose radius and length are 7Å and
10Å respectively. If NA = 6.02  1023, find molecular weight of virus [2001]
(A) 3.08  10 kg/mol
3
(B) 3.08  10 kg/mol
4

(C) 1.54  10 kg/mol


4
(D)15.4 kg/mol
21. Assuming fully decomposed, the volume of CO2 released at STP on heating 9.85 g of BaCO3 (Atomic
mass, Ba = 137) will be [2000]
(A) 2.24 L (B) 4.96 L
(C) 1.12 L (D)0.84 L

92
SOME BASIC CONCEPTS OF CHEMISTRY CHEMISTRY

22. Haemoglobin contains 0.334% of iron by weight. The molecular weight of haemoglobin is approximately
67200. The number of iron atoms (Atomic weight of Fe is 56) present in one molecule of haemoglobin is
[1998]
(A) 4 (B) 6
(C) 3 (D) 2
23. A metal oxide has the formula Z2O3. It can be reduced by hydrogen to give free metal and water. 0.1596
g of the metal oxide requires 6 mg of hydrogen for complete reduction. The atomic weight of the metal is
[1988]
(A) 27.9 (B) 159.6
(C) 79.8 (D) 55.8
24. Percentage of Se (at. mass = 78.4) in peroxidase anhydrase enzyme is 0.5% by weight, then minimum
molecular mass of peroxidase anhydrase enzyme is [AIIMS 2008]
(A) 1.576  10 4
(B)1.576  10 3

(C) 15.76 (D) 2.136  104


25. The specific heat of a metal is 0.16. Its approximate atomic weight would be [AIIMS 1999]
(A) 40 (B)16
(C) 32 (D) 64

93
CHEMISTRY ATOMIC STRUCTURE

ATOMIC STRUCTURE
INTRODUCTION
Atom, the ultimate electrically neutral particle of any substance made up of mainly three fundamental particles
(electron, proton and neutron) which show its existence in chemical reaction where it represents particular
property for each element.
In this chapter, we deal about the structure of atom based on different atomic models starting from Dalton’s
atomic theory to quantum mechanical model of atom developed by Erwin Schrödinger.

DALTON’S ATOMIC THEORY


Dalton, in 1808, proposed that matter was made up of extremely small, indivisible particles called atoms (in
Greek atom means which cannot be cut). This concept was accepted for number of years.
The main postulates of Dalton’s atomic theory are
• Matter is made up of small indivisible particles, called atoms.
• Atoms can neither be created nor destroyed. This means that a chemical reaction is just a simple rearrangement
of atoms and the same number of atoms must be present before and after the reaction.
• Atom is the smallest particle of an element which takes part in a chemical reaction.
• Atoms of the same element are identical in all respects especially, size, shape and mass.
• Atoms of different elements have different mass, shape and size.
• Atoms of different elements combine in a fixed ratio of small whole numbers to form compound atoms, called
molecules.
However, the researches done by various eminent scientists and the discovery of radioactivity have established
beyond doubt, that atom is not the smallest indivisible particle but had a complex structure of its own and was
made up of still smaller particles like electrons, protons, neutrons, etc. At present about 35 different subatomic
particles are known but the three particles namely electron, proton and neutron are regarded as the fundamental
particles.
We shall now take up the brief study of these fundamental particles. The existence of electrons in atoms was first
suggested, by J.J. Thomson, as a result of experimental work on the conduction of electricity through gases at low
pressures and high voltage, which produces cathode rays consisting of negatively charged particles, named as
electrons. The e/m ratio for cathode rays is fixed whose values is 1.76  108 C/g.
We know that an atom is electrically neutral, if it contains negatively charged electrons it must also contain some
positively charged particles. This was confirmed by Goldstein in his discharge tube experiment with perforated
cathode. Applying high voltage across the electrodes of a discharge tube it was found that some rays were
coming from the side of the anode which passed through the holes in the cathode. These anode rays (canal rays)
consisted of positively charged particles formed by ionization of gas molecules by the cathode rays. The charge
to mass ratio (e/m value) of positively charge particles was found to be maximum when the discharge tube was
filled with hydrogen gas as hydrogen is the lightest element. These positively charged particles are called protons.
e/m varies with the nature of gas taken in the discharge tube. The positive particles are positive residues of the
gas left when the gas is ionized.
The neutral charge particle, neutron was discovered by James Chadwick by bombarding boron or beryllium with
–particles.

94
ATOMIC STRUCTURE CHEMISTRY

Discovery of Electron (Not in CBSE)


In 1897, J.J. Thomson placed a positively charged plate on one side of the tube and a negatively charged
plate on the other side of tube. The detecting screen was coated with zinc sulfide. Observations are:
 The beam was deflected away from the negative plate toward the positive plate.
 He realized particles that made up the beam must be (-)vely charged since like charges repel & opp.
charges attract.
 By balancing the deflections made by magnet with that made by electrical field, Thomson was able to
calculate ratio of charge to mass of an e- =1.76x1011 C/kg. e/m is known as specific charge.
 Electrons are deflected into a parabolic path by electric field &circular path by a magnetic field.

Note: A vacuum tube with metal plates attached to a high voltage source produces a greenish beam called
cathode rays. These rays move from the cathode (negative charge) to the anode (positive charge).

Note: If P is reduced in discharge tube to 1mm then coloured glow (depends on nature of gas) is observed
when P is further reduced to 0.01mm then invisible cathode rays come out.

Properties of Cathode Rays


 They travel in straight lines as they cast a sharp shadow of the solid object placed in their path
 They rotate the light paddle wheel which shows they are made up of material particles
 They produce heating effects (on metal plates) and ionize the gases
 They produce X-Rays and affect the photographic plates. They penetrate through the thin metal sheets

MILIKAN OIL DROP EXPERIMENT (NOT IN CBSE)


 In 1906 Robert Millikan passed mineral oil through an atomizer into an apparatus where he could
observe the drops with a magnifier and make measurement on them as they drifted downward.
 He found that the least charge on any of the droplets was 1.60  10–19 coulombs and that larger droplets
always had a charge that was some multiple of this value.
 Knowing Thomson’s work of charge to mass ratio and the charge on an individual electron, it was
possible to calculate the mass of the electron as 9.11  10–31 kg.
 He measured the charge of an e– by balancing the pull of gravity on oil droplets with an upward electrical force.
 Knowing the e/m ratio that Thomson had calculated, Millikan was able to calculate the charge on each droplet.
 He found that all droplets had a charge of 1.60  10–19 coulombs or multiples of that charge.
 The conclusion was that this had to be the charge of an electron

95
CHEMISTRY ATOMIC STRUCTURE

CHADWICK’S DISCOVERY OF NEUTRON(NOT IN CBSE)

DISCOVERY OF PROTON–GOLDSTEIN (1886) PROPERTIES OF ANODE


RAYS:(NOT IN CBSE)
 They travel in straight lines though slower than cathode rays and are also made up of material particles
 e/m ratio is not constant and depends on the gas taken in the tube
 Charge is not constant and depends on the gas (q = ne)
 Mass is not constant and is almost equal to the mass of the gas. m (proton) = 1.67  10–24gm
Characteristics of the three fundamental particles are:
Electron Proton Neutron

Symbol E or e– P n

Approximate relative mass with


1/1836 1 1
respect to proton

Approximate relative charge with


–1 +1 No charge
respect to proton

Absolute mass in kg 9.10910–31 1.67310–27 1.67510–27

Absolute mass in amu 5.48510–4 1.007 1.008

Absolute charge (coulomb) 1.60210–19 1.60210–19 0

Absolute charge (e.s.u.) 4.8  10–10 4.8  10–10 0

The atomic mass unit (amu) is 1/12 of the mass of an individual atom of 6C12, i.e. 1.660  10–27 kg.
The neutron and proton have approximately equal masses of 1 amu and the electron is about 1836 times
lighter, its mass can sometimes be neglected as an approximation.
The electron and proton have equal, but opposite, electric charges while the neutron has no charge.

96
ATOMIC STRUCTURE CHEMISTRY

ATOMIC MODELS
We know about the fundamental particles of the atom. Now let us see, how these particles are arranged in an
atom to suggest a model of the atom.
Thomson’s Model (Not in CBSE)
J.J. Thomson, in 1904, proposed that there was an equal and opposite positive charge enveloping the
electrons in a matrix. This model is called the plum – pudding model after a type of Victorian dessert in
which bits of plums were surrounded by matrix of pudding.

electron
Positive sphere

This model could not satisfactorily explain the results of scattering experiment carried out by Rutherford who
worked with Thomson.
RUTHERFORD’S -PARTICLE SCATTERING EXPERIMENT(NOT IN CBSE)
– particles emitted by radioactive substance were shown to be dipositive helium ions (He 2+) having a mass
of 4 units and 2 units of positive charge.
Rutherford allowed a narrow beam of –particles to fall on a very thin gold foil of thickness of the order of
0.0004 cm and determined the subsequent path of these particles with the help of a zinc sulphide fluorescent
screen. The zinc sulphide screen gives off a visible flash of light when struck by an -particle, as ZnS has the
remarkable property of converting kinetic energy of -particle into visible light. [For this experiment,
Rutherford specifically used  particles because they are relatively heavy resulting in high momentum].
Alpha particle

Fluorescent screen (ZnS)


or photographic film

Thin Gold foil

Observations
• Majority of the –particles pass straight through the gold strip with little or no deflection.
• Some –particles are deflected from their path and diverge.
• Very few –particles are deflected backwards through angles greater than 90.
• Some were even scattered in the opposite direction at an angle of 180 [Rutherford was very much
surprised by it and remarked that “It was as incredible as if you fired a 15–inch shell at a piece of tissue
paper and it came back and hit you”]. There is far less difference between air and bullet than there is
between gold atoms and -particles assuming of course that density of a gold atom is evenly distributed.

97
CHEMISTRY ATOMIC STRUCTURE

Conclusions
• The fact that most of the -particles passed straight through the metal foil indicates the most part of the
atom is empty.
• The fact that few -particles are deflected at large angles indicates the presence of a heavy positively
charge body, i.e. for such large deflections to occur  - particles must have come closer to or collided
with a massive positively charged body.
• The fact that one in 20,000 have deflected at 180° backwards indicates that volume occupied by this
heavy positively charged body is very small in comparison to total volume of the atom.
1
N ( )  …(1)
 
sin 4  
2
1
N ( )  …(2)
K .E.2
N ( )  Z 2 …(3)

1
N ( )  …(4)
r2
Here, Z = atomic number of element of metal foil
K.E. = K.E. of -particle (initially)
 = Scattering angle
r = Distance of screen from foil
N = no. of particles scattered

Radius of Nucleus
R = R0A1/3 …(1)
or R = 1.4 × 10–15A1/3 metre …(2)
–15 –15
value of R0 can be 1.1 × 10 to 1.44 × 10 metre.
Rutherford’s Atomic Model (Not in CBSE)
On the basis of the above observation, and having realized that the rebounding -particles had met something
even more massive than themselves inside the gold atom, Rutherford proposed an atomic model as follows.
• All the protons (+ve charge) and the neutrons (neutral charge), i.e. nearly the total mass of an atom is
present in a very small region at the centre of the atom. The atom’s central core is called nucleus.
• The size of the nucleus is very small in comparison to the size of the atom. Diameter of the nucleus is
about 10–13 cm while the atom has a diameter of the order of 10–8 cm. So, the size of atom is 105 times
more than that of nucleus.
• Most of the space outside the nucleus is empty.
• The electrons, equal in number to the net nuclear positive charge, revolve around the nucleus with fast
speed in various circular orbits.
• The centrifugal force arising due to the fast speed of an electron balances the coulombic force of
attraction of the nucleus and the electron remains stable in its path. Thus according to him atom consists
of two parts (a) nucleus and (b) extra nuclear part.

98
ATOMIC STRUCTURE CHEMISTRY

DEFECTS OF RUTHERFORD’S ATOMIC MODEL


Position of Electrons
The exact positions of the electrons from the nucleus are not mentioned.
2. Stability of the Atom
Neils Bohr pointed out that Rutherford’s atom should be highly unstable.
According to the law of electro-dynamics, the electron should therefore,
continuously emit radiation and lose energy. As a result of this a moving +
electron will come closer and closer to the nucleus and after passing through a
spiral path, it should ultimately fall into the nucleus.
It was calculated that the electron should fall into the nucleus in less than 10 –8 sec. But it is known that
electrons keep moving outside the nucleus.
To solve this Problem, Neils Bohr proposed an improved form of Rutherford’s atomic model.
Before going into the details of Neils Bohr model we would like to introduce you some important atomic terms.
Example 1: What would have happened if neutrons had been used in Rutherford’s experiment? Explain your answer
Solution: They would not have been repelled so it is unlikely that any would ‘bounce back’. Some could
be absorbed by nucleus.
Example 2: What would have happened if aluminium had been used instead of gold in the alpha scattering
experiment? Explain
Solution: The charge on the nucleus is much smaller so deflection would be smaller.
Example 3: For cathode rays, the value of e/m -
(A) Is independent of the nature of the cathode and the gas filled in the discharge tube
(B) Is constant
(C) Is –1.7588  108C/g
(D) All of the above are correct
Solution:(D) Cathode rays consists of electrons which are fundamental particles of matter.
Example 4: Which has highest e/m ratio
(A) He2+ (B) H+
(C) He1+ (D) H
Solution:(B) Mass of H+ is minimum
Example 5: Arrange the following particles in increasing order of values of e/m ratio : Electron (e), proton
(p), neutron (n) and a-particle ()
(A) n, p, e,  (B) n, , p, e
(C) n, p, , e (D) e, p, n, 
Solution:(B) Electron Proton Neutron -particle
e 1 unit 1 unit zero 2 units
m 1/1837 unit 1 unit 1 unit 4-units
e/m 1837 1 zero ½
Example 6: Mass of neutron is..... times the mass of electron -
(A) 1840 (B) 1480
(C) 2000 (D) None
Solution: (A) Mass of neutron = 1.675  10–27 kg, mass of electron = 9.108  10–31 kg.

99
CHEMISTRY ATOMIC STRUCTURE

ATOMIC TERMS
• Atomic number (Z): The atomic number of an element is the number of protons contained in the
nucleus of the atom of that element.
• Nucleons:Protons and neutrons are present in a nucleus. So, these fundamental particles are collectively
known as nucleons.
• Mass number (A): The total number of protons and neutrons, i.e. the number of nucleons present in the
nucleus is called the mass number of the element.
• Nuclide: Various species of atoms in general. A nuclide has specific value of atomic number and mass
number.

IUPAC Notation of an Atom (nuclide)


Let X be the symbol of the element. Its atomic number be Z and mass number be A. Then the element can be
A
represented as Z X .
• Isotopes: Atoms of the element with same atomic number but different mass number, e.g. 11H, 12H, 13H
. There are three isotopes of hydrogen.
• Isobars: Atoms having the same mass number but different atomic numbers, e.g. 32
15 P and 32
16 S are
called isobars.
• Isotones: Atoms having the same number of neutrons but different number of protons, e.g. 14
6 C, 16 15
8 O, 7 N

are called isotones.


• Isoelectronic: Atoms, molecules or ions having same number of electrons are isoelectronic e.g. N2,
CO, CN–.
• Nuclear isomer: Nuclear isomers (isomeric nuclei) are the atoms with the same atomic number and
same mass number but with different radioactive properties.
Example of nuclear isomers is
Uranium –X (half–life 1.4 min) and
Uranium –Z (half–life 6.7 hours)
The reason for nuclear isomerism is the different energy states of the two isomeric nuclei.
Other examples are
69 69
30 Zn 30 Zn (T1/2 = 13.8 hr) (T1/2 = 57 min)
80 80
35 Br 35 Br (T1/2 = 4.4 hour) (T1/2 = 18 min)
• Isosters: Molecules having same number of atoms and also same number of electrons are called
isosters, e.g.
(i) N2 and CO
(ii) CO2 and N2O
(iii) HCl and F2
• Atomic mass unit: Exactly equal to 1/12 of the mass of 6C12 atom.
1 amu = 1.66 10–27 kg = 931.5 MeV
Example 7: Ion that is iso-electronic with CO is-
(A) CN– (B) O+2
(C) O−2 (D) N +2
Solution:(A) 14 electrons.

100
ATOMIC STRUCTURE CHEMISTRY

Example 8: The specific charge of a proton is 9.6 × 107 C kg-1, then for an α-particles it will be-
(A) 2.4 × 107C kg–1 (B) 4.8 × 107C kg–1
(C) 19.2 × 107C kg–1 (D) 38.4 × 107C kg–1
e e 2e e 9.6 107
Solution:(B) for P = 9.6 107 ; for = = = = 4.8 107
m m 4m 2m 2
Example 9: The triad of nuclei that is isotonic is- [IIT-1988]
14 15 17 12 14 19
(A) C,
6 7
N, 9
F (B) C,
6
N,
7 9
F
14 14 17 14 14 19
(C) C,
6
N,
7 9
F (D) C,
6 7
N, 9
F
Solution:(A)
Example 10: Element with mass no. 81 contains 31.7% more neutrons as compared to protons. Assign atomic
symbol.
Solution: Mass number = 81, i.e., p + n = 81
31.7
If protons = x, then neutrons = x +  x = 1.317 x
100
81
 x + 1.317 x = 81 or 2.317 x = 81 or x = = 35
2.317
Thus, protons = 35, i.e., atomic no. = 35
81
Hence, the symbols is 35
Br

Example 11: Ion with mass no. 37 possesses 1 unit of (-)ve charge. If ion contains 11.1% more ‘n’ than e-,
find symbol?
Solution: Suppose number of electrons in the ion = x
11.1
Then number of neutrons = x + x = 1.111x
100
No. of electrons in the neutral atom = x –1  No. of protons = x – 1
Mass number = No. neutrons + No. of protons  37 = 1.111 x + x – 1 or 2.111 x = 38 or x =18
 No. of protons = Atomic no. = x − 1 = 18 − 1 = 17
37
Hence, the symbols of the ion will be 17 Cl −1

SOME IMPORTANT CHARACTERISTICS OF A WAVE


A wave is a sort of disturbance which originates from some vibrating source and travels outward as a
continuous sequence of alternating crests and troughs. Every wave has five important characteristics, namely,
wavelength (), frequency (), velocity (c), wave number (  ) and amplitude (a).
Crest  Crest


Trough Trough

101
CHEMISTRY ATOMIC STRUCTURE

Ordinary light rays, X–rays, –rays etc. are called electromagnetic radiations because similar waves can be
produced by moving a charged body in a magnetic field or a magnet in an electric field. These radiations
have wave characteristics and do not require any medium for their propagation.
• Wave length (): The distance between two neighbouring troughs or crests is known as wavelength. It is
denoted by  and is expressed in units used for length, i.e. cm, m, nanometers (1 nm = 10–9m) or
Angstrom (1 Å = 10–10 m).
• Frequency (): The frequency of wave is the number of times wave passes through a given point in a
medium in one second. It is denoted by (nu) and is expressed in cycles per second (cps) or hertz (Hz).
• 1Hz = 1cps.
• The frequency of a wave is inversely proportional to its wavelength ()
1 c
 or  =
 
• Velocity: The distance traveled by the wave in one second is called its velocity. It is denoted by c and is
expressed in cm sec–1.
c
c =  or  =

• Wave number (  ) : It is defined as number of wavelengths per cm. It is denoted by  and is expressed in cm–1.
1 
= (or)  =
 c
• Amplitude: It is the height of the crest or depth of the trough of a wave and is denoted by a. It
determines the intensity or brightness of the beam of light.

Electromagnetic Spectrum

Example12: Frequency ratio between violet (400 nm) and red (750 nm) radiations in the visible spectrum, is
𝑣1 750 15
Solution: 𝑣2
= 400 = 8

Example 13: Calculate the wave-number of lines having the frequency of 5 × 1016 cycles per sec
𝑣
Solution: 𝑣̅ = 𝑐 = 1.66 × 108

102
ATOMIC STRUCTURE CHEMISTRY

Limitations of Wave Theory


The wave theory of light could not explain the following phenomena
1. Black body radiation
2. Photoelectric effect
3. Line spectra of atoms

Blackbody Radiation
When solids are heated they emit radiation over a wide range of wavelengths. For example, when an iron rod
is heated in a furnace, it first turns to dull red and then progressively becomes more and more red as the
temperature increases. As this is heated further, the radiation emitted becomes white and then becomes blue
as the temperature becomes very high.
In terms of frequency, it means that the frequency of emitted radiation goes from a lower frequency to a
higher frequency as the temperature increases. The red colour lies in the lower frequency region while blue
colour belongs to the higher frequency region of the electromagnetic spectrum.
The ideal body, which emits and absorbs radiations of all frequencies, is called a black body and the
radiation emitted by such a body is called black body radiation.
➢ The exact frequency distribution of the emitted radiation (i.e., intensity versus wavelength curve of the
radiation) from a black body depends only on its temperature.
➢ At a given temperature, intensity of radiation emitted increases with decrease of wavelength, reaches a
maximum value at a given wavelength and then starts decreasing with further decrease of wavelength.
➢ The maxima shifts to shorter wavelengths with increasing temperature

PLANCK’S QUANTUM THEORY


When a black body is heated, it emits thermal radiations of different wavelengths or frequency. To explain
these radiations, Max Planck put forwarded a theory known as Planck’s quantum theory. The main points of
quantum theory are
• Substances radiate or absorb energy discontinuously in the form of small packets or bundles of energy.
• The smallest packet of energy is called quantum. In case of light the quantum is known as photon.

103
CHEMISTRY ATOMIC STRUCTURE

• The energy of a quantum is directly proportional to the frequency of the radiation. E  (or) E = h where  is the
frequency of radiation and h is Planck’s constant having the value 6.626  10–27 erg – sec or 6.626  10–34 J–sec.
• A body can radiate or absorb energy in whole number multiples of a quantum h, 2h, 3h,…, nh.
Where n is the positive integer.
• Neils Bohr used this theory to explain the structure of atom.
Example 14: Suppose 10–17J of energy is needed by the interior of human eye to see an object. How many
photons of green light (=550 nm) are needed to generate this minimum amount of energy?
(A) 14 (B) 28
(C) 39 (D) 42
Solution:(B) Let the number of photons required = n
hc 10−17   10−17  550 10−9
n = 10−17 ; n = = = 27.6 = 28 photons
 hc 6.626 10−34  3 108
Everyday Evidence for Photons
 Red light is used in photographic darkrooms, it is not energetic enough to break X-Ag bond in black
& white films
 Ultraviolet light causes sunburn but visible light does not because UV photons are more energetic.
 Our eyes detect color because photons of different energies trigger different chemical reactions in
retina cells.

PHOTOELECTRIC EFFECT
Sir J.J. Thomson observed that when a light of certain frequency strikes the surface of a metal, electrons are
ejected from the metal. This phenomenon is known as photoelectric effect and the ejected electrons are
called photoelectrons.
Wave theory predicts that
 emission of photoelectrons should happen at all frequencies.
 Electrons in the metal would absorb energy continuously from radiation of any frequency, and be emitted
when they had absorbed enough energy.This process would take longer at lower frequencies but should
still happen.
 Also, there should be no maximum kinetic energy of the emitted electrons.
A few metals, which are having low ionization energy like Cesium, show this effect under the action of
visible light but many more show it under the action of more energetic ultraviolet light.
Light

electrons
– +

Evacuated quartz tube

– +
A

An evacuated tube contains two electrodes connected to a source of variable voltage, with the metal plate
whose surface is irradiated as the anode. Some of the photoelectrons that emerge from this surface have

104
ATOMIC STRUCTURE CHEMISTRY

enough energy to reach the cathode despite its negative polarity, and they constitute the measured current.
The slower photoelectrons are repelled before they get to the cathode. When the voltage is increased to a
certain value Vo, of the order of several volts, no more photoelectrons arrive, as indicated by the current
dropping to zero. This extinction voltage (or also referred as stopping potential) corresponds to the maximum
photoelectron kinetic energy i.e. eVo = ½ mv2
The experimental findings are summarised as below:
• Electrons come out as soon as the light (of sufficient energy) strikes the metal surface.
• The light of any frequency will not be able to cause ejection of electrons from a metal surface. There is a
minimum frequency, called the threshold (or critical) frequency, which can just cause the ejection. This
frequency varies with the nature of the metal. The higher the frequency of the light, the more energy the
photoelectrons have. Blue light results in faster electrons than red light.
• Photoelectric current is increased with increase in intensity of light of same frequency, if emission is
permitted, i.e. a bright light yields more photoelectrons than a dim one of the same frequency, but the
electron energies remain the same.
Light must have stream of energy particles or quanta of energy (h). Suppose, the threshold frequency of
light required to eject electrons from a metal is o, when a photon of light of this frequency strikes a metal it
imparts its entire energy (ho) to the electron.
E = ho E > ho

K.Emax = h – ho
K. E. = 0

Metal

“This energy enables the electron to break away from the atom by overcoming the attractive influence of the
nucleus”. Thus each photon can eject one electron. If the frequency of light is less than o there is no ejection
of electron. If the frequency of light is higher than o (let it be ), the photon of this light having higher
energy (h), will impart some energy to the electron that is needed to remove it away from the atom. The
excess energy would give a certain velocity (i.e. kinetic energy) to the electron.
h = ho + K.E.
h = ho + ½ mv2
½ mv2 = h–ho
Where  = frequency of the incident light
o = threshold frequency
ho is the threshold energy (or) the work function denoted by  = ho (minimum energy of the photon to liberate
electron). It is constant for particular metal and is also equal to the ionization potential of gaseous atoms.
The kinetic energy of the photoelectrons increases linearly with the frequency of incident light. Thus, if the
energy of the ejected electrons is plotted as a function of frequency, it result in a straight line whose slope is
equal to Planck’s constant ‘h’ and whose intercept is ho.

K.E.
of
Photoelectrons

105
CHEMISTRY ATOMIC STRUCTURE

Example 15: 25 watt bulb emits monochromatic yellow light of λ = 0.57  10-6 m. Cal. the rate of emission of
quanta per second.
Solution: Energy emitted by the bulb = 25 watt = 25J s-1 (1 watt = 1 J s-1)
c
Energy of one photon (E) = h = h

Here,  = 0.57 m = 0.57 10 m −6
(1m = 10-6 m)
Putting c = 3 108 ms −1 , h = 6.62 10−34 J s, we get
25Js −1
 No. of photons emitted per sec = −19
= 7.18 1019
3.48 10 J
Example 16: Iodine molecule dissociates into atoms after absorbing light of 4500Å. If one quantum of
radiation is absorbed by each molecule, calculate the kinetic energy of iodine atoms. (Bond
energy of I2 = 240 kJ/mol) [IIT-1995]
Solution: Bond energy of I2 = 240 kJ mol -1 = 240  103 J mol-1
240 103
= J molecule−1
6.023 1023
= 3.984 10−19 J molecule−1
6.626 10−34 Js 108 ms −1
hc
Energy absorbed = = = 4.417 10−19 J
 4500 10 m−10

Kinetic energy = Absorbed energy – Bond energy


 Kinetic energy = 4.417 10−19 − 3.984 10−19 J = 4.33 10−20 J
4.33 10−20
 Kinetic energy of each atom of Iodine = = 2.165 10−20
2
Example 17: Photoelectric emission is observed from a surface for frequencies 1 and 2 of the incident
radiation (1>2). If the maximum kinetic energies of the photoelectrons in two cases are in ratio
1:K then the threshold frequency 0 is given by
 2 −1 K 1 − 2
(A) (B)
K −1 K −1
K 2 − 1  2 −1
(C) (D)
K −1 K
Solution:(B) KE1 = h1 – ho ; KE2 = h2–ho
KE1 1
It is given that; =
KE 2 K
h 1 − h o 1 K −
= ; ( K1 − 2 ) =  o ( K −1) ;  0 = 1 2
h 2 − h o K K −1
Example 18: The threshold frequency  for a metal is 6 × 1014 s–1. Calculate the kinetic energy of an electron
emitted when radiation of frequency  = 1.1 × 1015 s–1 hits the metal.
1
Solution: K.E. = meV2 = h ( – 0)
2
 K.E. = (6.626 × 10–34) (1.1 × 1015 – 6 × 1014)
 K.E. = (6.626 × 10–34) (5 × 1014)= 3.313 × 10–19 J

106
ATOMIC STRUCTURE CHEMISTRY

Example 19: When electromagnetic radiation of wavelength 310 nm fall on the surface of Sodium, electrons
are emitted with K.E. = 1.5 eV. Determine the work function (W0) of Sodium.
12400
Solution: h = = 4 eV
3100
1
meV2 = 1.5 eV
2
1
 h0 = W0 = h– meV2 = 4 – 1.5 = 2.5 eV
2
Example 20: When electromagnetic radiation of wavelength 300 nm falls on the surface of sodium, electrons
are emitted with a kinetic energy of 1.68 × 105 J mol–1. What is the minimum energy needed to
remove an electron from sodium? What is the maximum wavelength that will cause a
photoelectron to be emitted?
Solution: The energy (E) of a 300 nm photon is given by h =hc/ 
6.626  10−34 J s  3.0  108 m s −1
= = 6.626 × 10–19 J
300  10−9 m
The energy of one mole of photons
= 6.626 × 10–19 × 6.022 × 1023 = 3.99 × 105 J mol–1
The minimum energy needed to remove a mole of electrons from sodium
= (3.99 – 1.68) 105 J mol–1 = 2.31 × 105 J mol–1
2.31105 J mol-1
The minimum energy for one electron = −1
= 3.84 × 10–19 J
6.022  10 electrons mol
23

This corresponds to the wavelength


hc 6.626 10−34 Js  3.0 108 ms −1
 = = = 517 nm
E 3.84 10−19 m
(This corresponds to green light)
Example 21: The threshold frequency 0 for a metal for a metal is 7.0 × 1014 s–1. Calculate the kinetic energy of
an electron emitted when radiation of frequency  = 1.0 × 1015 s–1 hits the metal.
Solution: According to Einstein’s equation Kinetic energy –1/2 mev2 = h( – 0) = (6.626 × 10–34 J s) (1.0 ×
1015 s–1 – 7.0 × 10–14 s–1) = (6.626 × 10–34 J s) × (3.0 × 1014 s–1) = 1.988 × 10–19 J

BOHR’S ATOMIC MODEL


Bohr developed a model for hydrogen atom and hydrogen like one–electron species (hydrogenic species). He
applied quantum theory in considering the energy of an electron bound to the nucleus.

IMPORTANT POSTULATES
• An atom consists of a dense nucleus situated at the centre with the electron revolving around it in circular
orbits without emitting any energy. The force of attraction between the nucleus and an electron is equal
to the centrifugal force of the moving electron.
• Of the finite number of circular orbits around the nucleus, an electron can revolve only in those orbits
whose angular momentum (mvr) is an integral multiple of factor h 2
nh
mvr =
2

107
CHEMISTRY ATOMIC STRUCTURE

where m = mass of the electron


v = velocity of the electron
n = orbit number in which electron is present
r = radius of the orbit
• As long as an electron is revolving in an orbit it neither loses nor gains energy. Hence, these orbits are
called stationary states. Each stationary state is associated with a definite amount of energy and it is also
known as energy levels. The greater the distance of the energy level from the nucleus, the more is the
energy associated with it. The different energy levels are numbered as 1,2,3,4, (from nucleus onwards) or
K, L, M, N etc.
• Ordinarily an electron continues to move in a particular stationary state without losing energy. Such a
stable state of the atom is called as ground state or normal state.
• If energy is supplied to an electron, it may jump (excite) instantaneously from lower energy (say 1) to
higher energy level (say 2, 3, 4, etc.) by absorbing one or more quanta of energy. This new state of
electron is called as excited state. The quantum of energy absorbed is equal to the difference in energies
of the two concerned levels.
Since the excited state is less stable, atom will lose it’s energy and come back to the ground state.
Energy absorbed or released in an electron jump, (E) is given by
E = E2 – E1 = h
where E2 and E1 are the energies of the electron in the first and second energy levels, and  is the
frequency of radiation absorbed or emitted.

Note: If the energy supplied to hydrogen atom is less than 13.6 eV, it will accept or absorb only
those quanta which can take it to a certain higher energy level, i.e. all those photons having
energy less than a particular energy level will not be absorbed by hydrogen atom. But if
energy supplied to hydrogen atom is more than 13.6 eV then all photons are absorbed and
excess energy appears as kinetic energy of emitted photo electron.

RADII OF VARIOUS ORBITS OF HYDROGEN ATOM


Consider, an electron of mass ‘m’ and charge ‘e’ revolving around a nucleus of charge Ze (where, Z = atomic
number and e is the charge of the proton) with a tangential velocity v. r is the radius of the orbit in which
electron is revolving.
By Coulomb’s law, the electrostatic force of attraction between the moving electron and nucleus is
KZe2
Coulombic force =
r2
v

1
m

K= (where o is permittivity of free space)


4 0
s
diu

K = 9 109 Nm2 C–2


ra

In C.G.S. units, value of K = 1 dyne cm2 (esu)–2


mv 2
The centrifugal force acting on the electron is
r
Since the electrostatic force balance the centrifugal force, for the stable electron orbit.
2
mv 2 KZe
= 2 …(1)
r r
2
(or) v2 = KZe …(2)
mr

108
ATOMIC STRUCTURE CHEMISTRY

According to Bohr’s postulate of angular momentum quantization, we have


nh
mvr =

nh
v=
2 πmr
n 2h 2
v2 = … (3)
4π 2 m 2 r 2
Equating (2) and (3)
KZe2 n 2h 2
= 2 2 2
mr 4π m r
n 2h 2
Solving for r we get r =
4π 2 mKZe2
where n = 1,2,3, …, 
Hence, only certain orbits whose radii are given by the above equation are available for the electron. The
greater the value of n, i.e. farther the energy level from the nucleus the greater is the radius.
The radius of the smallest orbit (n = 1) for hydrogen atom (Z = 1) is ro.
1  ( 6.626  10 )
2
2 −34
n 2h 2
ro = 2 2 = = 5.29 10–11 m = 0.529 Å
4π me K 4  ( 3.14 )  9  10 −31  (1.6  10 −19 )  9  109
2 2

Radius of nth orbit for an atom with atomic number Z is simply written as
n2
rn = 0.529× Å
Z

CALCULATION OF ENERGY OF AN ELECTRON


The total energy (E) of the electron is the sum of kinetic energy and potential energy.
Kinetic energy of the electron = ½ mv2
KZe2 2
Potential energy =  columbic force × dr =  2
dr = -KZe
r r
KZe2
Total energy = 1/2 mv2 – … (4)
r
From equation (1) we know that
mv 2 KZe2
= 2
r r
KZe2
 ½ mv2 =
2r
Substituting this in equation (4)
KZe2 KZe2 KZe2
Total energy (E) = – = −
2r r 2r
Substituting for r, gives us
2π2 mZ2e4 K 2
E= where n = 1, 2, 3, …
n 2h 2
This expression shows that only certain energies are allowed to the electron. Since this energy expression
consist of so many fundamental constant, we are giving you the following simplified expressions.
Z2
E = –21.8 10–12 2 erg per atom
n

109
CHEMISTRY ATOMIC STRUCTURE

Z2 Z2
= –21.8 10–19 J per atom = –13.6  eV per atom
n2 n2
(1 eV = 3.83 10–23 kcal)
1 eV = 1.602 10–12 erg
(1 eV = 1.602 10–19J)
Z2
[E = –313.6  2 kcal / mole (1 cal = 4.18 J)]
n
The energies are negative since the energy of the electron in the atom is less than the energy of a free
electron, i.e. the electron is at infinite distance from the nucleus which is taken as zero. The lowest energy
level of the atom corresponds to n = 1, and as the quantum number increases, E becomes less negative.
When n = , E = 0, which corresponds to an ionized atom, i.e. the electron and nucleus are infinitely
separated.
H ⎯→ H++ e– (ionization).

CALCULATION OF VELOCITY
We know that
nh nh
mvr = ;v =
2π 2πmr
By substituting for r we are getting
2πKZe2
v=
nh
where excepting n and Z all are constants

v = 2.18 108 cm/sec.

Some Formulae Related with Bohr Model

mv 2 Ze2
(i) = 2
r r
nh
(ii) mvr =
2
2 Ze2
(iii) v= = 2.188  108 Z/n cm/sec
nh
n2 h2  n2 
(iv) r= = 0.529   Å
4 2 mZe2 Z
−2 2 mZ 2e4 PE
(v) ET = 2 2
= −KE=
nh 2
−Ze2
(vi) ET = = K.E. + P.E.
2r
−2 2 mK 2 Z 2e4
(vii) ET =
n2 h2

110
ATOMIC STRUCTURE CHEMISTRY

–19
Z2 Z2 –12
Z2
(viii) ET = –21.8×10 J/atom = – 13.6 2 eV/atom = 21.8 × 10 erg = – 1312 kJ/mol
n2 n n2
(ix) E1<E2<E3< ...... <E (E = 0)
(x) (E2– E1) > (E3–E2) > (E4–E3) > .......,
(xi) For H-atom: E1 = –13.6 eV
E2 = – 3.4 eV
E3 = –1.5 eV
E4 = –0.85 eV
E5 = –0.54 eV
2
(xii) I.E. = 13.6 Z eV/atom
0.657 1016 z 2

(xiii) Number of revolution per sec by an e =
n3
1.52 10−16 n3
(xiv) Time taken for one revolution =
z2
Example 22: For which of the following species, Bohr theory doesn’t apply
(A) H (B) He+
2+
(C) Li (D) Na+
Solution:(D) Bohr theory is not applicable to multi electron species.
Example 23: If the radius of 2nd Bohr orbit of hydrogen atom is r2. The radius of third Bohr orbit will be
4
(A) r2 (B) 4r2
9
9
(C) r2 (D) 9r2
4
n2 h2
Solution:(C) r=
4 2 mZe2
r2 22 9
 =  r3 = r2
r3 32 4
Example 24: Number of waves made by a Bohr electron in one complete revolution in 3rd orbit is
(A) 2 (B) 3
(C) 4 (D) 1
Solution:(B) Circumference of 3rd orbit = 2r3
According to Bohr, angular momentum of electron in 3rd orbit is
h h 2 r3
mvr3 = 3 or = by De-Broglie equation
2 mv 3
h
=
mv
2 r3
 =  2r3 = 3
3
i.e. circumference of 3rd orbit is three times the wavelength of electron or number of waves made
by Bohr electron in one complete revolution in 3rd orbit is three.

111
CHEMISTRY ATOMIC STRUCTURE

RH
Example 25: The degeneracy of the level of hydrogen atom that has energy − is
16
(A) 16 (B) 4
(C) 2 (D) 1
RH
Solution: En = −
n2
R R
 − H2 = − H i.e. for 4th sub-shell
n 16
1 2 3

n= 4
l = 0

m=0 -1 0 +1 -2 –1 0 +1 +2 –3 –2 –1 0 +1 +2 +3
one s three p five d Seven f
i.e. 1 + 3 + 5 + 7 = 16
 degeneracy is 16
Merits of Bohr’s Theory
• The experimental value of radii and energies in hydrogen atom are in good agreement with that
calculated on the basis of Bohr’s theory.
• Bohr’s concept of stationary state of electron explains the emission and absorption spectra of hydrogen
like atoms.
• The experimental values of the spectral lines of the hydrogen spectrum are in close agreement with that
calculated by Bohr’s theory.
Limitations of Bohr’s Theory
• It does not explain the spectra of atoms having more than one electron.
• Bohr’s atomic model failed to account for the effect of magnetic field (Zeeman effect) or electric field
(Stark effect) on the spectra of atoms or ions. It was observed that when the source of a spectrum is
placed in a strong magnetic or electric field, each spectral line further splits into a number of lines. This
observation could not be explained on the basis of Bohr’s model.
• de Broglie suggested that electrons like light have dual character. It has particle and wave character. Bohr
treated the electron only as particle.
• Another objection to Bohr’s theory came from Heisenberg’s Uncertainty Principle. According to this
principle “It is impossible to determine simultaneously the exact position and momentum of a small
moving particle like an electron”. The postulate of Bohr that electrons revolve in well defined orbits
around the nucleus with well defined velocities is thus not tenable.

SPECTRUM
Spectrum is the impression obtained on the photographic plate when radiations of different wavelengths are passed
through prism. An instrument used to separate the radiation of different wavelengths (or frequencies) is called
spectroscope or a spectrograph. The branch or science dealing with the study of spectra is called spectroscopy.
1. Emission spectrum:
When the radiation emitted from some source e.g. from the sun or by passing electric discharge through a
gas at low pressure or by heating some substance to high temperature etc, is passed directly through the
prism and then received on the photographic plate, the spectrum obtained is called ‘Emission spectrum’.
Depending upon the source of radiation, the emission spectra are mainly of two type:

112
ATOMIC STRUCTURE CHEMISTRY

(a) Continuous spectrum:


When white light from any source such as sun, a bulb or any hot glowing body is analysed by passing
through a prism it is observed that it splits up into seven different wide band of colours from violet to red.
These colours are so continuous that each of them merges into the next. Hence the spectrum is called
continuous spectrum.

(b) Discrete spectra: It is of two types


(i) Band spectrum

Dark space

Band

Band spectrum contains colourful continuous bands sepearted by some dark space.
Generally molecular spectrum are band spectrum
(ii) Line Spectrum:

This is the ordered arrangement of lines of particular wavelength separated by dark space eg. hydrogen
spectrum.
Line spectrum can be obtained from atoms.
2. Absorption spectra:
When white light from any source is first passed through the solution or vapours of a chemical substance
and then analysed by the spectroscope, it is observed that some dark lines are obtained in the continuous
spectrum. These dark lines are supposed to result from the fact that when white light (containing
radiations of many wavelengths) is passed through the chemical substance, radiations of certain
wavelengths are absorbed, depending upon the nature of the element.

113
CHEMISTRY ATOMIC STRUCTURE

HYDROGEN SPECTRUM

When hydrogen gas at low pressure is taken in the discharge tube and the light emitted on passing electric
discharge is examined with a spectroscope, the spectrum obtained is called the emission spectrum of
hydrogen.
Line spectrum of hydrogen is observed due to excitation or de-excitation of electron from one stationary orbit
to another stationary orbit

INTERPRETATION OF HYDROGEN SPECTRUM

According to the Bohr’s theory, electron neither emits nor absorbs energy as long as it stays in a particular
orbit. However, when an atom is subjected to electric discharge or high temperature, and electron in the atom
may jump from the normal energy level, i.e. ground state to some higher energy level i.e. exited state. Since
the life time of the electron in excited state is short, it returns to the ground state in one or more jumps.
During each jump, energy is emitted in the form of a photon of light of definite wavelength or frequency. The
frequency of the photon of light thus emitted depends upon the energy difference of the two energy levels
concerned (n1, n2) and is given by
-2π2mZ2e4K 2 1 1
h = E2 – E1 =  2 − 2
h2  n2 n1 

2π2 mZ2e4 K 2 1 1
=  2 − 2
h3  n1 n2 

114
ATOMIC STRUCTURE CHEMISTRY

The frequencies of the spectral lines calculated with the help of above equation are found to be in good
agreement with the experimental values. Thus, Bohr’s theory elegantly explains the line spectrum of
hydrogen and hydrogenic species.
Bohr had calculated Rydberg constant from the above equation.
c 2π2 mZ2e4 K 2  1 1 
= =  2 − 2
 h3  n1 n2 
1 2π 2 mZ2e4 K 2 1 1
==  2 − 2
 h 3c  n1 n2 
2π 2 me4 K 2
where = 1.097  107m–1 or 109678 cm–1, i.e. Rydberg constant (R)
h 3c
1 1 1
 = = RZ2  2 - 2 
  n1 n2 
Further application of Bohr’s work was made, to other one electron species (Hydrogenic ion) such as He+ and
Li2+. In each case of this kind, Bohr’s prediction of the spectrum was correct.
Various Series of spectrum lines:
Series of Lines Transition Spectrum Zone Wave Length
1. Lyman n2 = 2,3,4, ... to n1 = 1 Ultraviolet < 3800 Å
2. Balmer n2 = 3,4,5 ... to n1 = 2 Visible 3800-7800 Å
3. Paschen n2 = 4,5,6 ... to n1 = 3 Infrared > 7800 Å
4. Bracket n2 = 5,6, 7... to n1 = 4 " "
5. Pfund n2 = 6, 7, 8 ... to n1= 5 " "
6. Humphury n2 = 7, 8, 9 ... to n1 = 6 " "

(ii)Wave number ( ) and wavelength () of spectral lines:

1 1 1
= = RH Z 2  2 − 2 
  n1 n2 
2 2 me 4
Here, RH = (RH = Rydberg constant = 109678 cm–1)
ch3
n(n − 1)
(iii) Total no. of spectrum line =
2 (where n is the highest stationary state)
Total Spectral Lines:
n(n − 1)
if multiple electrons and n – 1 if a single electron is present
2
∆n(∆n+1)
(iv) When electrons transition from n2 to n1, total number of spectral lines =
2

Example 26: If electron make transition from 7thexcited state to 2ndorbit in H atom sample find the max.
number of spectral lines observed.
 6 +1  7
Solution: n = 8 – 2 = 6 spectral lines = 6   = 6  = 21
 2  2

115
CHEMISTRY ATOMIC STRUCTURE

Example 27: An electron in isolated hydrogen atom is in 4thexcited state, then, upon de excitation:
(A) the maximum number of possible photons will be 10
(B) the maximum number of possible photons will be 6
(C) it can emit two photons in ultraviolet region
(D) if an infrared photon is generated, then a visible photon may follow this infrared photon.
Solution: (A) and (D)
Example 28: In a hydrogen like sample, ions are in a particular excited state, if electrons make transition upto
1st excited state, then it produces maximum 15 different types of spectral lines then electrons
were in
(A) 5th state (B) 6th state
(C) 7th state (D) 8th state
n ( n + 1)
Solution: = 15  n = 5  n − 2 = 5  n = 7
2
Example 29: (a) What are the frequency and wavelength of a photon emitted during a transition from n = 5 to
n = 2 in H atom?
(b) In which region of EM spectrum will this radiation lie?
 1 1 
Solution: According to Rydberg formula,  = R  2
− 2
 n1 n2 
Here, R = 109, 677cm−1 , nl 2 = 5, n1 = 2
 1 1  −1 21 −1
  = 109,677  − 2  cm = 109,677  cm = 23032.2cm−1
2 5 
2
100
1 1
= = = 434 10−7 cm = 434 10−9 m = 434nm
 23032.2cm −1

3 108 ms −1
c
v= = = 6.911015 s −1
 434 10 m−9

(b) The wavelength, as calculated above, lies in the visible region. Otherwise too, as the jump is on
the 2nd orbit, the line will belong to Balmer series and hence lie in thevisible region.
Alternatively, as discussed later under Bohr’s model, on page 2/30 energy of an electron in the
−21.8 10−19
nth shell is given by En = J
n2
Energy released when electron undergoes transition from n = 5 to n = 2, will be
−21.8 10−19  21.8 10−19  −19  1 1
E = E5 − E2 = −−  J = 21.8 10  −  J
 25 4 
2 2
5  2 
21
= 21.8 10−19  J = 4.58 10−19 J
100
But E = hv
E 4.58 10−19 J
 v= = −34
= 6.911014 s-1 or E
h 6.626 10 Js
c 3.0 108 ms −1
= = = 434 10−9 m = 434nm
v 6.911014 s −1

116
ATOMIC STRUCTURE CHEMISTRY

Example30: Wavelength of 1stline in Balmar series in 656 nm. Cal. wavelength of 2nd line and limiting line
in Balmar series.
13  1 1 
Solution: According to Rydberg formula, v = = R 2 − 2 
  n1 n2 
For the Balmer series, n1 = 2 and for the Ist line, n2 = 3
1 1 1 1 1 5 5R
 = R 2 − 2  = R −  = R = …(i)
656 2 3  4 9 36 36
For the second line, n1 = 2, n2 = 4.
1 1 1 1 1  3 3R
 = R 2 − 2  = R −  = R  …(ii)
 2 4   4 16  16 16
 5 16
Dividing eqn. (i) by eqn. (ii), we get S =  or  = 485.9nm
656 36 3
1 1 1  R
For the limiting line, n1 = 2, n2 =   = R 2 − 2  = …(iii)
 2   4
 5
Dividing eqn. (i) by eqn.(ii), we get =  4 or  = 364.4nm
656 36
Alternatively, first calculate R from eqn. (i) and substitute ineqn. (ii) and (iii).
Example 31: What transition in H spectrum would have same λ as Balmer transition n = 4 to n = 2 of He+
spectrum? [IIT-1993]
+
Solution: For He ion, we have
1 1 1
= Z 2 RH  2 − 2 
  n1 n2 
1 1 3
= ( 2 ) RH  2 − 2  = RH
2
…(i)
2 4  4
1  1 1 
Now for hydrogen atom = RH  2 − 2  …(ii)
  n1 n2 
Equating equations (i) and (ii), we get
1 1 3
2
− 2=
n1 n2 4
Obviously, n1 = 1 and n2 = 2
Hence, the transition n = 2 to n = 1 in hydrogen atom will have the same wavelength as the
transition, n = 4 to n = 2 in He+ species.
Example 32: Calculate the wavelength for the emission transition if it starts from the orbit having radius
1.3225 nm and ends at 211.6 pm. Name the series to which this transition belongs and the region
of the spectrum.
2
0.529 52.9n
Solution: Radius of nth orbit of H-like particles = = Å = pm
Z Z
52.9n12
r1 = 1.3225nm = 1322.5pm =
Z

117
CHEMISTRY ATOMIC STRUCTURE

52.9n22 r1 1322.5 n12 n12 n


r2 = 211.6pm =  = = 2 or 2 = 6.25 or 1 = 2.5
Z r2 211.6 n2 n2 n2
 If n2 = 5, n1 = 5. thus, the transition is from 5th orbit to 2nd orbit. It belongs to Blamer series.
1 1 1
v = 1.097 107 m−1  2 − 2  = 1.097  107 m−1
2 5  100
1 100
or = = m = 434 10=9 m = 434nm
v 1.097  21107
Example 33: (i) Energy associated with 1st orbit in H atom is –2.18 × 10–18 J atom–1. What is the energy
associated with fifth orbit?
(ii) Calculate the radius of Bohr’s fifth orbit for hydrogen atom.
2.18 10−18 2.18 10−18
Solution: (i) En = 2
J  E5 = − 2
= −8.72 10−20 J
n 5
(ii) For H− atom, rn = 0.529  n Å
2
r5 = 0.529  52 = 13.225
Example 34: Electron energy in H atom is given by En = (–2.18 × 10–18)/n2 J. Cal. energy required to remove
an electron completely from the n = 2 orbit. What is the longest wavelength of light in cm that
can be used to cause this transition?
 2.18 10−18 J atom −1  −19 −1
Solution: E = E − E2 = 0 −  − 2  = 5.45  10 J atom
 2 

hc ( 6.626 10 Js )  (3 10 ms )


−34 8 −1
c
E = hv = h or  = =
 E 5.45 10−19 J
= 3.647 10−7 m = 3.647 105 cm
Example 35: Estimate dif. in energy b/w 1st & 2nd Bohr’s orbit for H. At what min. atomic no., transition
from n = 2 to n = 1 energy level would result in X-rays emission with  = 3.0 × 10–8 m? Which H
like species does this correspond to? [IIT-1993]
 1 1 
Solution: ΔE = RhcZ 2  2 − 2 
 n1 n2 
−1
Here, R = 1.0967 10 m
7

h = 6.626 10−34 Jsec,c = 3 108 m/sec


n1 = 1, n2 = 2 and for H-atom, Z = 1
1 1 
E2 − E1 = 1.0967 107  6.626 10−34  3 108  − 
1 4 
3 −19
E = 1.0967  6.626  3  10−19 J = 16.3512 10 J
4
16.3512 10−19 J
= eV = 10.22eV
1.6 10−19
hc  1 1 
E = = RhcZ 2  2 − 2 
  n1 n2 

118
ATOMIC STRUCTURE CHEMISTRY

1 1 1  3
= RZ 2  −  = RZ 2 
 1 4  4
Given,  = 3 10−8 m
1 3
 −8
= 1.0967  Z 2  107
3 10 m 4
108  4 40
 Z2 = = 4
3  3 1.0967 107
9 1.0967
 Z =2
So it correspondence to He+ which has 1 electron like hydrogen.
Example 36: The binding energy of an electron in the ground state of the He atom is equal to
24.6 eV. The energy required to remove both the electrons from the atom will be
(A) 59 eV (B) 81 eV
(C) 79 eV (D) None of these
z2 22
Solution:(C) Ionization energy of He = 2 13.6 = 2 13.6 = 54.4 eV
+
n 1
Energy required to remove both the electrons
= binding energy + ionization energy = 24. 6 + 54.4= 79 eV
Example 37: O2 undergoes photochemical dissociation into one normal oxygen and one excited oxygen atom,
1.967 eV more energetic than normal. The dissociation of O2 into two normal atoms of oxygen
atoms requires 498 KJ mole–1. What is the maximum wavelength effective for photochemical
dissociation of O2?
Solution: O2⎯→ ON + Oexcited ;
O2⎯→ ON + ON
498 103
E = 498 10 J / mole =
3
J per molecule = 8.268 10–19 J
6.023 1023
Energy required for excitation = 1.967 eV = 3.146 10–19J
Total energy required for photochemical dissociation ofO2 = 8.268 10–19 + 3.146 10–19
hc
= 11.414  10–19 J; = 11.414 10–19 J

−34
6.626 10  3 108
= = 1.7415 10–7 m = 1741.5 Å
11.414 10−19
Example 38: Show that frequencies of emitted photons are additive but their wavelengths are not.
Solution:
n=3

E2 → 3

n=2 X
E

E1 → 3 E1 → 2

n=1

119
CHEMISTRY ATOMIC STRUCTURE

 E1 → 3 = E1 → 2 + E2 →3 ;

 h1 → 3 = h1 → 2 + h2 → 3


 1 → 3 = 1 → 2 + 2 → 3;i.e. frequencies like energies are additive, on the other hand
 E1 → 3 = E1 → 2 + E2 → 3

hc hc hc 1 1 1
 = + ; = +
1→3 1→2 2→3 1→3 1→2 2→3
1 2→3 + 1→2 1→2 2→3
 = ; 1→3 =
1→3 1→22→3 1→2 + 2→3
i.e. wavelengths are not additive.

DUAL NATURE & UNCERTAINTY PRINCIPLE


De-Broglie equation
The wavelength of the wave associated with any material particle was calculated by analogy with photon.
In case of photon, if it is assumed to have wave character, its energy is given by
E = h …(i) (according to the Planck’s quantum theory)
Where is the frequency of the wave and ‘h’ is Planck’s constant
If the photon is supposed to have particle character, its energy is given by
E = mc2 …(ii) (according to Einstein’s equation)
where ‘m’ is the mass of photon, ‘c’ is the velocity of light.
By equating (i) and (ii)
c c h
h= mc2; But = ; h = mc 2 (or) =
  mc
The above equation is applicable to material particle if the mass and velocity of photon is replaced by the
mass and velocity of material particle. Thus for any material particle like electron
h h
= (or) = ; where mv = p is the momentum of the particle.
mv p

DUAL CHARACTER
Particle and Wave Character of Matter and Radiation
In case of light some phenomenon like diffraction and interference can be explained on the basis of its wave
character. However, the certain other phenomenon such as black body radiation and photoelectric effect can
be explained only on the basis of its particle nature. Thus, light is said to have a dual character. Such studies
on light were made by Einstein in 1905.
Louis de Broglie, in 1924 extended the idea of photons to material particles such as electron and he proposed
that matter also has a dual character-as wave and as particle.
Derivation of de Broglie Equation
The wavelength of the wave associated with any material particle was calculated by analogy with photon.

120
ATOMIC STRUCTURE CHEMISTRY

In case of photon, if it is assumed to have wave character, its energy is given by


E = h (According to the Planck’s quantum theory) ….. (i)
Where the frequency of the wave and ‘h’ is is Planck’s constant
If the photon is supposed to have particle character, its energy is given by
E = mc2 (According to Einstein’s equation) ….. (ii)
Where ‘m’ is the mass of photon, ‘c’ is the velocity of light.
By equating (i) and (ii)
h = mc2
But  = c/
c
h = mc2

(or)  = h /mc
The above equation is applicable to material particle if the mass and velocity of photon is replaced by the
mass and velocity of material particle. Thus, for any material particle like electron.
h
 = h/mv (or) =
p
Where mv = p is the momentum of the particle.
Derivation of Angular Momentum from de Broglie Equation
According to Bohr’s model, the electron revolves around the nucleus in circular orbits. According to de
Broglie concept, the electron is not only a particle but has a wave character also.
If the wave is completely in phase, the circumference of the orbit must be
equal to an integral multiple of wave length ()
Therefore, 2r = n
where ‘n’ is an integer and ‘r’ is the radius of the orbit
But  = h/mv
 2r = nh /mv
(or) mvr = n h/2
which is Bohr’s postulate of angular momentum, where ‘n’ is the principal quantum number.
“Thus, the number of waves an electron makes in a particular Bohr orbit in one complete revolution is equal
to the principal quantum number of the orbit”.
Alternatively
2r 2r 2mvr
Number of waves ‘n’ = = =
 h mv h
where v and r are the velocity of electron and radius of that particular Bohr orbit in which number of waves
are to be calculated, respectively.
The electron is revolving around the nucleus in a circular orbit. How many revolutions it can
make in one second
Let the velocity of electron be v m/sec. The distance it has to travel for one revolution 2r, (i.e., the
circumference of the circle).
v
Thus, the number of revolutions per second is =
2r

121
CHEMISTRY ATOMIC STRUCTURE

Common unit of energy is electron volt which is amount of energy given when an electron is accelerated by a
potential of exactly 1 volt. This energy equals the product of voltage and charge. Since in SI units coulombs
 volts = Joules, 1 eV numerically equals the electronic charge except that joule replaces coulombs.
Relation Between  and Kinetic Energy
h
λ= (from de Broglie equation)
mv
Squaring both side
h2
2 =
m2 v 2
h2
 2 =
1
2m × × mv2
2
2
h
 2 =
2m × K.E.
h
=
2K.E × m

Example 39: Two particles A and B are in motion. If the wavelength associated with particle A is
5 10–8 m, calculate the wavelength associated with particle B if its momentum is half of A.
Solution: According to de Broglie equation
h h
A = and B =
pA pB
A p
= B
B pA
But pB = ½ pA (given)
A 1/ 2p A
= =½
B pA
B = 2A = 2  510–8 m = 10–7 m
Example 40: Calculate the de Broglie wavelength of a ball of mass 0.1 kg moving with a speed of 60 ms–1.
h 6.6×10−34
Solution: = =
mv 0.1× 60
 = 1.1  10–34 m
This is apparent that this wavelength is too small for ordinary observation.
Although the de Broglie equation is applicable to all material objects but it has significance only in case of
microscopic particles.
Since, we come across macroscopic objects in our everyday life, de Broglie relationship has no significance
in everyday life.
[Distinction between the wave- particle nature of a photon and the particle- wave nature of a sub atomic
particle]

Photon Sub Atomic Particle


1
1. Energy = h Energy = mv2
2
c h
2. Wavelength = Wavelength =
 mv

122
ATOMIC STRUCTURE CHEMISTRY

Note: We should never interchange any of the above.

• What can you say about wave nature of the fast train?
Although the de Broglie equation is applicable to all material objects but it has significance only in case
of microscopic particles. Since, we come across macroscopic objects in our everyday life, de Broglie
relationship has no significance in everyday life.
h h
(i) = =
mv p
1 2
(ii) mv = eV (for electron)
2
h
(iii) =
2emV
12.25
(iv) = (Here  is in Å, V is in volt) (for electron)
V
h
(v) =
2m K.E.
12.25
(vi) = (for electron) (Here  is in Å and K.E. is in eV)
K.E.

Example 41: An electron is moving with a kinetic energy of 4.55 10–25 J. What will be de Broglie wave
length for this electron?
(A) 5.28 10–7 m (B) 7.28 10–7 m
(C) 2 10–10 m (D) 3 10–5 m
1 2 2  4.55 10−25
Solution:(B) KE = mv = 4.55 10–25; v 2 = = 1106 ; v = 103 m/s
2 9.110−31
h 6.626 10−34
De Broglie wave length  = = = 7.28 10–7 m
mv 9.110−31 103
MATTER WAVES
 Light was thought to be a wave, but Einstein showed that it also acts particle-like.Electrons were
“known” to be particles mass & charge.French physicist: What if electrons behaved as waves also
 If there is a particle of momentum p, its motion is associated with a wave of wavelength: λ = h/mv
 De Broglie λ of a moving cow would be trillions times smaller than atomic dimensions & far too small to
be detected.
 De Broglie λ of a C60 molecule (buckyball) travelling at a few m/s is approx. the size of molecule itself
(about 1nm).
 De Broglie λ of an electron travelling at a few m/s has a de Broglie λ equivalent to width of a human hair (a
fraction of a millimetre). That is large enough for the quantum wave to actually show up in experiments.
 Bohr had just invented his “Quantum Hypothesis”, because it explained the Hydrogen-spectrum. He gave
no explanation, as to why angular momentum should be “quantized”. De Broglie proved it. 2πr = nλ
where λ = h/mv

123
CHEMISTRY ATOMIC STRUCTURE

 Planck’s constant is so small that we don’t observe the wave behaviour of ordinary objects – their de
Broglie wavelengths could be many orders of magnitude smaller than the size of a nucleus!
 De Broglie’s hypothesis was confirmed independently by Clinton Davisson (USA) and George Thomson
(UK) in 1927
Example 42: If the Planck’s constant h = 6.6  10–34 Js, the de-Broglie wavelength of a particle having
momentum of 3.3 10–24kg ms–1 will be -
(A) 0.002 Å (B) 0.02 Å
(C) 0.2 Å (D) 2Å
h
Solution:(D) =
mv
Example 43: K.E. of the electron is 4.55  10–25 J. Its de Broglie wave length is -
(A) 4700 Å (B) 8300Å
(C) 7200Å (D) 7400Å
h
Solution:(C) =
2mKE
Example 44: For particles having same kinetic energy, the de Broglie wavelength is -
(A) Directly proportional to its velocity (B) Inversely proportional to its velocity
(C) Independent of velocity and mass (D) Unpredictable.
h 1 2
Solution:(A) =  KE = mv
mv 2
2KE
mv =
v
h
 =
2 KE
v
 v 
 = h 
 2 KE 
Example 45: Velocity of helium atom at 300K is 2.40 × 102 meter per sec. What is its wave length? (mass
number of helium is 4) -
(A) 0.416 nm (B) 0.83 nm
(C) 803 Å (D) 8000Å
h
Solution: =
mv
4.0  10−3
mass of helium = kg. and
6.023 1023
h = 6.62  10–34
6.023 1023 1
 = 6.62  10–34 −3
 = 0.416  10–9 meter
4.0 10 2.4 102
 = 0.416 nm

124
ATOMIC STRUCTURE CHEMISTRY

HEISENBERG’S UNCERTAINTY PRINCIPLE


 At quantum level, one can’t measure any property of a particle without interacting with it in some way
 This introduces an unavoidable uncertainty into the result. One can never measure all the properties exactly
 Shine light on electron and detect reflected light using a microscope
 Minimum uncertainty in position is given by the wavelength of the light so to determine the position
accurately, it is necessary to use light with a short wavelength.
 By E = h c/λ, a photon with a short wavelength has a large energy. Thus, it would impart a large ‘kick’ to
the electron
 To determine its momentum accurately, electron must only be given a small kick i.e. using light of long wavelength!
 Use light with short wavelength: accurate measurement of position but not momentum
 Use light with long wavelength: accurate measurement of momentum but not position.
 More accurate the position (i.e., smaller Δx), less accurately you know the momentum (i.e., larger Δp);
and vice versa

Note:
h & h
• ( x )( p )  ( x )( v ) 
4 4 m
• h (For energy and time) & h
.t  . 
4 4

Example 46: Why electron cannot exist inside the nucleus according to Heisenberg’s uncertainty principle?
Solution: Diameter of the atomic nucleus is of the order of 10–15m
The maximum uncertainty in the position of electron is 10–15 m.
Mass of electron = 9.1 10–31 kg.
h
x. p =
4
x  (m.v) = h/4
h 1 6.63  10−34 1
v =  =  −15
4 x.m
4
22 10  9.1 10−31
7
v = 5.80  1010 ms–1
This value is much higher than the velocity of light and hence not possible.
Example 47: If uncertainty in position and momentum are equal, the uncertainty in velocity is -
h 1 h
(A) (B)
2 2m 
h
(C) (D) None

125
CHEMISTRY ATOMIC STRUCTURE

h h
 x.m  v = (m  v) =
2
Solution:(B) or
4 4
h h 1 h
mv = or v  =
4 4 m 2
2m 
Example 48: The uncertainty in position and velocity of a particle are 10–10 m and 5.27 x 10–24ms–1
respectively. Calculate the mass of the particle is (h = 6.625 x 10–34 J-s)
(A) 0.099 kg (B) 0.99 g
(C) 0.92 kg (D) None
Solution:(A) According to Heisenberg’s uncertainty principle,
h
m=
4  x.  v
6.625 10−34
or = = 0.099 kg.
4  3.143 10−10  5.27 10−24
Example 49: On the basis of Heisenberg’s uncertainty principle, show that the electron cannot exist within the
nucleus.
Solution: Diameter of the nucleus is of the order of 10–15m. The maximum uncertainty in the position of
electron within the nucleus will be 10–15 m. Now, from Heisenberg’s uncertainty principle.
h h
x.mv  or vmin =
4 4 mx
SOMMERFELD MODEL OF ATOM
 High resolving power spectroscopes revealed some very fine spectral lines which Bohr was not able to explain.
 Acc. to Sommerfeld, stationary orbits in which e- are revolving around nucleus are not circular but
elliptical in shape.
 Electron revolves in elliptical path with nucleus at one of its foci (Major and a minor axis).
 Lengths of 2 axis approach to equal value and ultimately become equal i.e. path become circular.
 k is known as azimuthal quantum no. n/k = length of major axis/length of minor axis
 With increase in value of k, path becomes more and more elliptical and eccentric. When k = n, path
becomes circular.
 When transition of e- from higher to lower occurs, it will be diff. from Bohr model as there are many values of k.
 In this way Sommerfeld was able to explain the reason behind those fine spectral lines.
 Even frequencies of some of those fine spectral lines came to be in well agreement with frequencies by
Sommerfeld.

126
ATOMIC STRUCTURE CHEMISTRY

Zeeman Effect
He showed that in presence of strong magnetic field atom shows additional line in spectrum. This is because
elliptical orbits can only take up certain orientations w.r.t. external field, rather than processing freely. Orientation
is associated to quantum no. known as magnetic quantum no. m & it can have (2k -1) values for a given k.

QUANTUM MODEL
Quantum mechanics is a theoretical science that deals with the study with motion of microscopic object(s)
that have both observable wave like and particle like properties. It specifies the motion that these object obey.
When classical mechanics is applied to macroscopic object (for which, wave like properties is insignificant)
the result is the same as concluded those from classical mechanics.
Quantum mechanics depend on:
1. Planck’s theory of quantisation
2. de-broglie equation
3. Heisenberg uncertainty principle
The fundamental equation of quantum mechanics was developed by Schrodinger and it won him noble prize
for physics in 1933. For a system (such as atom or a molecule whose energy does not vary with time) the
Schrodinger equation is written as
Ĥ = E where H is mathematical operator called Hamiltonian operator.
This equation [d 2nd order differential] can also be written as
d 2 d 2 d 2 8 2 m
+ + + 2  E − V  = 0
dx 2 dy 2 dz 2 h
where x, y and z are co-ordinate axis.
m = mass of e–; h = Planck’s constant;  = wave function; E = total electric energy; V = total potential energy
8 2 m
or 2 + ( E − V ) = 0
h2
(Schrodinger wave equation)
 Wave function Ψ is a function of coordinates of e- & has no physical significance. Contains all info about
e-& quantum mechanics helps to extract that.
 Ψ2 represents the probability density of finding the electron around the nucleus. This was explained by
Max Born.
 Initially, electrons were thought to travel in orbits (2D, travels around nucleus at fixed distance in a
circular path)
 Quantum theory describes electrons as existing in orbitals (3D region, distance from nucleus varies, no
fixed path)
 Electron's movement cannot be known precisely.We can only map the probability of finding the
electron at various locations outside the nucleus which is known as electron cloud.
 Probability map is called an orbital.It is calculated to confine max. probability of finding an electron. (up
to 90%).
 Energy of the electron is quantized into sublevels.
 Electrons have discrete energies, not because they are in shells but because they can only have certain
wavelengths
 Line spectra are not due to electrons jumping from shell to shell (as in Bohr’s model). Instead they’re due
to electrons transforming from one wavelength (waveform) to another

127
CHEMISTRY ATOMIC STRUCTURE

Difference between Orbit & Orbital

Orbit Orbital
1. Well defined circular path around nucleus 3D space around nucleus, probability of finding e-
is max.
2. Represents planar motion of e- around nucleus Represents 3D motion of an electron around nucleus
3. Not in accordance with wave nature & In accordance with both
Heisenberg uncertainty principle
4. All orbits are circular and disc like Diff. orbitals have diff. shapes.
2
5. Max. no. of e- in any orbit = 2n Max. no. of e- in any orbital = 2

Basics about Quantum nos.


 Each electron is a wave that can be described by a series of “quantum numbers”
 Quantum numbers are quantizedvalues used to describe electrons in an atom
 An electron’s unique “fingerprint” that describes it position and behavior
 There are four quantum numbers: n, l, ml, s &combination of the first 3 defines an “orbital”
 Define energy, size, shape & orientation of orbital. Along with 4th quantum no., they store entire info
about the e-.
 Quantum theory of atom agrees completely with periodic table, which had been around for 30 years
and was developed without any knowledge of electron arrangements.

Principle Quantum Number (Bohr, 1913)


It tells the main shell in which the electron resides and the approximate distance of the electron from the
nucleus. It also tells the maximum number of electrons a shell can accommodate is 2n 2, where n is the
principal quantum number.
Shell K L M N O P Q
Principal quantum number (n) 1 2 3 4 5 6 7
Maximum number of electrons
that may be present 2 8 18 32 50 72 98

Azimuthal Quantum No. (Sommerfeld, 1915)


 (l) can be zero or any positive integer smaller than n.i.e.l = 0, 1, 2, …n-1 and indexes shape of orbital
s (sharp) sub-shell: l = 0p (principal) sub-shell: l = 1 d (diffused) sub-shell: l = 2
f (fundamental) sub-shell: l = 3
 If n can be thought of as shells, l can be thought of as “subshells” dividing each shell into subsections
(l = 0 → n - 1)
 Heisenberg Uncertainty Principle states that you cannot measure momentum & exact position of e-
at same time.
 What you can measure is the probability that an electron will be found in a certain area, called an orbital.
 Orbital angular momentum of anelectron is given by √𝑙(𝑙 + 1) h/2π where h/2π is also written as ħ
E.g. Orbital angular momentum of an electron is 4s orbital is 0 as l = 0

128
ATOMIC STRUCTURE CHEMISTRY

Example 50: For a p-electron, orbital angular moment is


(A) 2 (B) 
(C) 6 (D) 2
h
Solution: Orbital angular momentum L = ( + 1) where =
2
 L for p electron = 1(1 + 1) = 2

Magnetic Quantum No.(Sommerfeld and Debye, 1915)


 (ml) can have any positive or negative integral value b/w 0 &l. i.e. ml= 0, 1, 2,. l&indexes
orientation of orbital
 Allowed values of ml for that particular value of l determines no. of degenerate orbitals i.e. having
equivalent energy
 It also gives the quantized value of Z-component of orbital angular momentum (LZ = m h/2π)
 For p subshell, m = 0 (pz), m = +1 (px) &m = –1 (py). For d subshell, m = 1 (dxz), m = –1 (dyz), m = +2 (dx2-y2)

Spin Quantum Number (Pauli, 1925)


 ms or s = +1/2 or –1/2&indexes spin direction of an e- within an orbital
 Magnets could further split lines in line spectra & some elements exhibit paramagnetism i.e. attracted
by ext. magnet
h
Spin angular momentum = s ( s + 1)
2
Magnetic moment = n ( n + 2 ) B.M. where n= no. of unpaired electrons

Example 51: Magnetic moments of V (Z = 23), Cr (Z = 24) and Mn (Z = 25) are x, y, z. Hence
(A) z y x (B) x = y = z
(C) x z y (D) xyz
Solution: (C) Magnetic moments = n(n + 2) B.M . where n is the number of unpaired electron
V ( Z = 23) (Ar) 3d34s2 n = 3, 15 BM = x
Cr (Z = 24) (Ar) 3d5 4s1 n = 6, 48 BM = y
Mn (Z = 25) (Ar) 3d5 4s2 n = 5, 35 BM = z
Example 52: The quantum number not obtained from the Schrödinger’s wave equation is
(A) n (B) l
(C) m (D) s
Solution:(D) n, l and m quantum numbers can be obtained from Schrodinger equation. s is obtained from
spectral evidence.

QUANTUM ARITHMETIC
Sub shell Value of l Value of m No. of orbitals Max. no. of e–
s 0 0 1 2
p 1 0, ±1 3 6
d 2 0, ±1, ±2 5 10
f 3 0, ±1, ±2, ±3 7 14

129
CHEMISTRY ATOMIC STRUCTURE

SHAPES AND SIZE OF ORBITALS


An orbital is the region of space around the nucleus within which the probability of finding an electron of
given energy is maximum (90–95%). The shape of this region (electron cloud) gives the shape of the orbital.
It is basically determined by the azimuthal quantum number , while the orientation of orbital depends on the
magnetic quantum number (m). Let us now see the shapes of orbitals in the various subshells.
s–orbitals: These orbitals are spherical and symmetrical y

about the nucleus. The probability of finding the


electron is maximum near the nucleus and keep on
decreasing as the distance from the nucleus increases. 1s 2s
nucleus x
There is vacant space between two successive s–
orbitals known as radial node. But there is no radial
node for 1s orbital since it is starting from the nucleus. Z radial node

The size of the orbital depends upon the value of principal quantum number(n). Greater the value of n, larger
is the size of the orbital. Therefore, 2s–orbital is larger than 1s orbital but both of them are non-directional
and spherically symmetrical in shape.
p–orbitals (l = 1): The probability of finding the p–electron is maximum in two lobes on the opposite sides
of the nucleus. This gives rise to a dumb–bell shape for the p–orbital. For p–orbital l = 1. Hence, m = –1, 0,
+1. Thus, p–orbital have three different orientations. These are designated as p x, py and pz depending upon
whether the density of electron is maximum along the x, y and z axis respectively. As they are not spherically
symmetrical, they have directional character. The two lobes of p–orbitals are separated by a nodal plane,
where the probability of finding electron is zero.
Y

X py pz
px
Z

The three p-orbitals belonging to a particular energy shell have equal energies and are called degenerate orbitals.
d–orbitals (l = 2): For d–orbitals, l = 2. Hence, m=–2,–1,0,+1,+2. Thus, there are 5d-orbitals. They have
relatively complex geometry. Out of the five orbitals, the three (dxy, dyz, dzx) project in between the axis and
the other two dz and dx − y lie along the axis.
2 2 2

Z X

Y
d z2
d x2 − y 2

Dough–nut shape or Baby soother shape Clover leaf shape


X X Y

Y Z Z

dxz dyz
dxy

130
ATOMIC STRUCTURE CHEMISTRY

Example 53: Sub-shell that comes after f is called g. No. of g-orbitals in g-sub-shell and no. of orbitals in
principal orbital resp are-
Solution: = 4. No. of Orbitals = 9.
Example 54: The maximum probability of finding electron in the dxy orbital is -
(A) Along x-axis (B) Along y-axis
(C) At an angle of 45º from x & y-axis (D) At an angle of 90º from x & y-axis
Solution: (C)
Example 55: Which of the following orbitals have no spherical nodes?
(A) 1s (B) 2s
(C) 2p (D) 3p
Solution: (AC)
Example 56: Which of the following sets of quantum number are incorrect?
(A) n = 3, l = 2, m = + 1, s = +1/2 (B) n = 3, l = 3, m = + 3, s = +1/2
(C) n = 4, l = 0, m = 0, s = –1/2 (D) n = 5, l = 2, m = + 4, s = –1/2
Solution: (B) and (D)
Example 57: Find the number of nodal planes in a p x orbital? [IIT-2000]
Solution: 1.
Example 58: Find number of radial nodal surface in 3s and 2p? [IIT-2005]
Solution: Use n − − 1;3 − 0 − 1 = 2 and 2 − 1 − 1 = 0

ELECTRON FILLING PRINCIPLES


The atom is built up by filling electrons in various orbitals according to the following rules.

Aufbau Principle:
This principle states that the electrons are added one by one to the various orbitals in order of their increasing
energy starting with the orbital of lowest energy. The increasing order of energy of various orbital is
1s,2s,2p,3s,3p,4s,3d,4p,5s,4d,5p,6s,4f,5d,6p,5f,6d,7p……………………
How to remember such a big sequence? To make it simple we are giving you the method to write the increasing
order of the orbitals. Starting from the top, the direction of the arrows gives the order of filling of orbitals.
1s

2s 2p

3s 3p 3d

4s 4p 4d 4f

5s 5p 5d

6s 6p

7s

Alternatively, the order of increasing energies of the various orbitals can be calculated on the basis of (n+l) rule.
The energy of an orbital depends upon the sum of values of the principal quantum number (n) and the
azimuthal quantum number (l). This is called (n+ l) rule. According to this rule,

131
CHEMISTRY ATOMIC STRUCTURE

“In neutral isolated atom, the lower the value of (n+ l) for an orbital, lower is its energy. However, if the two
different types of orbitals have the same value of (n+ l), the orbitals with lower value of n has lower energy’’.
Illustration of (n + l) rule
Type of orbitals Value of n Values of l Values of(n+ l) Relative energy
1s 1 0 1+0=1 Lowest energy
2s 2 0 2+0=2 Higher energy than 1s orbital
2p 2 1 2+1=3 2p orbital (n=2) have lower
3s 3 0 3+1=3 energy than 3s orbital (n=3)

(n + l) rule is applicable for multi electronic systems only. For uni-electronic system like H, order of energy
of orbitals is not “significantly influenced” by l. Now can you write the order of energy of orbitals for uni-
electronic system?

Pauli’s Exclusion principle


According to this principle, an orbital can contain a maximum number of two electrons and these two
electrons must be of opposite spin.
Two electrons in an orbital can be represented by  or 

Hund’s rule of maximum multiplicity


This rule deals with the filling of electrons in the equal energy (degenerate) orbitals of the same sub shell
(p,d and f). According to this rule,
“Electron pairing in p,d and f orbitals cannot occur until each orbital of a given subshell contains one
electron each or is singly occupied & that too with the same spin.
This is due to the fact that electrons being identical in charge, repel each other when present in the same
orbital. This repulsion can, however, be minimized if two electrons move as far apart as possible by
occupying different degenerate orbitals. All the electrons in a degenerate set of orbitals will have same spin.
Multiplicity is given by 2|S| + 1. Can you now comment why the rule is called Hund’s rule of maximum
multiplicity?

Exchange Energy (Half & fully filled stability)


Stabilizing effect arises whenever 2 or more e- with same spin are present in degenerate orbitals of a subshell.
These e- tend to exchange their positions & energy released due to this exchange is called exchange energy.
Electronic configuration of Cr (24):[Ar] 3d44s2 but actually it is [Ar]3d54s1

4 + 3 + 2 + 1 = 10
In the case of Cr, if the electronic configuration is 3d4, then the electrons can be exchanged in six ways only:

3 + 2 +1 = 6
Hence, the number of exchanges is maximised when Cr has half-filled d subshell.

132
ATOMIC STRUCTURE CHEMISTRY

Extra stability is due to


1. Relatively small shielding
2. Smaller coulombic repulsion energy
3. Larger exchange energy

Note:
eh
1. Magnetic Moment= n ( n + 2)B.M. ,1 B.M. = = 9.27 10−24 JT −1 ;
4 me
n = total no. of unpaired e–
2. Radial nodes or spherical nodes = n – l– 1
3. Planar / Angular nodes = l
4. Total nodes = n – 1
5. No. of e–s in any sub-shell = 2(2l +1)
6. No. of orbitals in any sub-shell = (2l +1)
h
7. Orbital angular momentum of e– = l ( l + 1) 
2
h
8. Spin angular momentum of e– =  s ( s + 1) 
2
9. No. of Max. e– in any shell = 2n2
10. Max. number of orbitals in any shell= n2
11. Max. number of sub-shell in any shell = n
12. Excited state of e– is always equals to (n–1)
13. According to sommerfield no. of elliptical orbit is equal to (n–1)
14. Value of l = 0 to (n – 1)
15. Penultimate shell = (n – 1)
n
16. Spin multiplicity = 2S + 1 (Here  S = and n = total no. of unpaired electrons)
2

Example 59: Magnetic moment of X3+ ion of 3d series is 35 B.M. What is atomic number of X3+?
Solution: n = 5  E.C.is 3d5 ; adding 3e we get  Ar  3d 6 4s 2 which gives Z = 26
Example 60: Calculate total spin, magnetic moment for the atoms having at. no. 7, 24 and 36.
Solution: The electronic configuration are
7N : 1s2, 2s2, 2p3 unpaired electron = 3
2 2 6 2 6 5 1
24Cr : 1s , 2s 2p , 3s 3p 3d , 4s unpaired electron = 6
36Kr : 1s2, 2s2 2p6, 3s2 3p6 3d10, 4s2 4p6 unpaired electron = 0
Total spin for an atom = ± 1/2 × no. of unpaired electron
For 7N, it is = ± 3/2
For 24Cr, it is = ± 3
For 36Kr, it is = 0
Also magnetic moment = n ( n + 2)
For 7N, it is = 15
For 24Cr, it is = 48
For 36Kr, it is = 0

133
CHEMISTRY ATOMIC STRUCTURE

Example 61: Write down the four quantum numbers for V and VI electrons of carbon atom.
Solution: 6C : 1s2, 2s3 2p2
1 1
V electron: n = 2 l = 1 m = – 1 or +1 s=+ or –
2 2
1 1
VI electron: n = 2 l=1 m=0 s = + or –
2 2
Example 62: The electronic configuration of an element is 1s2 2s2 2p6 3s2 3p6 3d5 4s1. This represents its
[IIT-2000]
(A)excited state (B)ground state
(C) cationic form (D) anionic form
Solution: (B)
Example 63: If N atom had electronic configuration 1s, it would have energy lower than that of normal ground
state configuration 1s2 2s2 2p3 because electrons would be closer to nucleus yet, 1s7 is not
observed because it violates- [IIT-2002]
(A)Heisenberg Uncertainty principle (B) Hund’s rule
(C) Pauli’s exclusion principle (D) None
Solution: (C)
Example 64: Max. no. of electrons that can have principal quantum no., n = 3 & spin quantum no., ms = – 1/2,
is. [IIT-2011]
Solution: 3s, 3p, 3d can have s = -1/2  no. of e = 9 .
Example 65: The orbital diagram in which the Auf-bau principle is violated - [IIT-1988]
2s 2p 2s 2p
(A) (B)
2s 2p 2s 2p
(C) (D)
Solution: (B)

h
Example 66: The orbital angular momentum for an electron revolving in an orbit is given by l ( l + 1). .
2
this momentum for an s-electron will be given by
h
(A) zero (B)
2
h 1 h
(C) 2. (D) + .
2 2 2
Solution: For s-electron, = 0
h
 orbital angular momentum 0 ( 0 + 1) =0
2

134
ATOMIC STRUCTURE CHEMISTRY

Example 67: The electrons identified by quantum numbers n and :


(a) n = 4, =1 (b) n = 4, = 0
(c) n = 3, =2 (d) n = 3, =1
Can be placed in order of increasing energy as:
(A) (c) < (d) < (b) < (a) (B) (d) < (b) < (c) < (a)
(C) (b) < (d) < (a) < (c) (D) (a) < (c) < (b) < (d)
Solution:(B) (a) 4p (b) 4s (c) 3d (d) 3p
According to Bohr Bury’s (n + ) rule, increasing order of energy (d) < (b) <(c) < (a).

Note: If the two orbitals have same value of (n + ) then the orbital with lower value of n will be filled first.

Example 68: Which of the following sets of quantum numbers is correct for an electron in 4f orbital?
(A) n = 4, = 3, m = +1, s = +1/2 (B) n = 4, = 4, m = - 4, s = -1/2
(C) n = 4, = 3, m = +4, s = +1/2 (D) n = 3, = 2, m = -2, s = +1/2
Solution: The possible quantum numbers for 4f electron are n = 4, = 3, m = -3, -2, -1, 0, 1, 2, 3 and
s = ½ of various possibilities only option (A) is possible.

Rules for finding group no.


1. If last shell contains 1 or 2 electrons, then group no. is 1 & 2 respectively
2. If last shell contains more than 2 electrons, then group no. = no. of e + 10
3. If electrons are present in (n – 1)d orbital & ns then group no. = Total electrons in (n– 1)d &ns
Note: For H & H-like species, all orbital of same shell have same energy:
1s < 2s = 2p < 3s = 3p = 3d < 4s = 4p = 4d = 4f

Example 69: H like species Li2+ is in a spherically symmetric state S1 with 1 radial node. Upon absorbing light
the ion undergoes transition to a state S2. State S2 has one radial node and its energy is equal to
the ground state energy of H atom. [IIT-2010]
(i) The state S1 is-
(A)1s (B) 2s (C)2p (D) 3s
(ii) Energy of state S1 in units of H atom ground state energy is –
(A) 0.75 (B) 1.50 (C) 2.25 (D) 4.50
2+
Solution: The spherically symmetric state S1 of Li with one radial node is 2s. Upon absorbing light, the
ion gets excited to state S2, which also has one radial node. The energy of electron in S2 is same
as that of H-atom in its ground state.
( 3) E1
2
Z2
 En = 2 E1 where E1 is the energy of H-atom in the ground state = 2 for Li 2+
n n
En = E1  n=3
 State S2 of Li 2+ having one radial node is 3p .
Orbital angular momentum quantum number of 3p is 1.
( 3) E = 2.25E
2

Every of state S1 =
( 2)
2 1 1

(i) (B)
(ii) (C)

135
CHEMISTRY ATOMIC STRUCTURE

EFFECTIVE NUCLEAR CHARGE & SCREENING EFFECT


Due to the presence of electrons in the inner shells, the electron in the outer shell will not experience the full
positive charge of the nucleus (Ze). The effect will be lowered due to the partial screening of positive charge
on the nucleus by the inner shell electrons. This is known as the shielding of the outer shell electrons from
the nucleus by the inner shell electrons, and the net positive charge experienced by the outer electrons is
known as effective nuclear charge (Zeffe)
Order of shielding effect: s > p > d > f
Due to spherical shape of s orbital, the s orbital electron spends more time close to the nucleus in comparison to p
orbital electron which spends more time in the vicinity of nucleus in comparison to d orbital electron.
1. Zeff decreases with increase in l, that is, s orbital electron will be more tightlybound to nucleus than p
orbital electron and so on.
2. Since extent of shielding is different for e-s in different orbitals, it leads to splitting of energy levels
within same shell.
3. Lower the value of (n + l) for an orbital, lower is its energy.
4. Energies of orbitals in same subshell decreases with increase in atomic number (Zeff).
E.g. E2s(H) >E2s(Li) >E2s(Na) >E2s(K)
5. e-e repulsions increases energy & more Zeff decreases energy. Depending on these 2, electrons achieve
stability.

SLATOR’S RULE - CALCULATING VALUE OF Zeff (Zeff = Z – )


Screening Constant () is calculated as follows:
• Step 1: Write the electron configuration of the atom in the following form:
(1s) (2s, 2p) (3s, 3p) (3d) (4s, 4p) (4d) (4f) (5s, 5p) . . .
• Step 2: Identify the electron of interest, and ignore all electrons in higher groups (to the right in the list
from Step 1). These do not shield electrons in lower groups
• Step 3: Slater's Rules is now broken into two cases:
o the shielding experienced by an s- or p- electron,
▪ electrons within same group shield 0.35, except the 1s which shield 0.30
▪ electrons within the n-1 group shield 0.85
▪ electrons within the n-2 or lower groups shield 1.00
o the shielding experienced by nd or nf valence electrons
▪ electrons within same group shield 0.35
▪ electrons within the lower groups shield 1.00

Note: e– present in (n + 1)th and higher orbits do not screen the e– concerned.
n2 I.E.
Zeff can also be calculated by Zeff. = (if I.E. is in eV)
13.6

Example 70: Cal. Zeff for (i) O (ii) 3d electron of Cu.


(i)  :1s ( 2 s 2 p )
2 6
Solution:
 = 0.35  5 + 0.85  2 = 1.75 +1.7 = 3.45
Z eff . = Z −  = 8 − 3.45 = 4.55.
( 2s 2 p ) ( 3s 3 p ) 3d 10 ;  = (18 1) + ( 9  0.35 ) = 21.15
2 8 8
(ii) 1s

136
ATOMIC STRUCTURE CHEMISTRY

Example71: Bromine atom possesses 35 electrons. It contains 6 electrons in 2p orbital, 6 electrons in 3p


orbital and 5 electrons in 4p orbital. Which of these electron experiences the lowest effective
nuclear charge?
Solution: 4p electrons, being farthest from the nucleus, experience the lowest effective nuclear charge.
These may be applied to bromine atom as follows:
35 Br = (1s 2 )( 2s 2 2 p 6 )( 3s 2 3 p 6 )( 3d 10 )( 4s 2 4 p 5 )
For 2 p6 , S = ( 0.35  7 ) + ( 0.85  2) = 2.45 + 1.70 = 4.15
Z eff . for 2p electron = 35 – 4.15 = 30.85
For 3 p6 , S = ( 0.35  7 ) + ( 0.85  8) + (1.0  2) = 2.45 + 6.80 − 2.0 = 11.25
Z eff . for 3p electron = 35 – 11.25 = 23.75
For 4 p5 , S = ( 0.35  6) + ( 0.85 18) + (1.0 10) = 2.10 + 15.30 + 10.0 = 27.40
Z eff . for 4p electron = 35 – 27.40 = 7.60
Thus, 4p has the lowest effective nuclear charge.
Example 72: Among the following pairs of orbitals which orbital will experience larger Zeff?
(i) 2s and 3s, (ii) 4d and 4f, (iii) 3d and 3p.
Solution: (i) 2 s is closer to the nucleus than 3 s. Hence, 2 s will experience larger effective nuclear charge.
(ii) 4 d (iii) 3 p (for same reasons)
Example 73: Unpaired electrons in Al & Si are present in 3p orbital. Which electrons will experience more Zeff
from the nucleus?
Solution: Silicon has greater nuclear (+14) than aluminium (+13). Hence, the unpaired 3 p electron in case
of silicon will experience more effective nuclear charge.
Example 74: Consider ground state of Cr atom (Z = 24). No. of electrons with l = 1 & 2 are, respectively:
(A) 16 and 5 (B) 12 and 5
(C) 16 and 4 (D) 12 and 4
Solution: Cr :  Ar  4s1 .

137
CHEMISTRY ATOMIC STRUCTURE

TOPICS FOR ADVANCE


Radial & angular wave functions

 ( r,  ,  ) = R ( r ) Y ( ,  ) Angular function yields allowed values of angular momentum & Z-


component. Characterized byl and m
for l = 0, there is just 1 value of m, m = 0, therefore, one spherical harmonic, which turns out to be a simple
constant.
For l = 1, there are three values of m, therefore, three functions.
R(r) is characterized by n &l, as this function satisfies the radial part of the Schrödinger equation, also known
as the radial function.

SCHRODINGER THEORY
(a) Schrodinger wave equation:
8 2 m
(i) 2 + ( E − V ) = 0 …(i) or
h2
(ii) E = Hˆ …(ii)
(b) Radial wave function curve:
It is the graph plotted between r vs r (distance from the nucleus)

(c) Probability wave function curve:


It is the graph plotted between 2 vs r (distance from nucleus)
For s orbitals, probability of finding electron is maximum on the surface of nucleus. For 2s orbital, the probability
density first decreases sharply to zero and again starts increasing. After reaching a small maxima, it decreases again
and approaches zero as the value of r increases further. The region where this probability density function reduces to
zero is called nodal surface or simply node generally nodes can be calculated by n – l– 1 for a orbital.

For s orbital, decay is fastest for 1s orbital.

138
ATOMIC STRUCTURE CHEMISTRY

(d) Radial Probability distribution curve:


It is the probability of finding e– in a shell of radius dr at a distance r from the nucleus.

For 1s →
R = Ke − r / a 0 R 2 = K 2 e −2r / a 0

4r2 2R2
(i) (ii) (iii)
R R

r/a0 r/a0 r/a0


For 2s →

(i) R
4r2 R2
(ii) R2 (iii)

r/a0 r/a0 r/a0


For 3s →

R2 4r2 R2
(i) R (ii) (iii)

r/a0
r/a0 r/a0

− r / 2a 0
For 2p → R = Kre

R R2
(i) 4r2 R2
(ii) (iii)

r/a0 r/a0 r/a0


For 3d →

R R2
4r2 R2
(i) (ii) (iii)

r/a0 r/a0 r/a0

139
CHEMISTRY ATOMIC STRUCTURE

WORKS RELATED WITH SCIENTISTS

Sr. Charge Mass


Particle Symbol Nature Discovered by
No. (in esu) × 10–10 (in amu)
1. Electron e– – 4.8029 0.0005486 J.J. Thomson

2. Proton p+ + + 4.8029 1.0072 Goldstein

3. Neutron n 0 1.0090 Chadwick

4. Positron e+, 1e0, + + + 4.8029 0.0005486 Anderson (1932)

5. Neutrino  0 0 < 0.00002 Pauli

Chamberlain Sugri

6. Anti-proton p – 4.8029 1.00787 and Weighland
(1955)

7. Photon h 0 0 0 Planck

8. Graviton G 0 0 0
Positive (mu
9. + + + 4.8029 0.1152 Yukawa (1935)
meson)
Negative(mu
10. – – – 4.8029 0.1152 Yukawa (1935)
meson)

11. Positive pi(meson) + + + 4.8029 0.1514

Negative
12 – – – 4.8029 0.1514 Powell (1947)
pi(meson)
13. Neutral pi (meson) 0 0 0 0.1454

140
ATOMIC STRUCTURE CHEMISTRY

1. 242 nm is just sufficient to ionise sodium atom. Ionisation energy of sodium atom will be:
−1 −1
(A) 494 kJ mol (B) 24.7 kJ mol
−1 −1
(C) 988 kJ mol (D) 247 kJ mol
2. If a moving particle has wavelength (  ) , the new wavelength when its kinetic energy is doubled is :
(A) 2 (B) 2
(C) λ/2 (D)

2

3. The potential energy of an electron in the 2nd orbit of Li+2 is :


(A) −13.6eV (B) −108.8eV
(C) –61.12 eV (D) –27.2 eV
4. If the radius of the first Bohr’s orbit is a0, then the radius of the third orbit would be.
(A) 3  a0 (B) 6  a0
(C) 9  a0 (D) None of these
st
5. Electron in 1 shell of H-atom gains energy twice the amount of its ionization energy. KE of the escaped
electron is:
(A) 10.2 eV (B) 3.4 eV
(C) 13.6 eV (D) 0.52 eV
6. The kinetic and potential energy (in eV) of electron present in third Bohr’s orbit of hydrogen atom are
respectively:
(A) – 1.51, – 3.02 (B) 1.51, – 3.02
(C) – 3.02, 1.51 (D) 1.51, – 1.51
7. If radius of the 2nd orbit in He+ is ‘r’ then the radius of the 6th orbit is:
(A) r (B) 9r
(C) 2r (D) None
+
8. The energy of an electron in the ground state of He is:
(A) Equal to the energy of an electron in the ground state of H atom
(B) Twice the energy of an electron in the ground state of H atom
(C) Four times the energy of an electron in the ground state of H atom
(D) There is no definite relation between the two energies.
9. The energy of an electron in the first Bohr orbit of H-atom is –13.6 eV. The possible energy value (s) of
the excited state (s) for electrons in Bohr orbits of hydrogen is (are)
(A) –3.4eV (B) –4.2eV
(C) –6.8 eV (D) +6.8 eV
−13.6
10. The energy of an electron in the H-atom is given by En = eV for high value of ‘n’, the
n2
E( n+1) − En is:
13.6  2
(A) 13.6eV (B) eV
n3
10.2 3.4
(C) eV (D) eV
n2 n

141
CHEMISTRY ATOMIC STRUCTURE

− 1
11. Select the incorrect graph for velocity of e in an orbit Vs. Z , and n:
n

(A) (B)

(C) (D)
12. In two H atoms A and B the electrons move around the nucleus in circular orbits of radius r and 4r
respectively. The ratio of the times taken by them to complete one revolution is
(A) 1 : 4 (B) 1 : 2
(C) 1 : 8 (D) 2 : 1
13. If ionizing energy of H atom is 13.6 eV, then the second ionizing energy of He+ should be
(A) 13.6 eV (B) 27.2 eV
(C) 54.4 eV (D) Cannot be predicted

14. Which transition is Be+3 would have the same wave length as in the case of 2nd line of the Balmer series
of He+?
(A) n = 4 to n = 2 (B) n = 5 to n = 3
(C) n = 8 to n = 4 (D) n = 7 to n = 2
15. Which of the following electron transitions in a hydrogen atom will require the largest amount of energy?
(A) from n = 1 to n = 2 (B) from n = 2 to n = 4
(C) from n = 5 to n = 1 (D) from n = 3 to n = 5
16. If the shortest wavelength of H atom in Lyman series is x, then longest wavelength in Balmer series of
He+ is
9x 36 x
(A) (B)
5 5
x 5x
(C) (D)
4 9

17. 100 W power source emits green light of wavelength  = 5000oA. How many photons per min are
emitted by source?
(A) 1.5  1022 (B) 3.5  1020
(C) 1.5  1018 (D) 3.5  1018
18. The wavelength of an electron moving with the velocity 1.6 × 106 ms– 1 is:
(A) 455 nm (B) 455 m
(C) 455 Å (D) 455 pm

142
ATOMIC STRUCTURE CHEMISTRY

19. A dye absorbs a photon of wavelength  and re-emits the same energy into two photons of wavelength 1
and 2 respectively. The wavelength  is related with 1and 2 as:
1 + 2 12
(A)  = (B)  =
12 1 + 2
1222 12
(C)  = (D)  =
1 + 2 ( 1 + 2 )
2

20. In which of the following shells will the moving electron trace maximum number of waves :
(A) 2 (B) 5
(C) 1 (D) 3
21. The value of x and u for a particle are 10 m and 6 × 10 ms– 1 respectively. The mass of the particle is :
–3 – 24

−9 −9
(A) 8.79 10 kg (B) 2.5110 kg
−8 −9
(C) 6.52 10 kg (D) 1.05 10 kg
22. Uncertainty in the position of an electron moving with a velocity 300 ms– 1, accurate upto 0.001% will be.
(A) 19.2 10−2 m (B) 5.76 10−2 m
(C) 1.92 10−2 m (D) 3.84 10−2 m
23. If E1, E2 and E3 are kinetic energy of electron, alpha particle and proton having same De-Broglie wave
length, then
(A) E1E2E3 (B) E1< E2< E3
(C) E2< E3< E1 (D) E1 =E2 =E3
24. If λ1 and λ2 denotes the de Broglie wavelength of two particles with same masses but charges in the ratio
of 1:2 after they are accelerated from rest through the same potential difference, then
(A) λ1 > λ2 (B) λ1 = λ2
(C) λ1 < λ2 (D) None
25. Uncertainty in the position of an electron moving with a v = 3 × 104 cm s– 1 accurate upto 0.011% will be:
(A) 1.92 cm (B) 7.68 cm
(C) 0.175 cm (D) 3.84 cm

26. Which quantum number was not given by the Schrödinger’s wave equation:
(A) Spin (B) Azimuthal
(C) Magnetic (D) Principal

27. For which d orbital will the electrons be found along the X axis and Y axis
(A) d xy (B) d xz
(C) d x2 − y 2 (D) d z 2
28. The value of l for n = 2 will be
(A) 0 and 1 (B) – 1 and +1
(C) 0 and 2 (D) None of these

143
CHEMISTRY ATOMIC STRUCTURE

29. The set of quantum numbers not applicable to an electron is:


1 1
(A) 1,1,1, + (B) 1, 0, 0, +
2 2
1 1
(C) 1, 0, 0, − (D) 2, 0, 0, +
2 2
30. Maximum number of electrons that can have principal quantum number, n = 3 and spin quantum
1
number ms = − is:
2
(A) 18 (B) 9
(C) 2 (D) 6
31. Radial nodes are maximum in:
(A) 4s (B) 4p
(C) 3d (D) 5f
32. Correct set of four quantum numbers for the valence (outermost) electron of rubidium (Z=37) is
1 1
(A) 5,0,0,+ (B) 5,1,0,+
2 2
1 1
(C) 5,1,1,+ (D) 6,0,0,+
2 2
33. Which of the following set of quantum nos. is not possible for an electron in ground state of atom with
atomic no. 19?
(A) n = 2, l = 0, m = 0 (B) n = 2, l = 1, m = 0
(C) n = 3, l = 1, m = −1 (D) n = 3, l = 2, m = 2
34. The number of spherical, planar and total nodes for 3d orbital are:
(A) 0, 2, 2 (B) 2, 1, 3
(C) 2, 0, 2 (D) 3, 1, 4
35. Which of the following graphs help to determine the shape of an orbital?
(A) Plots of Angular wave function (B) Plots of total probability density
(C) Plots of radial probability density (D) Both (A) and (B)
36. Which of the following is the correct representation of plot radial probability (4r2R2) in Y-axis vs
distance from the nucleus in X-axis for 1-electron of 4d-atomic orbital?
Y Y

(A) 2 2 2 2
4 r R (B) 4 r R

r X r X

Y Y

(C) 2 2 (D) 2 2
4 r R 4 r R

r X r X

37. If the nodes at infinity are not neglected, then what is the total number of radial and angular nodes of 5f-
orbitals?
(A) 4 (B) 3
(C) 5 (D) infinity

144
ATOMIC STRUCTURE CHEMISTRY

38. Out of the following, which is the correct match for radial probability of finding the electron of 2s
orbital?
Y A B

4pr2R22

r X
+ 2+
(A) A-H, B-He , C-Li (B) A- He+, B-H, C-Li2+
(C) A- Li2+, B-He+, C-H (D) Can’t say

39. The orbital angular momentum of an electron is the 4f orbital is


h h
(A) 6 (B) 2 
2 2
h
(C) 0 (D) 12 
2
40. Which of the following orbitals have orbital angular momentum of 6h
(A) 4f (B) 5f
(C) 2p (D) 5d
41. When 3d orbitals are completely filled. The new electron will enter in:
(A) 4s-orbitals (B) 4p-orbitals
(C) 4d-orbitals (D) None of these
42. In an atom which has 2K,8L,8M and 2N electrons in the ground state the total number of electrons
having l = 1 are:
(A) 20 (B) 8
(C) 12 (D) 10
43. Which of the following will have all the electrons paired:
(A) Ti+4 (B) Sc +3
(C) Ca +2 (D) all of these
1
44. The maximum number of electrons in the oxygen atom that have spin quantum number + is:
2
(A) 3 (B) 4
(C) 2 (D) 5
45. Which of the following is the correct set of quantum number for the outermost electron is zinc :
1 1
(A) n = 3, l = 2, m = 0, s = (B) n = 4, l = 0, m = 0, s =
2 2
1 1
(C) n = 3, l = 1, m = 1, s = (D) n = 4, l = 1, m = 0, s =
2 2
46. The 2p subshell cannot accommodate more than 6 electrons. This is in accordance to:
(A) Hund’s rule (B) Aufbau’s Principle
(C) Bohr’s rule of 2n electrons in a shell
2
(D) Pauli’s exclusion principle
47. In which of the following pairs the second species has a greater number of unpaired electrons
(A) Co and Ni (B) Fe+3 and Fe+2
(C) O and N (D) Ni and Ni+2

145
CHEMISTRY ATOMIC STRUCTURE

48. Which series of subshells is arranged in the order of decreasing energy for multi-electron atoms?
(A) 6s, 4f, 5d, 6p (B) 4f, 6s, 5d, 6p
(C) 5d, 5f, 6s, 6p (D) 6p, 5d, 4f, 6s
49. In a set of degenerate orbitals the electrons distribute themselves to retain similar spins as far as possible.
This statement is attributed to
(A) Pauli’s exclusion principle (B) Aufbau principle
(C) Hund’s Rule (D) Slater rule
50. The number of unpaired electrons present in the ground state of Cr is
(A) 3 (B) 4
(C) 5 (D) 6
51. Which of the following is having the maximum number of unpaired electrons?
(A) Mg2+ (B) Ti3+
(C) V3+ (D) Fe2+
52. Which of the following violates the Pauli Exclusion Principle?

(A) (B)

(C) (D)
53. The number of electrons with l = 2 in an atom having atomic number 23 is
(A) 2 (B) 3
(C) 4 (D) 5
54. Which is the incorrect statement?
(A) Hund’s rule deals with degenerate orbitals
(B) Pauli exclusion principal stated as only 2 e- may exist in same orbital & these electrons must have
opposite spin.
(C) In ground state of atom the orbitals are filled on the basis of increasing ( n + l ) value.
(D) The maximum number of electrons in the subshell with principal quantum number ‘n’ is equal to 2n 2
55. Consider the ground state of Cu atom (z = 29). The number of electrons with the azimuthal quantum
numbers l = 1 and 2 are respectively
(A) 16 and 6 (B) 12 and 10
(C) 12 and 9 (D) 16 and 5

56. Which of the following species will have magnetic moment zero?
(A) Mn+2 (B) Cu+2
+3
(C) Fe (D) Zn
57. d6 configuration will result in total spin of:
3 1
(A) (B)
2 2
(C) 2 (D) 1
58. The correct order of total number of node of atomic orbitals is:
(A) 4f > 6s > 5d (B) 6s > 5d > 4f
(C) 5f > 5d > 6s (D) 5d > 5f > 6s
59. The value of the magnetic moment of a particular ion is 2.83 Bohr magneton. The ion is
(A) Fe2+ (B) Ni2+
(C) Mn2+ (D) Co3+

146
ATOMIC STRUCTURE CHEMISTRY

147
CHEMISTRY ATOMIC STRUCTRUE

1. Write down the complete symbol for the atom with the given atomic number (Z) and atomic mass (A).
(i) Z = 17, A = 35, (ii) Z = 92, A = 233, (iii) Z = 4, A = 9
2. An element with mass number 81 contains 31.7% more neutrons as compared to protons. Assign the
atomic symbol.
3. An ion with mass number 37 possesses one unit of negative charge. If the ion contains 11.1% more
neutrons than electrons, find the symbol of the ion.
4. An ion with mass number 56 contains 3 units of positive charge and 30.4% more neutrons than electrons.
Assign the symbol of this ion.

5. (i) How is the wave length of a moving particle related to its kinetic energy?
(ii) How is the kinetic energy of the particle affected when the wave length is doubled?

6. According to Bohr’s model of an atom, electrons rotate around the nucleus in circular stationary orbits.
What is the significance of the term ‘stationary’ in the expression?
7. Calculate the radius of the 2nd orbit of Be+3, if the radius of the fourth orbit of He+ is x.

8. The ratio of the potential energy and total energy of an electron in a Bohr orbit of a hydrogen like species is :
(A) 2 (B) –2
(C) 1 (D) –1
9. The velocity of an electron is the third shell of Li+2 ion is :
−1
(A) 4.2 10 m/s (B) 2.110 ms
6 6

−6 −1
(C) 2.110 m/s (D) 6.3 10 ms
6

10. When electron of excited H-atom jumps from a higher to lower energy state, which of the following
statement is wrong:
(A) its potential energy decreases (B) its kinetic energy increases
(C) its angular momentum changes (D) The total energy increases
11. The energy of an electron in the first Bohr orbit of H atom is −13.6eV . Which of the following cannot
be a value of the energy in the excited states?
(A) −3.4eV (B) 1.51eV
(C) –0.85eV (D) –1.51 eV
12. Which of the following orbits has the same radius as that of the first-Bohr’s orbit for hydrogen atom.
(A) Second orbit of He+ (B) 3rd orbit of Li+2
(C) Third orbit of Be3+ (D) second orbit of Be+3

148
ATOMIC STRUCTRUE CHEMISTRY

13. The ratio of the kinetic energy of the n = 2 electron for the H atom to that of He+ ion is:
(A) 1/4 (B) 1/2
(C) 1 (D) 2
14. Kinetic energy of an electron in a given shell in a single electron species is
(A) 2PE (B) –PE
(C) –PE/2 (D) –2PE
15. Which hydrogen like species will have the same radius as that of the Bohr orbit of a hydrogen atom?
+2 +3
(A) n = 2, Li (B) n = 2, Be
+ +2
(C) n = 2, He (D) n = 3, Li
16. The ratio of the KE and total energy of an electron in a Bohr orbit of a hydrogen like species is
(A) –1 (B) 1
(C) –2 (D) 1/2
17. The ratio of the potential energy of an electron in the 2nd shell of H atom to that in the He+ ion is:
(A) 1/4 (B) 1/2
(C) 1 (D) 2
18. The ratio of the ionization energy of an electron in the 1 and 5th Bohr orbit is:
st

(A) 2 : 1 (B) 4 : 1
(C) 25 : 1 (D) 9 : 1
19. If time period of revolution of an electron is 1 Bohr orbit is ‘T’ then time period of revolution in the 3rd
st

orbit will be:


(A) 9T (B) T/3
(C) 27T (D) 18T
20. Ratio of frequencies of the revolution of an electron in the 1st and the 3rd Bohr orbit is
(A) 64/1 (B) 1/64
(C) 1/27 (D) 27/1

21. Velocity of an e in the 2nd shell of He+ changes as it drops to the 1st shell such that:
(A) it halves (B) it remains the same
(C) it doubles (D) it becomes four times
22. The electron in a single electron species undergoes transition from n2 to n3 ( E )1 and than n3 to
n5 ( E2 ) . If the electron was transported from n2 to n5 ( ET ) , then.
(A) ET = E1 + E2 (B) vT = v1 + v2
1 1 1
(C) = + (D) All of the above
T 1 2
23. Which transition is an H atom would required the largest amount of energy?
(A) n1 to n2 (B) n2 to n3
(C) n3 to n4 (D) n4 to n5
24. Radius of shell for an electron in the ground state of H atom is 0.53 Å. Radius of the Be +3 electron in a
similar shell is:
(A) 0.353 Å (B) 0.0883 Å
(C) 0.1325 Å (D) 0.265 Å

149
CHEMISTRY ATOMIC STRUCTRUE

25. How many spectral lines will be observed when an electron from 6th shell in a hydrogen atom drops to
the ground state?
26. What transition in H atom spectrum would have the same wavelength as Balmer transition n = 4 to n = 2
of He+ spectrum?
27. Calculate the wavelength for the emission transition if it starts from the orbit having radius 1.3225 nm
ends at 211.6 pm. Name the series to which this transition belongs and the region of the spectrum.

28. Which radiation in the Balmer series of H spectrum will have the highest wave number?
(A) n = 3 to n = 2 (B) n =  to n = 2
(C) n = 5 to n = 2 (D) n = 10 to n = 2
29. The number of spectral lines obtained in a transition from n = 4 to n = 1 are:
(A) 6 (B) 8
(C) 4 (D) 2
nd
30. The wavelength of the 2 line in the Balmer series of hydrogen spectrum is:
3RH
(A) (B) 4 RH
16
16 RH
(C) (D)
3RH 48
31. Which transition in He+ will have the same wavelength as the 2nd line in the Lyman series of Hydrogen?
(A) n = 5 to n = 1 (B) n = 4 to n = 2
(C) n = 6 to n = 2 (D) n = 5 to n = 2
32. The wave number for the longest wave length transition in the Balmer series of atomic hydrogen is :
(A) 2.5  105 m–1 (B) 1.52  107 m–1
(C) 2.5  107 m–1 (D)1.52  106 m–1
33. Which of the following statements is not true?
(A) Lyman spectral series of hydrogen atom lies in the ultraviolet region of electromagnetic radiation
(B) Balmer spectral series of hydrogen atom lies in the visible region of electromagnetic radiation
(C) Paschen spectral series of hydrogen atom lies in the visible region of electromagnetic radiation
(D) Brackett spectral series of hydrogen atom lies in the infrared region of electromagnetic radiation
34. In a single isolated atom an electron make transition from 5thexicited state to 2nd excited state then
maximum number of different types of photons observed is
(A) 3 (B) 4
(C) 6 (D) 15
35. Bohr model can explain:
(A) The spectrum of hydrogen atom only
(B) Spectrum of an atom or ion containing two electrons
(C) The spectrum of hydrogen molecule
(D) Solar spectrum

150
ATOMIC STRUCTRUE CHEMISTRY

36. Neon gas is generally used in the signboards. If it is emitted strongly at 616 nm. Calculate: (a) Frequency
of emission (b) distance travelled by the radiation in 30 sec (c) energy of quantum (d) no. of quanta
present if it produces 2J energy.
37. 100watt bulb emits monochromatic light of wavelength 400 nm. Cal. no. of photons emitted per second
by the bulb.
38. What is the number of photons of light with a wavelength of 4000 pm that provide 1 J of energy?
39. Lifetime of the molecules in the excited state are often measured by using a pulse radiation sources of
duration nearly in nano second range. If the radiation sources has the duration of 2 ns and the number of
photons emitted during the pulse sources is 2.5 1015 , calculate the energy of the sources.
40. A photon of 300 nm is absorbed by a gas which then emits two photons. One emitted photon has a
wavelength of 400 nm. Calculate the energy of the other photon that was emitted out.
41. If an electron is moving in the nth orbit, what is the number of waves formed? Explain.
42. What was the effect of Heisenberg’s uncertainty principle on Bohr’s model for the structure of an atom?
43. A microscope using suitable photons is employed to locate an electron in an atom within a distance of 0.1
Ǻ. What is the uncertainty involved in the measurement of its velocity?
44. Dual behavior of matter proposed by de Broglie led to the discovery of electron microscope often used for the
highly magnified images of biological molecules and other types of materials. If the velocity of electron in this
−1
microscope is 1.6 10 m s , calculate the de Broglie wavelength associated with this electron.
6

45. Similar to electron diffraction neutron diffraction microscope is also used for the determination of the
structure of molecules. If the wavelength used here is 800 pm, calculate characteristic velocity associated
with neutron.
−1
46. The velocity associated with proton moving in a potential difference of 1000 V is 4.37 10 m s . If a
5

hockey ball of mass 0.1 kg is moving with this velocity, calculate the wavelength associated with this
velocity.
47. If the position of an electron is measured within an accuracy of  0.002 nm, calculate the uncertainty in
h
momentum of the electron. Suppose the momentum of electron is nm, is there any problem in
4  0.05
defining this value?
48. Show that the circumference of the Bohr orbit for the hydrogen atom is an integral multiple of the
Broglie wavelength associated with the electron revolving around the orbit.
49. What is the significance of the uncertainty principle in our daily life?
50. All material particles have wave nature. How does the wave length of the particle depend on the mass of
the particle?

51. Generally, the limit of visible spectrum is


(A) 1000 to 3000 Å (B) 4000 to 7000 Å
(C) 8000 to 10000 Å (D) 12000 to 15000 Å
52. Which characteristic of the electromagnetic wave was not given by Maxwell’s wave theory?
(A) They constitute a continuous flow of energy.
(B) They travel in vacuum with a speed of 3  108ms–1.
(C) They have particulate nature.
(D) They do not require a material medium to travel.

151
CHEMISTRY ATOMIC STRUCTRUE

53. Yellow light emitted from a sodium lamp has wave number: 1.724  106 m–1. Its frequency and wavelength is:
(A) V = 2.05  1012s–1,  = 115 nm (B) V = 5.17  1014s–1,  = 580 nm
(C) V = 1.1  1012s–1,  = 423 nm (D) V = 6.8  1011s–1,  = 289nm
54. Planck’s constant has same dimensions as that of
(A) power (B) work
(C) radiant energy (D) angular momentum
55. 60 watt bulb emits monochromatic light of wavelength 400 nm. Calculate no. of photons emitted per sec
by the bulb:
(A) 2  10–20 (B) 1.207  1020
(C) 3  10–20 (D) 3  1020
56. If two radiations have same amount of total energy but different frequencies, then :
(A) the light with higher frequency will be brighter
(B) both the radiations will have same brightness
(C) the intensity of light does not depend on its frequency
(D) the light with lower frequency is brighter
57. Which of the following radiation will have the highest wave number?
(A) Cosmic rays (B) Infrared rays
(C) Radio waves (D)  rays
58. The experimental evidence in support of the idea that electronic energies in an atom are quantized is
(A) Electronic can lose energy in fixed amount during the transition from one state to another.
(B) Electron is a highly unstable particle
(C) Electron is a fast moving particle
(D) There was no experimental evidence in support of the statement
59. The wavelength of an electron that has been accelerated in a particle accelerator through a potential
difference of 100 million volts is:
−13 −13
(A) 2.44 10 m (B) 1.22 10 m
−10 −10
(C) 2.44 10 m (D) 1.22 10 m
60. That the electron can behave like a wave was experimentally proved by:
(A) JJ Thomson (B) G P Thomson
(C) W. Heisenberg (D) E. Schrödinger
61. Establishment of the dual behavior of matter led to the discovery of:
(A) Telescope (B) Electron microscope
(C) compound microscope (D) Spectroscope
th
62. An electron in the 4 shell traces
(A) 16 waves (B) 2 waves
(C) 8 waves (D) 4 waves
63. A dust particle having mass equal to 10 g, diameters 10– 4 cm and velocity 104 cm sec– 1. The error in
– 11

measurement of velocity is 0.2%. The uncertainty in position is:


−17
(A) 2.6 10−11 cm (B) 2.6 10 cm
−19
(C) 5.2 10−18 cm (D) 5.2 10 cm
64. If uncertainty in position is equal to the uncertainly in momentum then the uncertainty in the velocity of
the electron is:
(A) 7.98 1012 ms −1 (B) 3.16 1012 ms −1
(C) 3.16 1010 ms −1 (D) 7.98 1010 ms −1

152
ATOMIC STRUCTRUE CHEMISTRY

65. Heisenberg’s uncertainty principle is relevant for a particle with:


(A) Small radius (B) Negligible mass
(C) Small velocity (D) High velocity
−5 −1
66. The uncertainty in the momentum of an electron is 1.0 10 kg ms . The uncertainty in its position will

(
be. h = 6.63 10−34 kg m 2s −1 )
(A) 1.05 10−26 m (B) 1.05 10−28 m
(C) 5.27 10−30 m (D) 5.25 10−28 m

67. What is the difference between ‘l’ and ‘L’?

68. If the value of azimuthal quantum number for an electron is 1, then which of the following values of m is
not permissible:
(A) 3 (B) – 1
(C) 0 (D) 1

69. Using s, p, d notations, describe the orbital with the following quantum numbers.
(a) n = 1, l = 0 (b) n = 3, l = 1 (c) n = 4, l = 2 (d) n = 4, l = 3
70. Explain, giving reasons which of the following sets of quantum numbers are not possible?
1 1
(a) n = 0, l = 0, ml = 0, ms =  (b) n = 1, l = 0, ml = 0, ms = −
2 2
1 1
(c) n = 1, l = 1, ml = 0, ms = + (d) n = 2, l = 1, ml = 0, ms = −
2 2
1 1
(e) n = 3, l = 3, ml = −3, ms =  (f) n = 3, l = 1, ml = 0, ms = 
2 2
71. (i) An element has n = 3 . What are the possible values of l and ml ?

(ii) List the quantum numbers ( ml and l ) of electrons for 3d orbital.


(iii) Which of the following orbitals are possible? 1 p, 2s, 2 p and 3 f

72. Which of the following orbital is dumbbell shaped


(A) 4f (B) 5d
(C) 3p (D) 2s
73. Where is the maximum probability of finding an electron in the dxyorbital.
(A) Along the X axis
(B) Along the Y axis
(C) At an angle of 45o from the X axis and Y-axis
(D) At an angle of 90o from the X axis and Y-axis.

153
CHEMISTRY ATOMIC STRUCTRUE

74. Which d-orbital appears different in shape from the other d-orbitals
(A) d z 2 (B) d xz
(C) d xy (D) d x2 − y 2
75. Which of the following combination of quantum numbers is allowed?
n l m s n l m s
(A) 2 0 –2 1/2 (B) 3 3 –1 –1/2
(C) 4 3 –1 –1/2 (D) 1 1 0 –1/2
76. Which one of the following set of quantum numbers is not permitted for a 3p electron?
−1 1
(A) n = 3, l = 3, m = −2, s = (B) n = 3, l = 1, m = −1, s =
2 2
−1 −1
(C) n = 3, l = 1, m = 1, s = (D) n = 3, l = 1, m = 0, s =
2 2
77. The total number of electrons in the principle quantum number ‘n’ is :
(A) n2 (B) 2n2
(C) 2n (D) n
78. Which of the following statement is correct?
1
(A) The spin quantum number of an electron spinning in the clockwise direction is
2
1
(B) The spin quantum number of an electron spinning in the anti-clockwise direction is
2
(C) The spin one electron in the orbital is clockwise and that of the other electron is anticlockwise
(D) There is no classical analogue of the quantum mechanics model of the structure of an atom
79. What is the maximum number of electrons that have: n = 2, l = 1
(A) 2 (B) 10
(C) 5 (D) 6
80. Probability of finding an electron at a point within an atom is
2 2
(A) Equal to the ψ at that point (B) Proportional to the ψ at that point
2
(C) Inversely proportional to the ψ at that point (D) None of the above
81. The maximum number of electron that can have n = 4, l = 2, m = 0 is
(A) 10 (B) 6
(C) 2 (D)14
82. Which of the following are possible values of n, l, and m for an atom having maximum value of m = + 2?
(A) n = 4, l = 3, m = + 2 (B) n = 3, l = 2, m = – 2
(C) n = 3, l = 3, m = + 2 (D) n = 4, l = 3, m = – 2
83. The correct set of quantum number for a 3p electron is:
1 1
(A) 3, 0, 0, (B) 3, 1, 1, −
2 2
1 1
(C) 3, 0, 0, − (D) 3, 2, –2,
2 2
84. The shape of the orbital with l = 2 and m = 0 is:
(A) Spherical (B) Dumb-bell
(C) Triangle planar (D) square planar

154
ATOMIC STRUCTRUE CHEMISTRY

85. Which of the following orbitals cannot exist


(A) 3d (B) 3f
(C) 5p (D) 7s
86. The maximum number of electrons in a subshell having the same value of spin quantum number is given by:
(A) l + 2 (B) 2l + 1
(C) l (l + 1) (D) l ( l + 1)
87. Which of the following orbitals can have ms = +3
(A) 4f (B) 2p
(C) 3p (D) 5s
88. Which of these orbitals have three planar nodes:
(A) 4f (B) 3d
(C) 3p (D) 2p
89. The orbital angular momentum of 3p electron is:
(A) 3h (B) 6h
h
(C) zero (D) 2
2
90. The nodes present in the 3p-orbitals are
(A) one spherical, one planar (B) 2 spherical
(C) 2 planar (D) one planar
91. The orbital which is non-directional and has no planar node is :
(A) s (B) p
(C) d (D) f
92. Which two orbitals are located along the axis, and not between the axis?
(A) d xy , d z 2 (B) d xy , p z
(C) d yz , p x (D) pz , d x 2 − y 2

93. Give the electronic configuration of the last and outermost electron of chromium (at no : 24).
94. How many electrons in an atom at the most can have n + l = 4?
95. Write down the electronic configurations of the following ions:
(a) H − (b) Na + (c) O 2− (d) F −
96. An atom of an element contains 29 electrons and 35 neutrons. Deduce: (i) the number of protons and (ii)
the electronic configuration of the element.
97. Indicate the number of unpaired electrons in:
(a) P (b) Si (c) Cr (d) Fe and (e) Kr

98. An ion Cr+3 has x electrons in the 4s orbital:


(A) 0 (B) 1
(C) 2 (D) 4
99. The species M has completely filled 3d subshell. The atomic number of ‘M’ is:
+2

(A) 30 (B) 28
(C) 25 (D) 18

155
CHEMISTRY ATOMIC STRUCTRUE

100. No two electrons can have the same set of four quantum numbers because it violates:
(A) Hund’s rule (B) Pauli’s exclusion principle
(C) Heisenberg’s uncertainty principle (D) Aufbau’s Principle
101. The electronic configuration of Cs [Z = 55] is :
(A)  Xe 5s1 (B)  Xe 6s1
(C)  Xe 5s (D)  Xe 6s
2 2

102. The total number of unpaired electrons in Ha[Z = 105] are :


(A) 2 (B) zero
(C) 3 (D) 5
103. For which of the following sets of quantum numbers, an electron will have the highest energy?
n l m s n l m s
(A) 3 2 1 –1/2 (B) 4 3 –1 +1/2
(C) 4 1 –1 +1/2 (D) 5 0 0 –1/2
104. Ground state electronic configuration of O atom cannot be represented by

(A) (B)

(C) (D)
105. How many electrons in 19K have n = 3; l = 0?
(A) 1 (B) 2
(C) 4 (D) 3
106. The energy of an electron in an atomic orbital of a multi-electron atom depends upon
(A) The principal quantum number only
(B) The principal and azimuthal quantum numbers only
(C) The principal, azimuthal and magnetic quantum number only
(D) The principal, azimuthal, magnetic and spin quantum numbers

107. The bromine atom possesses 35 electrons. It contains 6 electrons in 2p-orbitals; 6 electrons in 3p-orbitals
and 5 electrons in 4p-orbitals. Which of these electrons experiences the lowest effective nuclear charge?
108. Among the following pairs of orbitals, which orbital will experience the larger effective nuclear charge?
(a) 2s and 3s (b) 4d and 4f (c) 3d and 3p
109. The unpaired electrons in Al and Si are present in 3p-orbitals. Which electron will experience more
effective nuclear charge from nucleus?

110. The magnetic moment of the chromium atom in its ground state is:
(A) 1.73 BM (B) 6.48 BM
(C) 5.916 BM (D) 3.872 BM

156
ATOMIC STRUCTRUE CHEMISTRY

35 233 (a) 4.87  1014 Hz (b) 9  109m


(i) 17 Cl (ii) 92 U (iii) 94 Be (c) 3.227 10−19 J (d) x = 6.2  1018
( 81
35 Br ) 2.012 1020 photons
37
17 Cl− 2.012 1016 quanta
8.25 10−10 J
56
26 Fe3+
(i)  1 (ii) 1
KE2 =  KE1 1.65 10−26 J
KE 4
5.79 106 ms−1
1
rBe+3 =  x 455 pm
8
494.47 m s −1
1.516 10−38 m
1.055 10−24 kg ms-1
h
=
mV

1s2 2s2 2p6 3s2 3p6 3d54s1

(a) H (1e− ) → 1 s1 ; H − ( 2e− ) → 1s 2


(b) Na + (10e− ) → 1s 2 , 2s 2 , 2 p 6 ,3s 0
( )
(c) O 8e − → 1s 2 , 2s 2 2 p 4 ;

O 2−
(10e ) → 1s ,2s
− 2 2
2 p6
(d) F ( 9e ) → 1s , 2 s 2 p ;
− 2 2 5

F − (10e− ) → 1s 2 , 2s 2 2 p 6
(i)29;(ii)1s2 ,2s2 2 p6 ,3s2 3 p6 3d10 ,4s1
(a) 3 (b) 2 (c) 6 (d) 4

157
CHEMISTRY ATOMIC STRUCTURE

1. A certain radiation is emitted at a frequency of 900 KHz. The wavelength of electromagnetic radiation is:
(A) 900 m (B) 270 m
(C) 2.70 km (D) 330 m
2. What is the optimum conditions required to study the conduction of electricity through gases?
(A) High pressure and low voltage (B) High pressure and high voltage
(C) Low pressure and high voltage (D) Low pressure and low voltage
3. Most penetrating is:
(A)  − particle (B)  − particle
(C) proton (D) neutron
4. When beryllium is bombarded with alpha particles (Chadwick’s experiment) extremely penetrating
radiations, which cannot be deflected by electrical or magnetic field are given out. These are:
(A) A beam of protons (B) Alpha rays
(C) A beam of neutrons and protons (D) A beam of neutrons
5. Number of protons, neutrons and electrons in the element 231 X 89 is
(A) 89, 89, 242 (B) 89, 142, 89
(C) 89,71, 89 (D) 89,231, 89
6. Nucleons are
(A) Only neutrons (B) neutrons + protons
(C)neutrons + protons + electrons (D) neutrons + electrons
7. The increasing order for the values of e/m (charge/mass) is
(A) e, p, n,  (B) n, p, e, 
(C) n, p, , e (D) n, , p, e
8. The ideal body, which emits and absorbs radiations of all frequencies, is called a black body and the
radiation emitted by such a body is called
(A) white body radiation (B) black body radiation
(C) visual radiation (D) none of these
9. Which of the following has least de-Broglie ?
(A)e– (B) p
(C)CO2 (D) SO2
10. The potential energy of an electron in the first Bohr orbit in the He+ ion is
(A)–13.6 eV (B) –27.2 eV
(C)–54.4 eV (D) –108.8 eV
11. Zeeman effect explains splitting of lines in
(A)magnetic field (B) electric field
(C)Both (A) and (B) (D) None of these
12. Which is true for Hydrogen spectrum?
(A)It is a continuous spectrum.
(B) It is a discontinuous spectrum.
(C)It is continuous but tends to become discontinuous at infinity.
(D)It is discontinuous but tends to become continuous at infinity.
13. A typical golf ball weighs 40.0 g. If it is moving with a velocity of 20.0 ms–1, its de-Broglie wavelength is
(A)1.66  10–34 nm (B) 8.28  10–32 nm
(C)8.28  10 nm–25
(D) 1.66  10–24 nm

158
ATOMIC STRUCTURE CHEMISTRY

14. Calculate de-Broglie wavelength of a CO2 molecule in Å at 27oC moving with a velocity of 102 ms–1
(A)1.5  10–27 Å (B) 10 Å
(C)9  10 Å
–11
(D) 0.9 Å
15. How many spectral lines would be observed in the visible region when an e– jumps from 5th excited state
to ground state in H-atom.
(A)6 (B) 10
(C)15 (D) 4
16. What is the ratio of radius of 2 Bohr orbit of He to 1 Bohr orbit of Li2+
nd + st

(A)1 : 4 (B) 4 : 1
(C)6 : 1 (D) 1 : 6
17. Which of the following statements concerning the quantum numbers are correct?
(I) Angular quantum number determines the three-dimensional shape of the orbital.
(II) The principal quantum number determines the orientation and energy of the orbital.
(III)Magnetic quantum number determines the size of the orbital.
(IV) Spin quantum number of an electron determines the orientation of the spin of electron relative to the
chosen axis.
The correct set of option is
(A)I and II (B) I and IV
(C) III and IV (D) II, III and IV
18. How many electrons in Cr (Chromium) atom have n + l = 4
(A) 6 (B) 7
(C) 8 (D) 5
19. Uncertainty in the position of an electron ( mass = 9.110 −31
kg ) moving with a velocity 300ms ,
−1

accurate upto 0.001% will be ( h = 6.63  10 Js )


−34

(A) 1.92 10−2 m (B) 3.84 10−2 m


(C) 19.2 10−2 m (D) 5.76 10−2 m
20. What is the correct orbital designation of an electron with the quantum number?
n = 4, l = 2, m = −2, s = +1/ 2
(A) 4 d xy (B) 4 d z 2
(C) 4d xz (D) 4 d yz
21. How many maximum electrons in Fe ( Z = 26) will have s = +1/ 2
(A) 15 (B) 13
(C) 11 (D) 10
22. . Which rule is violated?

(A) Hund’s (B) Aufbau


(C) Pauli (D) Both (A) and (B)
23. Which out of the following is a non-directional sub-shell?
(A) s (B) p
(C) d (D) f
24. The lowest value of n that allows g orbitals to exist is
(A) 3 (B) 4
(C) 5 (D) 6

159
CHEMISTRY ATOMIC STRUCTURE

25. Which has highest energy in H-atom?


(A) 2 s (B) 3p
(C) 3d (D) 4s
26. Which is the correct curve for 4 r 2 2 vs r for 1s

(A) (B)

(C) (D)
27. Total number of nodes in a 4p subshell are
(A) 2 (B) 3
(C) 1 (D) 4
28. There are 2K, 8L, 16M electrons in an atom. How many electrons have n + l = 4 in this species?
(A) 16 (B) 8
(C) 6 (D) 2
29. Which is correct order of paramagnetic nature of the species?
(A) V2+> Mn2+> Fe2+ (B) Mn2+> Fe2+> V2+
2+ 2+ 2+
(C) Mn > V > Fe (D) Fe2+> Mn2+> V2+
30. Which is true?
(A) 4s electron in Cu experiences a higher Zeff than its 3d electron.
(B) 4s electron in Cr experiences a lower Zeff than its 3d electron.
(C)s-subshell has directional characteristics.
(D) p-subshell is double dumb-bell shaped.
31. The electronic configuration 1s2, 2s2 2p6, 3s1 3p1 correctly describes
(A) Ground state of Na (B) Ground state of Si+
(C) Excited state of Mg (D) Excited state of Al3+
32. Number of angular nodes for 4d orbital is _____
(A) 4 (B) 3
(C) 2 (D) 1
33. The correct designation for  320 is
(A) 3d xy (B) 3d xz
(C) 3d x2 − y 2 (D) 3d z 2
34. Match the columns.
Column-I Column-II
(Atom/ Ion) (Electronic configuration)
(a) Cu (p) 1s22s22p63s23p63d10
(b) Cu2+ (q)1s22s22p63s23p63d10 4s2
(c) Zn2+ (r) 1s22s22p63s23p63d10 4s1
(d) Cr3+ (s) 1s22s22p63s23p63d9
(t) 1s22s22p63s23p63d3
(A) a-s, b-r, c-p, d-t (B) a-r, b-s, c-p, d-t
(C) a-r, b-s, c-t, d-p (D) a-r, b-t, c-p, d-s

160
ATOMIC STRUCTURE CHEMISTRY

35. The ratio of the mass of a proton to the mass of an electron is:
(A) 1 : 1837 (B) 9.1  102 : 1
(C) 1837 : 1 (D) 1.67  10–2 : 1
−3
36. The Fundamental particles in 147 N are;

(A) p = 7; n = 14; e = 10 (B) p = 7; n = 7; e = 10


(C) p = 7; n = 8; e = 7 (D) p = 7; n = 7; e = 7
37. An ion has mass number 31 and one neutron more than the number of protons. Calculate the charge on it
if there are 18 electrons.
(A) –2 (B) +3
(C) –1 (D) –3
38. Which of the following are isoelectronic with one another
(A) Na + and Ne (B) K + and O
(C) Ne and O (D) Na + and K +
39. Compare the energies of two radiations E1 with wavelength 800 nm and E2 with wavelength 400 nm.
(A) E1 = 2E2 (B) E1 = E2
1
(C) E2 = 2E1 (D) E2 = − E1
2
40. Select the correct statement:
(A) The ideal body, which emits and absorbs all frequencies, is called a black body.
(B) The exact frequency distribution of the emitted radiation from a black body depends upon its
temperature.
(C) The radiation emitted goes from a lower frequency to a higher frequency as the temperature increases.
(D) All of the above are correct statements.
41. Which one of the following is not the characteristic of Planck’s quantum theory of radiation?
(A) The energy is not absorbed or emitted in whole number or multiple of quantum.
(B) Radiation is associated with energy.
(C)Radiation energy is not emitted or absorbed continuously but in the form of small packets called
quanta.
(D)This magnitude of energy associated with a quantum is proportional to the frequency.
42. Of the following, radiation with maximum wavelength is
(A) UV (B) Radiowaves
(C) X-ray (D) IR
43. Which of the following relates to light as wave motion?
(A) Diffraction (B) Interference
(C) Both (A) and (B) (D) None of these
44. In hydrogen atom, the electrons are excited to the 5th energy level. The number of lines that may appear
in the spectrum will be
(A) 4 (B) 8
(C) 10 (D) 12
45. As an electron is brought from an infinite distance close to the nucleus of the atom, the energy of the
electron-nucleus system
(A) Increases to a greater positive value (B) Decreases to a smaller positive value
(C) Decreases to a greater negative value (D) Increases to a smaller negative value

161
CHEMISTRY ATOMIC STRUCTURE

46. Energy levels A, B, C of a certain atom corresponds to increasing value of energy, i.e, EA  EB  EC . If
1 , 2 and 3 are the wavelengths of radiations corresponding to the transitions C to B, B to A and C to
A respectively, which of the following statement is correct?

12
(A) 3 = 1 + 2 (B) 3 =
1 + 2
(C) 1 + 2 + 3 = 0 (D) 32 = 12 + 22
47. The first emission line of Balmer series in H spectrum has the wave number equal to:
9 RH 9 RH
(A) cm −1 (B) cm −1
400 144
3RH 5R
(C) cm−1 (D) H cm−1
4 36
48. The longest  for the Lyman series is ….. (Given RH = 109678 cm–1):
(A) 1215 (B) 1315
(C) 1415 (D) 1515
49. The ratio of area covered by second orbital to the first orbital is
(A) 1 : 1 (B) 1 : 16
(C) 8 : 1 (D) 16 : 1
50. In spectral series of hydrogen, the series which does not come in infrared region is
(A) Pfund (B) Bracket
(C) Paschen (D) Lyman
51. The radius of electron in the first excited state of hydrogen atom is
(A) a0 (B) 4a0
(C) 2a0 (D) 8a0
52. According to the Bohr theory, which of the following transitions in the hydrogen atom will give rise to
the least energetic photon
(A) n = 6 to n = 5 (B) n = 5 to n = 3
(C) n = 6 to n =1 (D) n = 5 to n = 4
53. Which transition in the hydrogen atomic spectrum will have the same wavelength as the transition, n = 4
to n =2 of He+ spectrum
(A) n = 4 to n = 3 (B) n = 3 to n = 2
(C) n = 4 to n = 2 (D) n = 2 to n = 1
54. The value of Planck constant is
(A) 6.62  10–34 Joule-second (B) 6.62  10–27 Joule-second
(C)6.62  10–30 Joule-second (D) 6.62  10–24 Joule-second
55. The energy of an electron in the first Bohr orbit of H atom is –13.6 eV. The possible energy value(s) of
the excited state(s) for electrons in Bohr orbits to hydrogen is (are)
(A) –3.4 eV (B) –4.2 eV
(C) –6.8 eV (D) +6.8 eV

162
ATOMIC STRUCTURE CHEMISTRY

56. Calculate de-Broglie wavelength of an electron travelling at 1% of the speed of light


(A) 2.73  10–24 m (B) 2.42  10–10 m
(C) 242.2  1010 m (D) None of these
57. The de-Broglie equation applies
(A) To electrons only (B) To neutrons only
(C)To protons only (D) All the material object in motion
58. In Sommerfeld’s modification of Bohr’s theory, the trajectory of an electron in a hydrogen atom is
(A)a perfect circle.
(B) a closed ellipse-like curve, narrower on one side and flatter on the other side.
(C)a closed loop on spherical surface.
(D)None of these
59. The uncertainty in the position of an electron (mass = 9.1  10–28g) moving with a velocity of 3.0  104
cms–1 accurate up to 0.011% will be
(A)1.92 cm (B) 7.66 cm
(C) 0.175 cm (D) 3.84 cm
60. The uncertainties in the velocities of two particles A and B are 0.05 and 0.02 ms–1 respectively. The mass
 x A 
of B is five times to that of the mass of A. What is the ratio of uncertainties in their positions  ?
 x B 
(A) 2 (B)0.25
(C) 4 (D) 1
61. 2 = 0 represents
(A)node (B) orbital
(C) angular wave function (D)wave function
62. The total number of atomic orbitals in fourth energy level of an atom is
(A) 4 (B)8
(C) 16 (D) 32
63. In  321 the sum of angular momentum, spherical nodes and angular node is:
6h + 4 6h
(A) (B) +3
2 2
6h + 2 6h + 8
(C) (D)
2 2
64. Which of the following is not permissible arrangement of electrons in an atom?
1 1
(A) n = 3, l = 2, m = −3, s = − (B) n = 5, l = 3, m = 0, s = −
2 2
1 1
(C) n = 4, l = 0, m = 0, s = − (D) n = 3, l = 2, m = −2, s = −
2 2
65. The number of radial nodes of 3s and 2p orbital are
(A) 2,0 (B) 0,2
(C) 1,2 (D) 2,1
66. The orbital angular momentum of an electron in 2s-orbtial is
1 h h
(A) . (B) 2.
2 2 2
h
(C)zero (D)
2

163
CHEMISTRY ATOMIC STRUCTURE

67. The number of valence electrons in the Fe atom is:


(A) 5 (B) 6
(C) 7 (D) 8
68. Which statement is correct for an electron that has n = 4 and m = – 2?
(A)The electron may be in a d-orbital
(B) The electron is in the nth orbit
(C) The electron may be in a p-orbital
(D) The electron must have a spin quantum number + 1/2
69. Which set is a correct for an electron in 4f-orbitals
1 1
(A) n = 3, l = 2, ml = −2, ms = + (B) n = 4, l = 4, ml = −4, ms = +
2 2
1 1
(C) n = 4, l = 3, ml = +1, ms = + (D) n = 4, l = 3, ml = +4, ms = +
2 2
70. Which is the correct order of increasing energies of the listed orbitals in the atom of titanium?
(At no. Z =22)
(A) 3s, 4s, 3p, 3d (B) 4s, 3s, 3p, 3d
(C) 3s, 4p, 3d, 4s (D) 3s, 3p, 4s, 3d
71. The most stable orbitals are:
(A) p2 and d3 (B) p4 and d4
3 5
(C) p and d (D) d5 and d7
72. Which one of the following orbitals, has shape of a baby soother?
(A) d xy (B) d x2 − y 2
(C) d z 2 (D) p y
73. Which of the following corresponds to one node?

(A) (B)

(C) (D)
74. Which have the same number of s-electrons as the d-electrons in Fe2+?
(A) Li (B) Na
(C) N (D) P
75. ψ310 has:
(A) 1 radial node and 1 angular node (B) 2 radial node and 1 angular node
(C) 1 radial node and 2 angular node (D) 2 radial node and 2 angular node
76. Magnetic moment of Fea+ (Z = 26) is 24 BM. Hence number of unpaired electrons and value of a
respectively are:
(A) 4, 2 (B) 2, 4
(C) 3, 1 (D) 0, 2
77. Which one of the orbitals has zero probability of finding electrons in xz plane?
(A)px (B) py
(C)pz (D)dxz

164
ATOMIC STRUCTURE CHEMISTRY

78. Orbital angular momentum for 3d orbital electron is


h h
(A) 3 (B) 2
2 2
h
(C) 6 (D) 0
2
79. The momentum of a particle which has a de Broglie wavelength of 0.1 nm is (in kg ms–1)
(A) 7.08 × 10–21 (B) 6.64 × 10–24
(C) 6.58 × 10–14 (D) 5.89 × 10–9
kn 2
80. Rydberg given the equation for all visible radiation in the hydrogen spectrum as  = . The value
n2 − 4
of k in terms of Rydberg constant is
R
(A) 4R (B)
4
4
(C) (D) R
R
81. The series limit values are 8208Å and 22800 Å for the quantum numbers n1 and n2 in the atomic spectrum
of the hydrogen. What is the wavelength for the radiation emitted for the transition n2→n1 in Å?
(A) 15282 (B) 12258
(C) 12825 (D) 15822
82. Which of the following sets of quantum numbers represents the highest energy of an atom?
(AIEEE 2007)
(A) n = 3, l = 0, m = 0, s = +1/2 (B) n = 3, l = 1, m = 1, s = +1/2
(C) n = 3, l = 2, m = 1, s = +1/2 (D) n = 4, l = 0, m = 0, s = +1/2
83. Energy will be absorbed for the process of separating
(A) a proton from a proton (B) an electron from proton
(C) an electron from an electron (D) a neutron from a neutron
84. The ratio of the radius of the nucleus and the radius of the atom according to Rutherford’s model is:
(A) 105 (B) 1010
(C) 10– 15 (D) 10–5
85. X2― has 56 electrons, the atomic number X is
(A) 56 (B) 58
(C) 28 (D) 54
86. Acc. to Bohr’s calculation, on Rutherford’s model how much time would it take for electron to spiral on
to the nucleus:
(A) 10–1 s (B) 10–2 s
(C) 10–8 s (D) 10–10 s
87. The particle produced when -particles were bombarded on lighter elements like Beryllium and Boron:
(A) Electron (B) -Particle
(C) Neutron (D) Proton
88. The electrons that constitute the cathode rays are from:
(A) The cathode (B) Gas in the electrode
(C) Both (A) and (B) (D) None
89. The scientist who discovered neutrons was:
(A) Chadwick (B) Mosley
(C) J.J. Thomson (D) Millikan

165
CHEMISTRY ATOMIC STRUCTURE

90. Most of -particles in Rutherford’s scattering Exp. passed through metal foil without undergoing any
deflection because
(A) There is a lot of empty space within the atom.
(B) Nucleus occupies a very small space within the atom.
(C) Both (A) and (B)
(D) Nucleus is positively charged.
91. If the -scattering experiment has been conducted using a foil of lighter elements then:
(A) Number of deflections would be negligible (B) The number of deflections would be more
(C) The deflections would be through greater angles (D) The beam would retrace its path
92. The nuclear volume in an atom is as the order
(A) 10–30 m–3 (B) 10–60 m–3
(C) 10–45 m3 (D) 1030 m–3
93. Which out of the given species are isobars:
I : P = x , n = y +1 II : P = x + 1, n = y + 1 III : P = x + 1, n = y
(A) I and II (B) I and III
(C) II and III (D) none of the gives
94. Which of the following species is iso-electronic with CO?
(A) HF (B) N2
(C) N2+ (D) O2-
95. The correct statement about isotopes is :
(A) They have same mass number (B) They have different number of neutrons
(C) They have different chemical properties (D) They have different atomic numbers
96. Which of the following pairs are Isotones?
24
(A) 147 N and 168 O (B) 2311 Na and 12 Mg
40
(C) 11 H and 21 D (D) 4019 K and 20
Ca
97. From the following nuclei choose the isotopes:
I: 8p + 8n II: 8p + 9n III: 18p + 22n
(A) I and II (B) II and III
(C) I and III (D) None of the above
98. From the following, select the Isobars:
I: 7p + 7n II: 7P + 8n III: 6p + 8n IV: 6p + 6n
(A) I and III (B) I and II
(C) II and III (D) III and IV
99. During Muliken’s oil drop experiment, out of the following, which is not a possible charge on oil droplet?
(A) 1.6×10−19 C (B) 2.4×10−19 C
−19
(C) 3.2×10 C (D) 4.8×10−19 C
100. Yellow light is more energetic than
(A) Violet (B) blue
(C) Indigo (D) Red
101. The spectrum of white light ranging from red to violet is called a continuous spectrum because:
(A) Diff. colours are seen as diff. bands in spectrum.
(B) Colours continuously absorb energy from spectrum.
(C) Violet merges into blue, blue into green and so on.
(D) Continuous band of colours, white light separating them.
102. While cooling a solid object emits radiations:
(A) With inc. frequency & dec. intensity
(B) Intensity first inc. & then dec. and frequency dec.
(C) Frequency dec. but intensity remains const.
(D) Frequency inc. but intensity remains constant

166
ATOMIC STRUCTURE CHEMISTRY

103. The name of the negatively charged fundamental particle ‘electron’ was proposed by:
(A) J.J. Thomson (B) Stoney
(C) Bohr (D) Mullikan
104. Which has the highest e/m ratio?
(A) He2+ (B) H+
(C) He+ (D) H
105. Which is not true with respect to the cathode ray?
(A) A stream of electrons (B) Charged particles
(C) Move with the speed of light (D) Can be deflected by magnetic yields
106. The source of anode rays in the cathode ray tube is:
(A) The negative electrode (B) The positive electrode
(C) Zinc sulphide coating (D) The gas in the tube
107. In Millikan’s experiment, if charge on oil droplet was found to be –1.28210–18C, no. of electrons
captured on drop are:
(A) 8 (B) 9
(C) 4 (D) 10
108. Which of the following is the main cause of late discovery of a neutron?
(A) Neutrons is a highly unstable particle (B) Neutron in the nucleus move very fast
(C) Neutrons is a charge-less particle (D) All of these
109. Limitation of the Thomson’s ‘Raisin pudding model’ was
(A) It did not account for the neutrality of the atom.
(B) It did not account for the total mass of the atom.
(C) It did not explain the observations of Rutherford’s experiment
(D) It did not predict the existence of the neutron.
110. If the radius of the atom is of the order 10– 9 cm and radius of the nucleus is of the order 10– 14 cm. What
fraction of the volume of the atom is occupied by the nucleus.
(A) 10– 27 (B) 10– 15
– 42
(C) 10 (D) 10– 9
111. Select the species with is iso-electronic with Na+ :
(A) Ne (B) Ca+2
(C) Ar (D) S– 2
112. ‘Y’ is an ion with two units of negative charge and mass number is 18. If the number of neutrons are 25%
more that the protons then the number of electrons is:
(A) 6 (B) 8
(C) 10 (D) 2
113. Establish the relation between: 40 40 40
18 Ar, 19 K, 20 Ca

(A) Isobars (B) Isotones


(C) Isotopes (D) Isomorphs
114. When an electron drops from a higher energy level to a lower energy level then :
(A) energy is absorbed (B) energy is emitted
(C) atomic number increases (D) atomic number decreases
115. The potential energy of the electron in an orbit of H-atom would be
(A)−mv2 (B) −e2/r
(C) –mv /2
2
(D) −e2/2r

167
CHEMISTRY ATOMIC STRUCTURE

x
116. An electron in an atom jumps in such a way that its kinetic energy changes from x to .
4
The change in potential energy will be:
3 3
(A) + x (B) − x
2 8
3 3
(C) + x (D) − x
4 4
117. If force of attraction between the electron and nucleus in 2nd orbit of Li2+ is, force of attraction if
electron present in 1st orbit of H is
6 12
(A) f (B) f
49 25
8 16
(C) f (D) f
81 27
118. The spacing between the orbits in terms of distance is maximum in the case of
(A) 1st and 2nd (B) 2nd and 3rd
rd th
(C) 3 and 4 (D) 4th and 5th
119. If velocity of an electron in the 2 orbit in He is ‘V’ then the velocity of the electron in the 6th orbit is:
nd +

(A) V/3 (B) 3V


(C) 5V/4 (D) 9V
120. The expression of speed of electron in Bohr radii is given by
n ( h / 2 ) 2 ( Ze2 /4 0 )
(A) v = 2 (B) v =
Ze ( 4 0 ) nh
n 2 ( h / 2 ) 4 2 ( Ze2 /42 0 )
(C) v = 2 (D) v =
Ze / ( 4 0 ) n2 h2
121. Which state of Be3+ has the same orbit radius as that of the ground state of hydrogen atom?
(A) 3 (B) 2
(C) 4 (D) 5
122. The kinetic energy of an electron in the second Bohr orbit of a hydrogen atom [a0 is Bohr radius]
h2 h2
(A) (B)
4 2 ma02 16 2 ma02
h2 h2
(C) (D)
32 2 ma02 64 2 ma02
123. The difference of nth and (n+1)th Bohr`s radius of H-atom is equal to its (n -1)th Bohr`s radius. The value
of n is
(A) 1 (B) 2
(C) 3 (D) 4
124. The radius of which of the following orbit is same as that of the first Bohr’s orbit of hydrogen atom?
(A) He+ ( n = 2) (B) Li2+ ( n = 2)
(C) Li2+ (n =3) (D) None of these
125. The frequency of revolution of an electron in a circular Bohr orbit is:
(A) Directly proportional to n (B) Inversely proportional to n
3
(C) Inversely proportional to n (D) Directly proportional to n2

168
ATOMIC STRUCTURE CHEMISTRY

126. According to Bohr’s theory, angular momentum of electron in any orbit of Hydrogen is directly
proportional to
1 1
(A) (B)
rn rn
(C) rn 2 (D) rn
127. When an electron makes a transition from (n + 1) state to nth state, the frequency of emitted radiations is
related to n according to (n>> 1):
2cRZ 2 cRZ 2
(A) v = (B) v =
n3 n4
cRZ 2 2cRZ 2
(C) v = (D) v =
n2 n2
128. The first shell of H atom and the 2nd shell of Be+3 have same radii then for an electron moving in these
shells
(A) H  Be +3
(B) H = Be +3

(C) H  Be +3
(D) There is no direct relation
129. When electrons are de-exciting from nth orbit of hydrogen atoms, 15 spectral lines are formed. The
shortest wavelength among these will be
11 900
(A) R (B)
900 11R
35 36
(C) (D)
36R 35R
130. Which of the following statements is incorrect?
(A) Energy of radiation inc. with dec. in wavelength
(B) Spectrum of H atom is exactly same as that of He+ ion
(C) Energy of radiation increases with increase in v
(D) Frequency of radiation is related to wave no. as v = C v.
131. Ratio of the energies of Violet light (1 = 4.10  10–5 cm) to that of red light (2 = 6.56  10–5 cm) is :
(A) 1.6 : 1 (B) 3 : 2
(C) 2 : 1 (D) 5 : 2
132. The eyes of certain members of reptile family pass a visual signal to the brain when the visual receptors
are struck by photons of wavelength 890 nm. If a total energy of 3.15×10−14J is required to trip signal,
what is the minimum number of photons that must strike the receptor?
(A) 3.05×1019 (B) 1.72×109
(C) 1.41×105 (D) 2.75×1010
133. A dye absorbs a photon of wavelength  and re-emits the same energy into two photons of wavelength 1
and 2 respectively. The wavelength  is related with 1and 2 as:
1 + 2 12
(A)  = (B)  =
12 1 + 2
1222 12
(C)  = (D) =
1 + 2 ( 1 + 2 )
2

134. If the uncertainty x in the position is along X-axis, then uncertainty in the momentum is along
(A) X-axis (B) Y-axis
(C) Z-axis (D) Any axis

169
CHEMISTRY ATOMIC STRUCTURE

135. Heisenberg’s uncertainty principle has no significance at the macroscopic level because.
(A) Particles with a significant mass have negligible velocity
(B) The motion of the particle is not disturbed by the photons incident on it
(C) Particles with significant mass have a small wavelength
(D) Particles with significant mass have high velocities
136. The uncertainty in position assuming uncertainty in momentum within 0.1% for tennis ball weighing 0.2
Kg moving with a velocity of 10 ms– 1is :
(A) 2.63 10−32 m (B) 2.63 × 10–30 m
(C) 3.62 × 10–32 m (D) 3.62 × 10–30 m
137. If energy of a photon of frequency v is given by E = hv& momentum of photon is p = h /  , then velocity
of light is:
E E
(A) (B)
p p
2
E
(C) E  p (D)  
 p
138. If the uncertainty in the position of an electron is equal to its de Broglie wave length, then uncertainty in
its velocity is:
(A) 0.04% (B) 2.59 %
(C) 0.01 % (D) 7.96 %
139. The zero probability of finding the electron in p x orbital is :
(A) Maximum on two opposite of the nucleus along the X axis.
(B) In the nucleus
(C) Same on all the sides around the nucleus
(D) None of these
140. Out of the three known subshells which subshell rosette shaped
(A) Fundamental (B) Diffused
(C) Sharp (D) Principle
141. While revolving around the nucleus, in which subshell will the electron spend the maximum time close to
the nucleus.
(A) s subshell (B) p subshell
(C) d subshell (D) f subshell
142. The numbers of electrons associated with each of the following respectively is
(i) Principal quantum number = 2
(ii) Azimuthal quantum number = 2
(iii) Magnetic quantum number = 2
(A) 2, 2, 2 (B) 2, 8, 8
(C) 8, 10, 2 (D) 2, 10, 8
−1
143. In an atom the total number of electrons having quantum number: n = 4, ml = 1 and ms = is
2
(A) 5 (B) 4
(C) 6 (D) 2
144. Probability of finding the electron in the orbital is?
(A) 100% (B) 5-10%
(C) 90-95% (D) 50-60%
145. The azimuthal quantum number for the twentieth electron in Manganese atom is :
(A) 0 (B) 1
(C) 2 (D) 3

170
ATOMIC STRUCTURE CHEMISTRY

146. Which quantum number determines shape of the orbital?


(A) Principal (B) Angular
(C) Magnetic (D) Spin
147. Total number of electrons in any orbit is:
l= n l = n −1
(A)  2(2l + 1) (B)  2(2 l + 1)
l =1 l =1
l = n +1 l = n −1
(C)  2(2 l + 1) (D)  2(2 l + 1)
l =0 l =0
148. The number of radial nodes in 4s orbitals is
(A) 2 (B) 3
(C) 4 (D) 5
149. The following quantum numbers are possible for how many orbitals? n = 3, l = 2, m = +2
(A) 3 (B) 2
(C) 1 (D) 4
150. Which orbital notation does not have a spherical node?
(A) n = 2; l = 0 (B) n = 2, l = 1
(C) n = 3, l = 0 (D) n = 1, l = 0
151. The nodal plane for py orbital is
(A) XZ plane (B) XY plane
(C) YZ plane (D) There is no nodal plane
152. Which of the following orbitals will have zero probability of finding the electron in yz plane?
(A) px (B) py
(C) pz (D) dyz
153. The probability of finding 1s electron is maximum
(A) Near the nucleus
(B) At a distance equal to Bohr radius from the nucleus
(C) At infinite distance from the nucleus
(D) At a distance equal to twice of the Bohr radius from nucleus.
154. The number of radial nodes in 4s orbitals is
(A) 2 (B) 3
(C) 4 (D) 5
155. The five d-orbitals are designated as d xy , d yz , d xz , d x2 − y2 and d z 2 . Choose the correct statement
(A) The shapes of the first three orbitals are similar but that of the fourth and fifth orbitals are different.
(B) The shapes of all the five d-orbitals are similar
(C)The shape of the first four orbitals are similar but that of the fifth orbitals is different
(D) The shapes of all the five d- orbitals are different
156. Wave function vs distance from nucleus graph of an orbital is shown.

The number of nodal sphere of this orbital is


(A) 1 (B) 2
(C) 3 (D) 4

171
CHEMISTRY ATOMIC STRUCTURE

157. The orbital angular momentum quantum number of an electron in the 3s orbital is :
(A) 0 (B) 1
(C) 2 (D) 3
158. The number of orbitals in a given sub-shell can be calculated by the formula:
(A) 2l + 1 (B) l + 1
(C) l (D) l 2
159. For a d-electron, the orbital angular momentum is
 h   h 
(A) 6  (B) 2 
 2   2 
 h 
(C) 
 h 
 (D) 2  
 2   2 
160. Number of electron in a sub-shell can be calculated from the formula.
(A) 2n2 (B) 2l + 1
(C) 2 ( 2l + 1) (D) n 2
161. When electrons are placed in 2 degenerate orbitals the energy is lower when spin are parallel. Statement
is based upon:
(A) Pauling exclusion principles (B) Bohr’s rule
(C) Hund’s rule (D) Aufbau principle
162. Which will have more number of unpaired electrons : Fe+2 or Fe+3
+2
(A) Fe (B) Fe+3
(C) Both have same number of unpaired electrons (D) Both do not have any unpaired electron
163. The number of electrons in the 3d orbital of Cu is :
(A) 2 (B) 1
(C) 10 (D) 9
164. How many electrons at the most can have n + l = 8 (n 7)
(A) 30 (B) 20
(C) 16 (D) 18
165. The energy of an electron of 2py orbital is
(A) Greater than 2px orbital (B) Less than 2pz orbital
(C) Equal to 2s orbital (D) Same as that of 2px and 2pz orbital
166. The aufbau principal implies that a new electron will enter an orbital for which:
(A) n has a lower value (B) l has a lower value
(C) (n + l) value is maximum (D) (n + l) value is minimum
167. Tick the correct statement on the aufbau principle?
(A) ( n −1) d subshell is always lower in energy than ns subshell
(B) ( n − 1) d subshell always has energy more than ns subshell
(C) 5d is lower in energy than 4f
(D) 5f is lower in energy than 7s
168. Which of the following sets of quantum numbers, represents the 19th electron of chromium?
n l m s n l m s
(A) 4 1 –1 1/2 (B) 4 0 0 1/2
(C) 3 2 –2 1/2 (D) 3 2 0 ½

172
ATOMIC STRUCTURE CHEMISTRY

169. The atomic number of the element having outer electronic configuration (3d)5 (4s)1 is
(A) 24 (B) 25
(C) 26 (D) 27
170. The conclusion that every additional electron enters the orbital with lowest possible energy has been
drawn from
(A) Pauli’s exclusion principle (B) Hund’s rule
(C) Aufbau principle (D) De-Brogile’s equation
171. Which series of subshells is arranged in the order of increasing energy for multi-electron atoms?
(A) 6s, 4f, 5d, 6p (B) 4f, 6s, 5d, 6p
(C) 5d, 4f, 6s, 6p (D) 4f, 5d, 6s, 6p
172. Correct energy value order is
(A) ns np nd ( n − 1) f (B) ns np ( n −1) d ( n − 2) f
(C) ns np ( n −1) d ( n −1) f (D) ns ( n −1) d np ( n −1) f
173. Which set of quantum numbers (n, l, m, s) represents the outermost electron in a gaseous aluminium atom?
1 1
(A)2, 1, 0, + (B)2, 1, −1, +
2 2
1 1
(C)3, 0, 0, + (D)3, 1, −1, +
2 2
174. Which of the following statements is correct?
(A)(n − 1) d sub−shell has lower energy than ns sub−shell.
(B)(n − 1) d sub−shell has higher energy than ns sub−shell.
(C)(n + 1) d sub−shell has lower energy than nf sub−shell
(D)nf sub−shell has lower energy than (n + 2)s sub−shell.
175. Which of the following element is represented by electronic configuration 1s 2 2s 2 2 p1x 2 p1y 2 p1z ?
(A) Nitrogen (B) Oxygen
(C) Fluorine (D) Sulphur

173
CHEMISTRY ATOMIC STRUCTURE

174
ATOMIC STRUCTURE CHEMISTRY

1. Given below are two statements : [JEE (M) 2021]


Statement-I: Two photons having equal linear momenta have equal wavelengths.
Statement-II: If the wavelength of photon is decreased, then the momentum and energy of a photon will
also decrease.
In the light of the above statements, choose the correct answer from the options given below.
(A) Statement-I is false but Statement-II is true
(B) Both Statement-I and Statement-I are true
(C) Both Statement-I and Statement-I are false
(D) Statement-I is true but Statement-II is false
2. A proton and a Li3+ nucleus are accelerated by the same potential. If Li and p denote the de Broglie
λ Li
wavelengths of Li3+ and proton respectively, then the value of is x  10–1. The value of x is ______.
λp
[Mass of Li3+ = 8.3 mass of proton] [JEE (M) 2021]
3. According to Bohr's atomic theory : [JEE (M) 2021]
z2
(a) Kinetic energy of electron is  2
n
(b) The product of velocity (v) of electron and principal quantum number (n). 'vn'  z 2 .
z3
(c) Frequency of revolution of electron in an orbit is  .
n3
z3
(d) Coulombic force of attraction on the electron is  4 . Choose the most appropriate answer from the
n
options given below:
(A) (c) only (B) (a) and (d) only
(C) (a) only (D) (a), (c) and (d) only
4. The plots of radial distribution functions for various orbitals of hydrogen atom against 'r' are given
below: [JEE (M) 2021]

(a) (b)

(c) (d)
The correct plot for 3 s orbital is:
(A) d (B) b
(C) a (D) c

175
CHEMISTRY ATOMIC STRUCTURE

5. Electromagnetic radiation of wavelength 663nm is just sufficient to ionize the atom of metal A. The
ionization energy of metal A in kJ mol–1 is_________. [h — 6.63  10–34 Js, c = 3.00 x 108 ms–1, NA =
6.02  1023 mol–1] [JEE (M) 2021]
6. The orbital having two radial as well as two angular nodes is [JEE (M) 2021]
(A) 5d (B) 4f
(C) 3p (D) 4d
7. A ball weighing 10 g is moving with a velocity of 90 ms–1. If the uncertainty in its velocity is 5%, then
the uncertainty in its position is _______ 10–33 m. [Given : h = 6.63  10–34 Js] [JEE (M) 2021]
8. The figure that is not a direct manifestation of the quantum nature of atoms is [JEE (M) 2020]

(A) (B)

(C) (D)
9. The number of subshells associated with n = 4 and m = –2 quantum numbers is [JEE (M) 2020]
(A) 2 (B) 8
(C) 4 (D) 16
10. The work function of sodium metal is 4.41  10 J. If photons of wavelength 300nm are incident on the
–19

metal, the kinetic energy of the ejected electrons will be (h = 6.63  10–34 Js ; c = 3  108 m/s)
___________  10–21 J [JEE (M) 2020]
11. Consider the hypothetical situation where the azimuthal quantum number, I, takes value 0, 1, 2, ... n + 1,
where n is the principal quantum number. Then, the element with atomic number [JEE (M) 2020]
(A) 9 is the first alkali metal (B) 6 has a 2p -valence subshell
(C) 8 is the first noble gas (D) 13 has a half-filled valence subshell
12. The region in the electromagnetic spectrum where the Balmar series lines appear is [JEE (M) 2020]
(A) Microwave (B) Ultraviolet
(C) Visible (D) Infrared
13. The shortest wavelength of H atom in the Lyman series is 1. The longest wavelength in the Balmar
series of He+ is [JEE (M) 2020]
5λ1 36λ1
(A) (B)
9 5
27λ1 9λ1
(C) (D)
5 5
14. In the sixth period, the orbitals that are filled are [JEE (M) 2020]
(A) 6s, 4f, 5d, 6p (B) 6s, 5d, 5f, 6p
(C) 6s, 6p, 6d, 6f (D) 6s, 5f, 6d, 6p
15. The correct statement about probability density (except at infinite distance from nucleus) is
[JEE (M) 2020]
(A) It can never be zero for 2s orbital (B) It can be zero for 3p orbital
(C) It can be zero for 1 s orbital (D) It can be negative for 2p orbital

176
ATOMIC STRUCTURE CHEMISTRY

16. Amongst the following which is not a postulate of Dalton's atomic theory [JEE (M) 2020]
(A) Matter is formed of indivisible atoms
(B) When gases combine or reproduced in a chemical reaction they do so in a simple ratio by volume
provided all gases are at the same T &P.
(C) During chemical reactions atoms remains conserved and only pass through rearrangement
(D) Some atoms have same properties including atomic mass
17. The correct order of the spin-only magnetic moments of the following complexes is : [JEE (M) 2020]
(I) [Cr(H2O)6] Br2
(II) Na4 [Fe(CN)6]
(III) Na3 [Fe(C2O4)3] (0 > P)
(IV) (Et4N)2 [CoCl4]
(A) (III) > (I) > (IV) > (II) (B) (III) > (I) > (II) > (IV)
(C) (I) > (IV) > (III) > (II) (D) (II)  (I) > (IV) > (III)
1 1
18. For the Balmer series in the spectrum of H atom, v = R H  2
− 2  , the correct statements among (l) to
 n1 n 2 
(IV) are: [JEE (M) 2020]
(I) As wavelength decreases, the lines in the series converge
(II) The integer n1 is equal to 2
(III) The lines of longest wavelength corresponds to n 2 = 3
(A) (I), (III), (IV) (B) (I), (II), (IV)
(C) (II), (III), (IV) (D) (I), (II), (III)
th
19. The de Broglie wavelength of an electron in the 4 Bohr orbit is [JEE (M) 2020]
(A) 4 πa 0 (B) 2 πa 0
(C) 6 πa 0 (D) 8 πa 0

20. The radius of the second Bohr orbit, in terms of the Bohr radius, a 0 in Li2+ is: [JEE (M) 2020]
2a 0 4a 0
(A) (B)
3 9
4a 2a
(C) 0 (D) 0
3 9
1
21. The number of orbitals associated with quantum number n = 5, ms = + is: [JEE (M) 2020]
2
(A) 11 (B) 25
(C) 50 (D) 15
 1 
22. For emission line of atomic hydrogen from ni = 8 to nf = n the plot of wave number ( v ) against  2 
n 
will be (The Rydberg constant, RH is in wave number unit). [JEE (M) 2019]
(A) Linear with slope - RH (B) Linear with intercept - RH
(C) Non linear (D) Linear with slope RH

177
CHEMISTRY ATOMIC STRUCTURE

23. Which of the following combination of statements is true regarding the interpretation of the atomic
orbitals? [JEE (M) 2019]
(a) An electron in an orbital of high angular momentum stays away from the nucleus than an electron in
the orbital of lower angular momentum.
(b) For a given value of the principal quantum number, the size of the orbit is inversely proportional to
the azimuthal quantum number.
h
(c) According to wave mechanics, the ground state angular momentum is h equal to .
2
(d) The plot of  Vs r for various azimuthal quantum numbers, shows peak shifting towards higher r
value.
(A) (b), (c) (B) (a), (d)
(C) (a), (b) (D) (a), (c)
24. Which of the graphs shown below does not represent the relationship between incident light and the
electron ejected form metal surface? [JEE (M) 2019]

(A) (B)

(C) (D)

25. The 71st electron of an element X with an atomic number of 71 enters into the orbital: [JEE (M) 2019]
(A) 4f (B) 6p
(C) 6s (D) 5d
26. The ground state energy of hydrogen atom is –13.6 eV. The energy of second excited state He+ ion in eV
is: [JEE (M) 2019]
(A) –6.04 (B) –27.2
(C) –54.4 (D) –3.4
27. Heat treatment of muscular pain involves radiation of wavelength of about 900 nm. Which spectral line
of H-atom is suitable for this purpose? [JEE (M) 2019]
5 –1 –34 8 –1
[RH = 1 × 10 cm , h = 6.6 × 10 Js,c = 3 × 10 ms ]
(A) Paschen, 5 → 3 (B) Paschen, → 3
(C) Lyman, → 1 (D) Balmer, →2
28. The de Broglie wavelength () associated with a photoelectron varies with the frequency (v)of the
incident radiation as, [v0 is threshold frequency]: [JEE (M) 2019]
1 1
(A)   3
(B)   1
( v − v0 ) 2 ( v − v0 ) 2
1 1
(C)   (D)  
( v − v0 )
1
4
( v − v0 )

178
ATOMIC STRUCTURE CHEMISTRY

29. What is the work function of the metal if the light of wavelength 4000 Å generates photoelectrons of
velocity 6 × 105 ms–1 from it? (Mass of electron = 9 × 10–31 kg; Velocity of light = 3 × 108 ms–1; Planck's
constant = 6.626 × 10–34 Js; Charge of electron = 1.6 × 10–19 JeV–1) [JEE (M) 2019]
(A) 0.9 eV (B) 4.0 eV
(C) 2.1 eV (D) 3.1 eV
30. If the de Broglie wavelength of the electron in n Bohr orbit in a hydrogenic atom is equal to 1.5 a0(a0 is
th

Bohr radius), then the value of n/z is: [JEE (M) 2019]
(A) 1.0 (B) 0.75
(C) 0.40 (D) 1.50
31. Ejection of the photoelectron from metal in the photoelectric effect experiment can be stopped by applying
0.5 V when the radiation of 250 nm is used. The work function of the metal is: [JEE (M) 2018]
(A) 5 eV (B) 4 eV
(C) 5.5 eV (D) 4.5 eV
32. The de-Broglie’s wavelength of electron present in first Bohr orbit of ‘H’ atom is:[JEE (M) 2018]
0.529
(A) Å (B) 2  0.529 Å
2
(C) 0.529 Å (D) 4  0.529 Å
33. Which of the following statements is false? [JEE (M) 2018]
(A) Photon has momentum as well as wavelength
(B) Splitting of spectral lines in electrical field is called Stark effect
(C) Frequency of emitted radiation from a black body goes from a lower wavelength to higher
wavelength as the temperature increases
(D) Rydberg constant has unit of energy.
34. The radius of the second Bohr orbit for hydrogen atom is: [JEE (M) 2017]
(Plank’s constant. h = 6.6262  10 Js; mass of electron = 9.1091  10 kg; charge of electron. e =
–34 –31

160210  10–19 C; permittivity of vacuum. 0 = 8.854185  10–12 kg–1m–3A2)


(A) 1.65Å (B) 4.76Å
(C) 0.529Å (D) 2.12Å
35. A stream of electrons from a heated filament was passed through two charged plates kept at a potential
h
difference V esu. If e and m are charge and mass of an electron, respectively, then the value of (where

 is wavelength associated with electron wave) is given by: [JEE (M) 2016]
(A) meV (B) 2meV
(C) meV (D) 2meV
36. Which of the following is the energy of a possible excited state of hydrogen? [JEE (M) 2015]
(A) –3.4 eV (B) +6.8 eV
(C) +13.6 eV (D) –6.8 eV
37. In an atom how many orbital(s) will have the quantum numbers n = 3, l = 2 and ml = +2?
[JEE (M) 2013]
(A) 5 (B) 3
(C) 1 (D) 7

179
CHEMISTRY ATOMIC STRUCTURE

−18  Z2 
38. Energy of an electron is given by E = −2.178  10 J  2  . Wavelength of light required to excite an
n 
electron in an hydrogen atom from level n = 1 to n = 2 will be: [JEE (M) 2013]
(h = 6.62  10 Js and c = 3.0  10 ms )
–34 8 –1

(A) 1.214  10–7 m (B) 2.816  10–7 m


(C) 6.500  10–7 m (D) 8.500  10–7 m
39. The electrons identified by quantum numbers n and l: [2012]
(a) n = 4, l = 1 (b) n = 4, l = 0 (c) n = 3, l = 2 (d) n = 3, l = 1
can be placed in order of increasing energy as:
(A) (c) < (d) < (b)< (a) (B) (d) < (b) < (c)< (a)
(C) (b) < (d) < (a)< (c) (D) (a) < (c) < (b)< (d)
40. Ionisation energy of He is 19.6  10 J atom . The energy of the first stationary state (n = 1) of Li2+ is:
+ –18 –1

[2010]
(A) 4.41  10 J atom
–16 –1
(B) –4.41  10 J atom
–17 –1

(C) –2.2  10 J atom


–15 –1
(D) 8.82  10–17 J atom–1

180
ATOMIC STRUCTURE CHEMISTRY

175
1. The number of protons, neutrons and electrons in 71 Lu, respectively, are : [NEET 2020]
(A) 104, 71 and 71 (B) 71, 71 and 104
(C) 175, 104 and 71 (D) 71, 104 and 71
2. The calculated spin only magnetic moment of Cr2+ ion is [NEET 2020]
(A) 4.90 BM (B) 5.92 BM
(C) 2.84BM (D) 3.87BM
3. Which of the following series of transitions in the spectrum of hydrogen atom fall in visible region?
[NEET 2019]
(A) Lyman series (B) Blamer series
(C) Paschen series (D) Brackett series
4. 4d, 5p, 5f and6p orbitals are arranged in the order of decreasing energy. The correct option is
[NEET 2019]
(A) 5f > 6p 5p > 4d (B) 6p > 5f > 5p > 4d
(C) 6p > 5f > 4d > 5p (D) 5f < 6p > 4d > 5p
5. Match the metal ions given in Column I with the spin magnetic moments of the ions given in Column II
and assign the correct code: [NEET 2018]
Column I Column II
3+
(a) Co (i) 8 BM
(b) Cr3+ (ii) 35 BM
3+
(c) Fe (iii) 3 BM
(d) Ni2+ (iv) 24 BM
(v) 15 BM
a b c d a b c d
(A) (iv) (i) (ii) (iii) (B) (i) (ii) (iii) (iv)
(C) (iv) (v) (ii) (i) (D) (iii) (v) (i) (ii)
6. Magnesium reacts with an element (X) to form an ionic compound. If the ground state electronic
configuration of (X) is 1s2 2s2 2p3,the simplest formula for this compound is [NEET 2018]
(A) Mg2X (B) MgX2
(C) Mg2X3 (D) Mg3X2
7. Which one is the wrong statement? [NEET 2017]
h
(A) de-Broglie's wavelength is given by  = , where m = mass of the particle, v = group velocity of the particle
mv
h
(B) The uncertainty principle is E  t 
4
(C) Half-filled and fully filled orbitals have greater stability due to greater exchange energy, greater
symmetry and more balanced arrangement
(D) The energy of 2s orbital is less than the energy of 2p orbital in case of Hydrogen like atoms

181
CHEMISTRY ATOMIC STRUCTURE

8. The electronic configurations of Eu (Atomic no. 63),Gd (Atomic No. 64) and Tb (Atomic No 65) are
[NEET 2016]
7 2 8 2 8 1 2
(A) [Xe]4f 6s , [Xe]4f 6s and [Xe]4f 5d 6s (B) [Xe]4f 5d 6s , [Xe]4f75f1 and [Xe]4f96s2
6 1 2

(C) [Xe]4f65d16s2, [Xe]4f75d16s2 and [Xe]4f85d16s2 (D) [Xe]4f76s2, [Xe]4f75d16s2 and [Xe]4f96s2
9. Two electrons occupying the same orbital are distinguished by [NEET 2016]
(A) Principal quantum number (B) Magnetic quantum number
(C) Azimuthal quantum number (D) Spin quantum number
10. How many electrons can fit in the orbital for which n = 3 and l = 1? [NEET 2016]
(A) 2 (B) 6
(C) 10 (D) 14
11. Gadolinium belongs to 4f series. It’s atomic number is 64. Which of the following is the correct
electronic configuration of gadolinium? [AIPMT 2015]
8 2 9 1
(A) [Xe]4f 6d (B) [Xe]4f 5s
7 1 2
(C)[Xe]4f 5d 6s (D) [Xe]4f65d25s2
12. Which is the correct order of increasing energy of the listed orbitals in the atom of titanium?
[AIPMT 2015]
(A) 3s 4s 3p 3d (B) 4s 3s 3p 3d
(C)3s 3p 3d 4s (D) 3s 3p 4s 3d
13. What is the maximum number of orbitals that can be identified with the following quantum numbers?
(n = 3, l = 1, ml = 0) [AIPMT 2014]
(A) 1 (B) 2
(C)3 (D) 4
14. Calculate the energy in joule corresponding to light of wavelength 45 nm: [AIPMT 2014]
(Planck’s constant h = 6.63  10–34 Js; speed of light c = 3  108 ms–1)
(A) 6.67  1015 (B) 6.67  1011
(C)4.42  10–15 (D) 4.42  10–18
15. Be2+ is isoelectronic with which of the following ions? [AIPMT 2014]
+ +
(A) H (B) Li
(C) Na+ (D) Mg2+
 Z2 
16. Based on equation E = −2.178  10−18 J  2  , certain conclusions are written. Which of them is not
n 
correct? [NEET 2013]
(A) Larger the value of n, the larger is the orbit radius.
(B) Equation can be used to calculate the change in energy when the electron changes orbit.
(C)For n = 1, the electron has a more negative energy than it does for n = 6 which mean that the electron
is more loosely bound in the smallest allowed orbit.
(D) The negative sign in equation simply means that the energy or electron bound to the nucleus is lower
than it would be if the electrons were at the infinite distance from the nucleus.

182
ATOMIC STRUCTURE CHEMISTRY

17. The value of Planck’s constant is 6.63  10–34 Js. The speed of light is 3  1017 nm s–1. Which value is closest
to the wavelength in nanometer of a quantum of light with frequency of 6  1015 s–1? [NEET 2013]
(A) 25 (B) 50
(C)75 (D) 10
18. The orbital angular momentum of a p-electron is given as: [2012 M]
h h
(A) (B) 3
2 2
3 h h
(C) (D) 6
2 2
19. If n = 6, the correct sequence for filling of electrons will be: [2011]
(A) ns→ (n – 2)f→ (n – 1)d→np (B) ns→ (n – 1)d→ (n – 2)f→np
(C)ns→ (n – 2)f→np→ (n – 1)d (D) ns→np→ (n – 1)d→ (n – 2)f
20. The energies E1 and E2 of two radiations are 25 eV and 50 eV, respectively. The relation between their
wavelengths i.e., 1 and 2 will be: [2011]
(A) 1 = 2 (B) 1 = 22
1
(C) 1 = 42 (D) 1 = 2
2
21. The total number of atomic orbitals in fourth energy level of an atom is: [2011]
(A) 8 (B) 16
(C)32 (D) 4

22. Maximum number of electrons in a subshell of an atom is determined by the following: [2009]
(A) 2l + 1 (B) 4l – 2
2
(C)2n (D) 4l + 2
23. The energy absorbed by each molecule (A2) of a substance is 4.4  10–19 J and bond energy per molecule
is 4.0  10–19 J. The kinetic energy of the molecule per atom will be: [2009]
(A) 2.2  10 –19
J (B) 2.0  10 –19
J
(C)4.0  10–20 J (D) 2.0  10–20 J
24. The orientation of an atomic orbital is governed by [2006]
(A) principal quantum number (B) azimuthal quantum number
(C) spin quantum number (D) magnetic quantum number
25. The energy of second Bohr orbit of the hydrogen atom is –328 kJ mol–1; hence the energy of fourth Bohr
orbit would be: [2005]
–1 –1
(A) –41 kJ mol (B) –82 kJ mol
(C)–164 kJ mol–1 (D) –1312kJ mol–1
26. Which of the following pairs present isotones? [AIIMS 2014]
77 78 195 190
(A) 33 As, 34 Se (B) 78 Pt, 76 Os
108 112 178 137
(C) 47 Ag, 48 Cd (D) 72 Hf, 56 Ba

183
CHEMISTRY ATOMIC STRUCTURE

27. A particle is moving 3 times faster than the speed of electron. If the ratio of wavelength of particle and
electron is 1.8  10–4, then particle is [AIIMS 2013]
(A) Neutron (B) -particle
(C)Deuteron (D) Tritium
28. Smallest wavelength occurs for: [AIIMS 2011]
(A) Lyman series (B) Balmer series
(C) Paschen series (D) Brackett series
29. Assume that you are travelling at a speed of 90 km/h in a small car with a mass of 1050 kg. If the
uncertainty in the velocity of the car is 1% (v = 0.9 km/h), what is the uncertainty (in meters) in the
position of the car? [AIIMS 2010]
−35 −37
(A) x  110 m (B) x  2 10 m
(C) x  2 10−36 m (D) x  4 10−38 m
30. In the ground state of Cu+, the number of shells occupied, subshells occupied, filled orbitals and unpaired
electrons respectively are [AIIMS 2010]
(A) 4, 8, 15, 0 (B) 3, 6, 15, 1
(C)3, 6, 14, 0 (D) 4, 7, 14, 2

184

You might also like